2ちゃんねる ■掲示板に戻る■ 全部 1- 最新50    

■ このスレッドは過去ログ倉庫に格納されています

分からない問題はここに書いてね443

1 :132人目の素数さん:2018/05/04(金) 23:15:16.29 ID:rvnZA+Ji.net
さあ、今日も1日がんばろう★☆

前スレ
分からない問題はここに書いてね442
https://rio2016.5ch.net/test/read.cgi/math/1522418128/

2 :132人目の素数さん:2018/05/04(金) 23:38:55.65 ID:PXR1I5Eh.net
空海と老子はどっちの方が頭が良いですか?

3 :132人目の素数さん:2018/05/05(土) 00:04:01.88 ID:BUSpq5hZ.net
削除依頼を出しました

4 :132人目の素数さん:2018/05/05(土) 02:28:14.85 ID:cos8i+vX.net
>>1
スレ立て乙

[前スレ.991] [前スレ.996] を一般化

nが奇数のとき
 f(x) = {x(1-x)}^{(n-1)/2} (2x-1),
 ∫[0,1] {x(1-x)}^(n-1) (2x-1)^2 dx = 2 n! (n-1)! / (2n+1)!

nが偶数(n≧4) のとき
 f(x) = {x(1-x)}^(n/2 -1) (2x-1)^2,
 ∫[0,1] {x(1-x)}^(n-2) (2x-1)^4 dx = 12 n! (n-2)! / (2n+1)!

n=2 のとき
 f(x) = x(1-x),
 1/30

最小ぢゃねゑだろうが…

5 :132人目の素数さん:2018/05/05(土) 03:28:10.17 ID:NwM7wsxx.net
{3^n+2^(n+1)}/{2^n+1}
が整数となるような自然数nをすべて決定せよ。

6 :132人目の素数さん:2018/05/05(土) 06:52:13.42 ID:wrHy7dhw.net
>>5
10000まで調べてひとつもない。問題合ってる?

7 :132人目の素数さん:2018/05/05(土) 07:43:01.59 ID:NY9sEJP5.net
n=0

8 :132人目の素数さん:2018/05/05(土) 08:17:59.87 ID:NwM7wsxx.net
>>6
合ってるよ
「すべて求めよ」には「1つもない」も含まれる

9 :132人目の素数さん:2018/05/05(土) 09:06:20.58 ID:UvwWzyVD.net
ほーん
なんで答え知っとんのや
スレタイ読める?

10 :132人目の素数さん:2018/05/05(土) 09:52:45.87 ID:tEdcrB57.net
答えなんて書かれてない

11 :132人目の素数さん:2018/05/05(土) 10:07:42.78 ID:1elA/pW/.net
n,pを自然数とするとき
np^n=p^(n+1)
のpを満たす自然数を求めよ

12 :132人目の素数さん:2018/05/05(土) 10:16:18.59 ID:NY9sEJP5.net
無理

13 :132人目の素数さん:2018/05/05(土) 10:25:29.02 ID:y3sGc/sx.net
>>11
p=n

14 :132人目の素数さん:2018/05/05(土) 12:09:43.97 ID:1elA/pW/.net
>>13 あたり
じゃあnしかないことの証明は?

15 :132人目の素数さん:2018/05/05(土) 12:14:15.76 ID:NY9sEJP5.net
p=0

16 :132人目の素数さん:2018/05/05(土) 13:10:32.85 ID:K6Vuq8aH.net
>>15 自然数じゃないっすね

17 :132人目の素数さん:2018/05/05(土) 13:12:14.09 ID:tmq4RMMx.net
p=q

18 :132人目の素数さん:2018/05/05(土) 13:19:10.86 ID:GyiImAHu.net
>>14
スレタイ読めない文盲はタヒねカス

19 :132人目の素数さん:2018/05/05(土) 14:22:48.89 ID:UMWHPgmy.net
>>5が出来る気がしない。これ元ネタなんだろう?数オリ系?

20 :132人目の素数さん:2018/05/05(土) 14:31:43.90 ID:NwM7wsxx.net
周長が1の円Cと、周長が1の凸n角形Knがある。ただしn≧3である。
Knの周及び内部の領域をDnとするとき、C全体をDnに含めることは不可能であることを示せ。

21 :132人目の素数さん:2018/05/05(土) 14:32:38.00 ID:NwM7wsxx.net
>>19
マスターデーモンらしい

22 :132人目の素数さん:2018/05/05(土) 15:24:47.69 ID:UMWHPgmy.net
>>21
何すかソレ?

23 :132人目の素数さん:2018/05/05(土) 17:01:02.48 ID:EMULr+4U.net
>>5
k:={3^n+2^(n+1)}/{2^n+1}-2=(3^n-2)/(2^n+1)が自然数となる自然数nを求めることが必要十分.
kが自然数と仮定する.
3^n-2=k(2^n+1)…@
@の両辺をmod 2で考えkは奇数2l-1.
@の両辺をmod 3で考えnは偶数2mでありl≡2(mod 3).k=6r-1とかける.
3^{2m}-2=(6r-1)(2^{2m}+1)の両辺をmod 6で考えmは偶数2s.
3^{4s}-2=(6r-1)(2^{4s}+1)の両辺をmod 5で考えr≡2(mod 5).k=30t-19とかける.
3^{4s}-2=(30t-19)(2^{4s}+1)の両辺をmod 10で考え,8≡6^s.
このような非負整数sは存在しない.よって{3^n+2^(n+1)}/{2^n+1}が整数となるnは存在しない.

24 :132人目の素数さん:2018/05/05(土) 17:07:06.96 ID:NY9sEJP5.net
0は自然数ですよ。

25 :132人目の素数さん:2018/05/05(土) 17:28:15.22 ID:vn0wm8Ig.net
基礎論以外の分野では自然数に0は含まれないと思います

26 :132人目の素数さん:2018/05/05(土) 17:34:52.58 ID:NwM7wsxx.net
連続関数に対しては必ず、任意の区間での定積分が定義できますか?

27 :132人目の素数さん:2018/05/05(土) 17:41:58.45 ID:vn0wm8Ig.net
できますね

28 :132人目の素数さん:2018/05/05(土) 17:42:09.76 ID:/CiIYXay.net
もしかしたらクソ簡単で叩かれるかも知れませんけど、この文の意味がわかりません。
最大数の集合ってことですか?

https://i.imgur.com/1DL0o0c.jpg

29 :132人目の素数さん:2018/05/05(土) 17:44:26.32 ID:vn0wm8Ig.net
左と右どっちが大きいですかってことですね

30 :132人目の素数さん:2018/05/05(土) 17:50:43.82 ID:/CiIYXay.net
>>29
εの値を変化させて、最大数が1 - ε/2の集合と1/2の集合を作りますよ ってことですか?

31 :132人目の素数さん:2018/05/05(土) 17:53:15.41 ID:vn0wm8Ig.net
1-ε/2と1/2の大きい方をaεとするということです

32 :132人目の素数さん:2018/05/05(土) 17:55:58.18 ID:/CiIYXay.net
>>31
てっきり集合のことかと勘違いしてました
スレの流れを邪魔してごめんなさい

33 :132人目の素数さん:2018/05/05(土) 18:12:45.84 ID:/CiIYXay.net
A:=[0,1)に対して sup=1 となることを証明せよ

34 :132人目の素数さん:2018/05/05(土) 18:14:27.77 ID:vn0wm8Ig.net
自明ですね

35 :132人目の素数さん:2018/05/05(土) 18:22:30.39 ID:/CiIYXay.net
>>34
この定理を利用して証明して欲しいです

(2)の方を証明するために、さっき質問したようにaεを定義して利用しろと言われているんですが、この先が分かりません
https://i.imgur.com/Ktd7e1o.jpg

36 :132人目の素数さん:2018/05/05(土) 18:22:38.64 ID:NY9sEJP5.net
>>25
>基礎論以外の分野では自然数に0は含まれないと思います
ウゾだーー

37 :132人目の素数さん:2018/05/05(土) 18:23:17.62 ID:y3sGc/sx.net
>>14
バカ?

38 :132人目の素数さん:2018/05/05(土) 18:47:19.33 ID:vn0wm8Ig.net
>>35
任意のεを選択したとき、1-ε<aε<≦1
となることを示しましょう
aεが必ず[0,1)に入ることも示せば、その定理より証明できたことになりますね

>>36
基礎論でも自然数が1以上をさすこともあります
0を含むという定義は小数派ですね

39 :132人目の素数さん:2018/05/05(土) 18:55:33.83 ID:NwM7wsxx.net
xy平面上の曲線C:y=f(x)(a≦x≦b)の長さが、ある初等関数g(x)を用いてg(b)-g(a)と表されるとする。
このとき、Cをx軸の周りに一回転させてできる曲面Dの面積も、ある初等関数h(x)を用いてh(a)-h(b)と表せることを示せ。

40 :132人目の素数さん:2018/05/05(土) 19:17:32.69 ID:UMWHPgmy.net
>>23
あってる?

41 :132人目の素数さん:2018/05/05(土) 19:20:24.86 ID:UMWHPgmy.net
>>39
f(x)は初等関数?

42 :132人目の素数さん:2018/05/05(土) 19:30:15.42 ID:UMWHPgmy.net
>>23
失礼しました。合ってるね。mod2、mod3、mod5で考えて必要条件出して、その後mod10で矛盾って何って思ったけど、背理法だからこういう事もあるのね。

43 :132人目の素数さん:2018/05/05(土) 19:35:13.52 ID:oI3zjkm/.net
sinh/hは1で y= sinuを微分するとy'= cosxの違いを教えてください

44 :132人目の素数さん:2018/05/05(土) 19:36:43.83 ID:vn0wm8Ig.net
0での微分では一致しますね

45 :132人目の素数さん:2018/05/05(土) 20:15:57.27 ID:oI3zjkm/.net
>>44
どういうことですか?詳しく教えてほしいです

46 :132人目の素数さん:2018/05/05(土) 20:41:42.90 ID:UcMeYSP2.net
東京大学理学部数学科は、東京大学の頂点ですか?

47 :132人目の素数さん:2018/05/05(土) 20:48:21.41 ID:2uxMooR/.net
>>4
整数係数だと、るじゃんどるみたいにウマい多項式はないのでしょうか

48 :132人目の素数さん:2018/05/05(土) 20:54:35.93 ID:S3jerne2.net
>>39
曲面は
y=f(x)cosθ
z=f(x)sinθ


49 :132人目の素数さん:2018/05/05(土) 21:05:10.07 ID:r0nMKpCd.net
217の一番なんですが、この一般項anはどうやって出したのですか?
等差数列と等比数列の足し算なので
an=(4n-2)+[-4(2)↑n-1]
はダメなのですか?
https://i.imgur.com/Xvj5iB6.jpg
https://i.imgur.com/Xvj5iB6.jpg

50 :132人目の素数さん:2018/05/05(土) 21:44:58.85 ID:WiSvwsft.net
>>49
問題も見せろや

51 :132人目の素数さん:2018/05/05(土) 22:00:51.56 ID:NY9sEJP5.net
>>38
>0を含むという定義は小数派ですね
基礎論とか整数論はやったこたないが、おらっちはずっと0は自然数だお

52 :132人目の素数さん:2018/05/05(土) 22:19:15.71 ID:/fJjnEsy.net
微分方程式の入り口に来たんだけど、一般解を求めよって言われたときに、
特異解のことも答えたほうがいいもの?

53 :132人目の素数さん:2018/05/05(土) 22:21:39.76 ID:66Kv0Reu.net
>>51
義務教育では1以上と習ったはずですが?

54 :132人目の素数さん:2018/05/05(土) 22:23:30.40 ID:66Kv0Reu.net
>>52
入り口ならそもそもないと思いますよ

55 :132人目の素数さん:2018/05/05(土) 22:44:32.10 ID:k4JBC7qB.net
>>39は成り立たん気がする。
結局
l(x) = ∫[a,x]√(1+f’(t)^2)dt が初等関数のとき
S(x) = ∫[a,x]πf(t)^2dt も初等関数であるか?
だけどS(x)が初等関数ならf(x)^2=(S’(x)/π)も初等関数になるけど
v(x) = √(1+f’(x)^2)とおいたとき
l(x) = ∫[a,x]v(t)dtは初等関数だけど
f(x)^2 = ∫[a,x]√(v(t)^2-1)dtが楕円積分になる例なんていくらでもありそうな希ガス。

56 :132人目の素数さん:2018/05/05(土) 23:01:30.41 ID:/fJjnEsy.net
うーん、普通はないのかな。
微分方程式の変数分離型の最初の問題からいきなり特異解がひっついてるんだけど。
なんか理由があるんだろうな
さんきゅ

57 :132人目の素数さん:2018/05/05(土) 23:15:14.78 ID:6jnEKWaB.net
変数分離型で分母が0になったりすると出て来るから
問題しだいだな

58 :132人目の素数さん:2018/05/06(日) 00:52:47.38 ID:KhrVKVJy.net
>>39

x軸に垂直な断面の円周の長さは 2πf(x)
xy-平面内の幅は g '(x) dx = √{1 + f '(x)^2} dx
h(b) - h(a) = ∫[a,b] 2πf(x) g '(x) dx
 = 2π{f(b)g(b)-f(a)g(a)} - ∫[a,b] 2πf '(x) g(x) dx
 = 2π{f(b)g(b)-f(a)g(a)} - ∫[a,b] 2π√{g '(x)^2 - 1} g(x) dx

う〜む

59 :132人目の素数さん:2018/05/06(日) 01:04:34.75 ID:KhrVKVJy.net
>>49

> 等差数列と等比数列の足し算なので

ちがいます、掛け算です。

 a_n = {(-1)^(n-1)} * 2n

60 :132人目の素数さん:2018/05/06(日) 01:06:26.13 ID:dOgOsLZa.net
前スレ>>953で素数の間隔について質問したものです
前スレ>>954さん、>>956さん、情報ありがとうございました

ベルトラン・チェビシェフの定理で証明されているのですね

解答ありがとうございました

61 :132人目の素数さん:2018/05/06(日) 01:09:22.86 ID:WFF9FQ3N.net
数列{an}は初項が1の隣接k項間漸化式である。例えばk=3のとき、0でない実数s,tを用いてa(n+2)=sa(n+1)+ta(n)と表される。
この数列がlim[n→∞] a(n)=+∞となるとき、a(j)>a(j-1)なるjを少なくとも何個持つといえるか。

62 :132人目の素数さん:2018/05/06(日) 01:44:22.54 ID:NxPHNYwv.net
>>58

> = 2π{f(b)g(b)-f(a)g(a)} - ∫[a,b] 2π√{g '(x)^2 - 1} g(x) dx

の最後の積分のみならず、前の方のf(x)すら初等的という仮定が使えない。
使えるのは “g(x)が初等関数” のみ。
到底できる気がしないんだけど。

63 :132人目の素数さん:2018/05/06(日) 02:28:51.16 ID:hl5U1bPk.net
>>23
mod 5でうまくいくようにしたのにmod 5で矛盾するっておかしいと思ったら
r≡2(mod 5)じゃなくてr≡3(mod 5)じゃないか

64 :132人目の素数さん:2018/05/06(日) 04:47:18.96 ID:tBbFLi0q.net
極限です、教えてください
https://i.imgur.com/x1NFUTY.jpg

65 :132人目の素数さん:2018/05/06(日) 05:31:53.10 ID:WFF9FQ3N.net
>>64
nまたはnの式で割って無理やりk/nを作ればいい

66 :132人目の素数さん:2018/05/06(日) 14:58:12.48 ID:I1r8PHjE.net
数列は漸化式である

67 :132人目の素数さん:2018/05/06(日) 15:36:46.99 ID:7UgnNpKw.net
じゃ、次の数列の漸化式教えて、
8,9,7,9,3,2,3,8,4,6,…
ある規則で並んでるんだけどね

68 :132人目の素数さん:2018/05/06(日) 15:57:55.67 ID:i0xgdZwj.net
>>67

> じゃ、次の数列の漸化式教えて、
> 8,9,7,9,3,2,3,8,4,6,…
> ある規則で並んでるんだけどね

教えて か〜ら〜の〜 ある規則で並んでるんだけどねwww

69 :132人目の素数さん:2018/05/06(日) 16:06:01.75 ID:KhrVKVJy.net
>>67

 a_n = [10^n・π] - 10・[10^(n-1)・π]  (小数点下n桁目)

3,1,4,1,5,9,2,6,5,3,5,8,9,7,9,3,2,3,8,4,6,2,6,4,3,3,8,3,2,7,9,5

70 :132人目の素数さん:2018/05/06(日) 16:48:52.61 ID:KhrVKVJy.net
>>64

√(nn+k) -n - k/(2k+1)
= k/{√(nn+k) +n} - k/(2n+1)
= k{(n+1) - √(nn+k)} / ( {√(nn+k) +n} (2n+1) )
= k(2n+1-k) / ( {(n+1)+√(nn+k)} {√(nn+k) +n} (2n+1) )

ここで 2n(2n+1)^2 < (分母) < (2n+2)(2n+1)^2

分子だけたすと Σ[k=0,2n] k(2n+1-k) = 2n(2n+1)(2n+2) /6

lim[n→∞] (与式) = 1/6,

71 :132人目の素数さん:2018/05/06(日) 16:53:00.50 ID:JUqpRGxX.net
√2を循環少数に展開する無限級数
Σ少数第k位/2^k を求めよ

72 :132人目の素数さん:2018/05/06(日) 17:11:12.46 ID:fwnVSR3d.net
A={1-1/n:n∈N}のminAって存在しますよね?
教授が存在しないって言ってたんですけどだれかご教授下さい

73 :132人目の素数さん:2018/05/06(日) 17:15:16.81 ID:FzJFoTTt.net
maxの間違えでしょうね

74 :132人目の素数さん:2018/05/06(日) 17:27:46.77 ID:7UgnNpKw.net
>>69 うぬ

75 :132人目の素数さん:2018/05/06(日) 22:19:11.73 ID:KhrVKVJy.net
>>64

別々に計算すると
 Σ[k=0,2n] {k/(2n+1) + n} = n + (2n+1)n = 2n(n+1),

Σ[k=0,2n] √(nn+k)
 = {n/2 + Σ[k=1,2n] √(nn+k) + (n+1)/2} - 1/2  …… 割線
 < ∫[nn,(n+1)^2] √x dx - 1/2
 = 2n(n+1) + 1/6,
から
(与式) < 1/6,

Σ[k=0,2n] √(nn+k)   …… 接線
 > ∫[nn-1/2,(n+1)^2 -1/2] √x dx
 = ∫[nn,(n+1)^2] √x dx - ∫[(n+1)^2 -1/2,(n+1)^2] √x dx + ∫[nn-1/2,nn] √x dx
 > 2n(n+1) + 2/3 - {n+1 - √(nn-1/2)}/2
 = 2n(n+1) + 1/6 - {n - √(nn-1/2)}/2
 = 2n(n+1) + 1/6 - 1/[4{n + √(nn-1/2)}]
から
(与式) > 1/6 - 1/[4{n + √(nn-1/2)}] → 1/6 (n→∞)

76 :132人目の素数さん:2018/05/06(日) 22:32:20.00 ID:KhrVKVJy.net
>>75

∫[nn,(n+1)^2] √x dx = [ (2/3) x^(3/2) ]_{x:nn→(n+1)^2}
 = (2/3) {(n+1)^3 - n^3}
 = (2/3) (3nn +3n +1)
 = 2n(n+1) + 2/3,
を使った。

77 :132人目の素数さん:2018/05/06(日) 22:49:46.53 ID:KhrVKVJy.net
>>71

√2 を循環小数で表わすのは無理っすぅ

78 :132人目の素数さん:2018/05/06(日) 23:56:29.96 ID:GoWoY/5G.net
もしかして:2進数展開

79 :132人目の素数さん:2018/05/07(月) 00:04:02.48 ID:h2biOA7U.net
お願い
(問)
0<a<1として、a1=a, an+1 = 4an*(1-an)  として漸化式で数列anを定義する。
lim[n->∞]an = 0 のとき、aN=0 となる自然数Nが存在することを示せ。

80 :132人目の素数さん:2018/05/07(月) 00:23:31.94 ID:2KHwpYyw.net
>>79
定義より
a[n]<1/2 ⇒ a[n+1] = 4a[n](1-a[n]) ≧ 2a[n]
である。lim a[n]=0からn≧N ⇒ a[n]<1/2となるNがとれるが、このときn≧Nに対して
1/2 > a[n] ≧ 2^(n-N)a[N]
により
0≦a[N]<2^(N-n)(1/2)
を得る。n→∞とすればa[N]=0。

81 :132人目の素数さん:2018/05/07(月) 03:58:55.21 ID:O7EK/B2R.net
>>79
y=4x(1-x) と y=x の グラフを 0≦x≦1 の範囲で描いて、
初期値aによって、a[n]の値がどのような“動き”をするか調べてみるとよい。
多くの場合は、大きくなったり、小さくなったりと、複雑な動きをすることが判ると思う。

しかし、特徴的な動きもみられる。

あるところでa[k]が3/4近辺の値をとると、次の値も、3/4近辺になる。当然、その次も3/4近辺だ。
3/4を含むある範囲では、常にこのループに陥り、a[k]=0となる様なことはない。
つまり、あるkで、a[k]の値が、この範囲に入るような値を取ると、lim[n->∞]a[n] = 0 となる様なことはない。

また、あるところでa[k]が、1/2 という値を取ると、a[k+1]=1、a[k+2]=0、となり、n≧k+2では常にa[n]=0となる
a[k]が1/2という値を取るためには、a[k-1]が 1/2=4x(1-x) の解 つまり、(2±√2)/4 という値を取っていた場合である。
さらに、(2±√2)/4=4x(1-x) の解を取っていると、.... というように、ある特別な値を取っていた時に限り、
あるところでa[k]=0となり、それ以後、常に0となる。

それ以外の値の場合は、大きくなったり、小さくなったり、あるいは、3/4近辺でぐるぐる回っていたりするだけで、
lim[n->∞]a[n] = 0 等という事は起こらない。このようなことを説明すればよい。

82 :132人目の素数さん:2018/05/07(月) 04:24:39.31 ID:O7EK/B2R.net
補足。というか、こちらの方が、本命かもしれない。
多くの場合、
lim[n->∞]a[n] = 0
というのを見ると、a[n]≠0 だけど、 nが大きくなるにつれて、|a[n]|が
どんどん小さくなるような場合を思い浮かべると思うけど、
この問題の場合の lim[n->∞]a[n] = 0 は、そのようなケースではなく、
あるところで、a[k]が0になり、その後は、定義から常に0という場合に限られる。

何故なら、あるところで、a[k]=ε、(ただし、εは非常に小さい値で、正)、を取ったとすると、

a[k+1]=4ε(1-ε)≒4ε=4a[k] となり、前項より大きくなる。

このような性質を持っていては、
“a[n]≠0 だけど、 nが大きくなるにつれて、|a[n]|がどんどん小さくなるような場合”
の、lim[n->∞]a[n] = 0 は起こらない。実際、図を描いても確かめられる。
従って、lim[n->∞]a[n] = 0 というのは、あるところで、a[k]=0 となり、その後全ての項が0
の場合に限られると結論できる。

83 :132人目の素数さん:2018/05/07(月) 05:18:54.47 ID:N+kxDSb8.net
xy平面上に点A(1,0)も単位円Cがある。
点Aの、C上の点Pにおける接線lに関する対称点をBとするとき、以下の問に答えよ。

(1)P(cosθ,sinθ)とするとき、Bの座標をθを用いて表せ。

以下、θは0≦θ<2πを動くものとする。

(2)Bの座標を(s,t)とおき、点KをK(s+t,st)により定める。Kが動いてできる曲線Tの式を求めよ。

(3)Tの長さを求めよ。

84 :132人目の素数さん:2018/05/07(月) 07:50:42.17 ID:P9aQi060.net
>>48
ds=√(1+(f')^2)dx
dS=f(x)dsdθ
s=s(x)初等関数
ds=s'(x)dx
dS=f(x)s'(x)dxdθ
S(x)初等関数とは限らない
ボクの考えた最強のアホな問題

85 :132人目の素数さん:2018/05/07(月) 08:08:08.16 ID:Q4lXAlAO.net
限らないことの証明は

86 :132人目の素数さん:2018/05/07(月) 11:24:22.82 ID:LPTyY7qu.net
>>79
 α = arcsin(√a) = arccos(1-2a)/2,
とおくと
 a_n = {sin(2^(n-1)・α)}^2 = {1 - cos(2^n・α)}/2

 α = (奇数)π/{2^(n0-1)} ⇔  a_n = 0 (n≧n0)

87 :132人目の素数さん:2018/05/07(月) 11:40:50.80 ID:LPTyY7qu.net
>>86

つまり、α/π の2進数展開が有限項で終わるとき。

88 :132人目の素数さん:2018/05/07(月) 12:47:28.38 ID:7NDVoze6.net
2017年早稲田理工の5番ですが
f(x)=x^3+x^2+px+q g(x)=-1/x+1
条件:f(x)=0の任意の解αに対してg(α)もf(x)=0の解である。

当然一つの解はαでもう一つは-1/α+1
模範解答だともう一つはg(g(α))=-α+1/αになります
この後に解が同じか違うかで場合分けという流れですが

はじめの2つの解と解と係数の関係でもう一つの解を出すと-α^2-2α/α+1がでてしまいますが、これはなんでしょうか?任意解を入れれば初めの3つの解のどれかと同じになるということでしょうか??

89 :132人目の素数さん:2018/05/07(月) 12:59:11.20 ID:LPTyY7qu.net
>>83

 P-接線: (cosθ)x + (sinθ)y = 1,
 Q (cos(2θ),sin(2θ)) とおくと、P は BQ の中点。
 B (2cosθ-cos(2θ),2sinθ-sin(2θ)) = (s,t)

 s+t = (√2){2sin(θ +π/4) - sin(2θ +π/4)},
 st = (3/2)sin(2θ) - (2-cosθ)sin(3θ) = 2sin(2θ) - 2sin(3θ) + (1/2)sin(4θ),

90 :132人目の素数さん:2018/05/07(月) 14:13:35.36 ID:gONr+mmN.net
接線の性質として
半径の中心と異なる端で,半径に垂直な直線は,この円の接線である。

と書いてあるがこれはどういうこと?
半径の中心と異なる端ってことは、円周か円の中心ってこと?

91 :132人目の素数さん:2018/05/07(月) 14:22:04.37 ID:gONr+mmN.net
>>90
自己解決した

つまり
半径の中心と異なる端ってことは円周か
円周上で半径に垂直な直線は接線だぜってことか

92 :132人目の素数さん:2018/05/07(月) 19:39:07.93 ID:pC2LS//C.net
未だ>>5解けないんだけど。これホントに解けるんかな?

93 :132人目の素数さん:2018/05/07(月) 19:54:58.69 ID:9iXmWBXM.net
>>92
>>23が正解らしいけども

94 :132人目の素数さん:2018/05/07(月) 21:05:04.27 ID:dkZJD9G+.net
>>93
いや、それは後で間違い指摘されて実際間違ってる。今のところ正解出てないと思う。でもこのスレ解けない問題上がってくる事もあるからその類かもしれないけど。

95 :132人目の素数さん:2018/05/08(火) 00:32:16.79 ID:VHCLxHr+.net
今ふっと思い立ったんだけどもしかして>>23が出題者でその解答が間違ってたのかな?

96 :132人目の素数さん:2018/05/08(火) 10:58:07.18 ID:mTlWCjoA.net
f=sin(ax)/axとした時に
甜0,1]fdx / 甜0,1]f^2dx
って求められますか?

教員曰く簡単らしいんですけれど全く無理でした…

97 :132人目の素数さん:2018/05/08(火) 11:17:44.07 ID:iiV8V26X.net
たぶん誤解してる

98 :132人目の素数さん:2018/05/08(火) 12:20:56.12 ID:rSTdfkqz.net
>>96

∫f(x)dx = (1/a)∫sin(ax)/x dx = (1/a)Si(ax),

∫{f(x)}^2 dx = ∫{sin(ax)/ax}^2 dx
 = - {sin(ax)}^2 /(aax) + (1/a)∫sin(2ax)/x dx
 = - {sin(ax)}^2 /(aax) + (1/a)Si(2ax),

にて簡単

99 :132人目の素数さん:2018/05/08(火) 13:35:30.51 ID:iiV8V26X.net
>>98
で?

100 :132人目の素数さん:2018/05/08(火) 13:43:59.02 ID:VvipJoyD.net
ででんでん

101 :132人目の素数さん:2018/05/08(火) 14:34:06.13 ID:hahWjMqy.net
蒸し蒸し

102 :132人目の素数さん:2018/05/08(火) 14:35:14.14 ID:ELxTBK1l.net
なるほど。簡単♡

103 :132人目の素数さん:2018/05/08(火) 14:38:13.13 ID:G9yXtLP2.net
云々

104 :132人目の素数さん:2018/05/08(火) 15:04:15.29 ID:wp3HQiZB.net
>>98
Si(x)とは?

105 :132人目の素数さん:2018/05/08(火) 15:13:24.97 ID:GtJxWtYj.net
正弦積分関数

106 :132人目の素数さん:2018/05/08(火) 16:13:37.69 ID:hahWjMqy.net
積分がわからんから姑息な手段を使ったのか

107 :132人目の素数さん:2018/05/08(火) 16:26:07.78 ID:4EprKRzX.net
>>98
間違ってるぞ

108 :132人目の素数さん:2018/05/08(火) 16:33:42.19 ID:3em1sf2J.net
かたつむりか。ターミネーターじゃないのか。

109 :132人目の素数さん:2018/05/08(火) 20:12:32.29 ID:p9FzWnM6.net
3次元の座標系を任意に回転させたいのですが、何回回転させればよいか、場合分けすることはできますか?
1回のとき、2回のとき、3回のときがあると思います。

110 :132人目の素数さん:2018/05/08(火) 20:39:28.76 ID:SYsXvvMG.net
情報が足りない

111 :132人目の素数さん:2018/05/08(火) 21:47:22.34 ID:iiV8V26X.net
>>109
質問になってない

112 :132人目の素数さん:2018/05/08(火) 21:55:54.22 ID:MbAKKwrj.net
SO(3)は3次元

113 :132人目の素数さん:2018/05/08(火) 22:09:00.06 ID:+3UojrGT.net
すまん、この問題がわからないんだが・・・・
https://imgur.com/4utNyqX
お願いします(´・ω・`)

114 :132人目の素数さん:2018/05/08(火) 22:17:14.40 ID:iiV8V26X.net
xz座標だけθ回転させるだけだよ
結局回転で変わるのは外積だけだが

115 :132人目の素数さん:2018/05/08(火) 22:17:58.83 ID:p9FzWnM6.net
109です。
わかりづらくてすみません。
座標系じゃなく3次元の立体図形を回転させるもきに、

116 :132人目の素数さん:2018/05/08(火) 22:23:00.28 ID:p9FzWnM6.net
書き間違えました。
少し質問を変えます。
座標系じゃなく3次元の直方体のような立体図形にします。
立体図形を任意に回転させるときに、例えば上下反対にするには1回転させますが、
左右を上下にして裏表反対にするには2回転必要かと思います。
ここで思ったのですが、2次元の場合は1回転で全ての回転を表されるので、3次元の場合は2回転あればすべての回転を表されると思ったのですが、いかがでしょうか。

117 :132人目の素数さん:2018/05/08(火) 22:30:46.76 ID:MbAKKwrj.net
>>116
SO(3)は3次元なので3種類の回転を用意せんとダメだっての

118 :132人目の素数さん:2018/05/08(火) 22:32:53.46 ID:MbAKKwrj.net
>>116
補足
ただしかける回数に制限がないなら2種類で可能。

119 :132人目の素数さん:2018/05/08(火) 22:37:40.37 ID:hahWjMqy.net
おいらに任せろ

120 :132人目の素数さん:2018/05/08(火) 22:49:41.27 ID:p9FzWnM6.net
116です。
回答ありがとうございます。
3種類の回転が必要ですか。SO(3)という数式を初めて見ました。
回転の軸を3次元中に自由に取れるとしても必ず3種類の回転が必要になりますでしょうか。

121 :132人目の素数さん:2018/05/08(火) 23:01:33.14 ID:MbAKKwrj.net
>>120
なるよ。もしA(x)B(y) (x,yはパラメータ)でSO(3)全体をカバーできたらそれは2次元空間から3次元空間への全射を与えてしまうけどそれは無理。

122 :132人目の素数さん:2018/05/08(火) 23:07:36.31 ID:rSTdfkqz.net
>>109 >>115 >>116 >>120

z軸のまわりにα
y'軸のまわりにβ(=∠zOz')
z'軸のまわりにγ

(α,β,γ)をオイラー角と云うらしい

123 :132人目の素数さん:2018/05/08(火) 23:08:55.52 ID:p9FzWnM6.net
>>121
なるほど、なんとなくわかりました。
>>116で書いたとおり、1回転とか2回転で達成できるような場合もあると思うのですが、そういった特殊な例はどういった場合とかって簡潔に言えますか?

124 :132人目の素数さん:2018/05/08(火) 23:10:34.09 ID:p9FzWnM6.net
>>122
ありがとうございます。オイラー角についてググってみます。

125 :132人目の素数さん:2018/05/08(火) 23:26:31.89 ID:q2L0o+ot.net
>>123
さあ?そこまでいくとわかんない。でも2個の回転行列A(x)、B(y)を指定すれば
A(x)B(y)の形の回転の全体は実2次元部分空間になるからシンプルな表現はあるとは思うけど。

126 :132人目の素数さん:2018/05/08(火) 23:31:33.29 ID:p9FzWnM6.net
>>125
ありがとうございます。教えていただいたのをもとに自分なりに勉強してみます。

127 :132人目の素数さん:2018/05/08(火) 23:53:08.87 ID:DjWjgehL.net
>>96
どうやらこの積分、分子分母にx^2を掛けても結果が変わらないそうです
どうやってそのことを導出したのでしょうか…?

128 :132人目の素数さん:2018/05/09(水) 00:09:52.11 ID:6yVxW1Wf.net
>>127
分子分母に同じ数かけたらそら同じ数になるのでは?

129 :132人目の素数さん:2018/05/09(水) 00:35:56.85 ID:V+z5IWFo.net
>>127
分子分母の被積分関数にx^2かけるの?全然違う値になるけど?問題間違ってない?

f(x):=sin(a*x)/(a*x);
romberg(f(x),x,0.0001,1)/romberg(f(x)^2,x,0.001,1),a:1,numer;
romberg(f(x)*x^2,x,0.0001,%pi)/romberg(f(x)^2*x^2,x,0.001,%pi),a:1,numer;
romberg(f(x),x,0.0001,1)/romberg(f(x)^2,x,0.001,1),a:2,numer;
romberg(f(x)*x^2,x,0.0001,1)/romberg(f(x)^2*x^2,x,0.001,1),a:2,numer;
romberg(f(x),x,0.0001,1)/romberg(f(x)^2,x,0.001,1),a:3,numer;
romberg(f(x)*x^2,x,0.0001,1)/romberg(f(x)^2*x^2,x,0.001,1),a:3,numer;
(%o69) 1.055384728709074
(%o70) 2.000000016713958
(%o71) 1.195429148161006
(%o72) 1.464505695121646
(%o73) 1.306210378186181
(%o74) 1.981775232275942

130 :132人目の素数さん:2018/05/09(水) 00:49:21.65 ID:hO1Ej9LV.net
ミスってたのでやりなおし。やっぱり同じ値にはならない??

>>127
f(x):=sin(a*x)/(a*x);
romberg(f(x) ,x,0.00001,1)/romberg(f(x)^2 ,x,0.00001,1),a:1,numer;
romberg(f(x)*x^2,x,0.00001,1)/romberg(f(x)^2*x^2,x,0.00001,1),a:1,numer;
romberg(f(x) ,x,0.00001,1)/romberg(f(x)^2 ,x,0.00001,1),a:2,numer;
romberg(f(x)*x^2,x,0.00001,1)/romberg(f(x)^2*x^2,x,0.00001,1),a:2,numer;
romberg(f(x) ,x,0.00001,1)/romberg(f(x)^2 ,x,0.00001,1),a:3,numer;
romberg(f(x)*x^2,x,0.00001,1)/romberg(f(x)^2*x^2,x,0.00001,1),a:3,numer;

(%o102) 1.054320648435071
(%o103) 1.104493895983902
(%o104) 1.193802891132201
(%o105) 1.4645056917975
(%o106) 1.303664967479121
(%o107) 1.981775220915906

131 :132人目の素数さん:2018/05/09(水) 02:22:57.05 ID:cq9H9+pa.net
>>127
答えとしては
4(sin(a)-acos(a))/(2a-sin(2a))
になるらしいです…

132 :132人目の素数さん:2018/05/09(水) 03:04:15.07 ID:bPbSE1pH.net
>>131
そんなはずはない。もとめる値をI(a)とおくと
lim[a→∞]I(a) = 1になるけど、その値振動するやん。

133 :132人目の素数さん:2018/05/09(水) 04:02:36.55 ID:6xr7P3xx.net
f+gが微分可能でfが微分可能でない例
fgが微分可能でfが微分可能でない例
f・gが微分可能でfまたはgが微分可能でない例
を教えて下さい

134 :132人目の素数さん:2018/05/09(水) 06:42:48.91 ID:OgcCeOxy.net
nCrで選ぶ個数に文字rを使う理由って何?

135 :132人目の素数さん:2018/05/09(水) 07:40:18.48 ID:ymGjZh6L.net
>>133
f:ディリクレ関数、g=-fでf+g=0は微分可能
f:ディリクレ関数、g=0でfg=0は微分可能
f=0、g:ディリクレ関数でf・g=0は微分可能

136 :132人目の素数さん:2018/05/09(水) 10:05:29.42 ID:8E+qLwNX.net
どのような楕円であっても、その周長(一周分)を求めることはできますか?

137 :132人目の素数さん:2018/05/09(水) 10:14:51.45 ID:OgcCeOxy.net
はい

138 :132人目の素数さん:2018/05/09(水) 11:07:56.72 ID:k89ZCTKX.net
鉛筆で楕円を描いて、直線を描いた場合との比較で、
鉛筆の重さの減少量と長さとの関係から計算する

139 :132人目の素数さん:2018/05/09(水) 11:41:54.29 ID:72cH2LZd.net
fが区間Iで微分可能関数であるとき
任意のx,y∈Iに対して|f(x)-f(y)|≦K|x-y| (Kは定数)が成立するならば
任意のx∈Iに対し、|f'(x)|≦Kが成立することを証明して下さい
極限取ることは分かってるのですが、x=yの場合や
|lim[x→y](f(x)-f(y))/(x-y)|=lim[x→y]|(f(x)-f(y))/(x-y)|が示せなく困っています

140 :132人目の素数さん:2018/05/09(水) 11:58:46.91 ID:XOhlVUhY.net
>>127 >>131
なんじゃそりゃ?

∫ p(x)dx ∫ x^2p(x)dx
―――― = ――――――
∫q(x)dx ∫x^2q(x)dx

ってやっちゃったのか? ’∫’ 無視して?恐ろしいな。

141 :132人目の素数さん:2018/05/09(水) 12:42:29.07 ID:GEUtkttz.net
>>136 >>137

(x/a)^2 + (y/b)^2 = 1, (0<b≦a)

とする。

L = 4∫[0,π/2] √{(a・cosθ)^2 + (b・sinθ)^2} dθ

= 4a ∫[0,π/2] √{ 1 - (k・sinθ)^2} dθ     k = √{1 - (b/a)^2} 離心率

= 4a E(k)

第二種の完全楕円積分、θはパラメータ

142 :132人目の素数さん:2018/05/09(水) 13:25:25.89 ID:D8i3yt6A.net
>>139
εδを使え

143 :132人目の素数さん:2018/05/09(水) 13:53:41.98 ID:k89ZCTKX.net
>>141
楕円なんたらとかかってな名前をつけて誤魔化さないで、
ちゃんと積分してください。

144 :132人目の素数さん:2018/05/09(水) 13:55:48.16 ID:0M/ItzX4.net
無知は力なり
とはよく言ったものだw

145 :132人目の素数さん:2018/05/09(水) 14:34:58.63 ID:8E+qLwNX.net
>>141
私は高校生です
√1-t^2 はt=sinθと置けと言われました
ksinθ=aと置いたらどうですか

146 :132人目の素数さん:2018/05/09(水) 14:38:18.51 ID:vfY0Zsg+.net
>>145
楕円の周を求めるのは難しくて、簡単な式で表すことができないということが知られています
つまり、解けません

147 :132人目の素数さん:2018/05/09(水) 14:39:21.32 ID:xpXWPMq2.net
ググれよ

https://ja.wikipedia.org/wiki/楕円積分

148 :132人目の素数さん:2018/05/09(水) 14:43:36.34 ID:Pq+VYP9S.net
>>145
だからどうした?それで解けたのか?

149 :132人目の素数さん:2018/05/09(水) 14:48:17.50 ID:72cH2LZd.net
>>142
それが出来ないんです…

150 :132人目の素数さん:2018/05/09(水) 15:03:33.73 ID:k89ZCTKX.net
>>146
難しのと解けないのとは違うよね。
難しいからごまかしてるんでしょ?

151 :132人目の素数さん:2018/05/09(水) 15:17:31.37 ID:8E+qLwNX.net
>>148
解けました

152 :132人目の素数さん:2018/05/09(水) 15:32:05.35 ID:vfY0Zsg+.net
>>150
>>151

>>147みてみてくださいね
初等関数で表せないとあるはずです
初等関数というのは、簡単な式ということです

153 :132人目の素数さん:2018/05/09(水) 16:17:42.04 ID:wgZFPRFj.net
>>151
よかったね、さようなら

154 :132人目の素数さん:2018/05/09(水) 16:26:06.35 ID:8E+qLwNX.net
楕円積分の解法を思いつきました
級数展開してから積分すれば良いですね
多項式なら簡単に積分できますので

155 :132人目の素数さん:2018/05/09(水) 16:35:53.36 ID:lEqSCdxW.net
劣等感婆も高校生を自称していたことがありましたね

156 :132人目の素数さん:2018/05/09(水) 16:41:53.59 ID:KCh8zfY4.net
何で性別女って設定なの?

157 :132人目の素数さん:2018/05/09(水) 16:46:14.40 ID:PWrBl1fd.net
f(xy)kg(xy)=0ってなんなんなんですか?
fとgの交点の座標を求めてkをだすのも=0ってのもなんなんなんですか?

158 :132人目の素数さん:2018/05/09(水) 16:47:58.92 ID:wgZFPRFj.net
昔、関数の連続性を高校の範囲で証明できると主張した馬鹿がいたが

159 :132人目の素数さん:2018/05/09(水) 17:28:42.17 ID:JB8c0kqr.net
>>157
f(x,y)=0の曲線とg(x,y)=0の曲線があった時、f(x,y)+kg(x,y)=0の曲線は、f(x,y)=0とg(x,y)=0の全ての交点を通るということです

160 :132人目の素数さん:2018/05/09(水) 17:52:26.77 ID:FkjzUCdK.net
>>96=>>136

161 :132人目の素数さん:2018/05/09(水) 18:42:01.73 ID:PWrBl1fd.net
>>159
A+KB=0でなぜB以外の全ての直線を表せるのだろ

162 :132人目の素数さん:2018/05/09(水) 18:47:40.43 ID:JB8c0kqr.net
kが変われば傾きも変わりますね

163 :132人目の素数さん:2018/05/09(水) 18:50:01.31 ID:AeGKfNVD.net
日本人は全員ゴミ

164 :132人目の素数さん:2018/05/09(水) 18:57:56.95 ID:PWrBl1fd.net
k(ax+bx+c)すると傾き以外も変わるのじゃ?

165 :132人目の素数さん:2018/05/09(水) 19:03:41.56 ID:PWrBl1fd.net
K倍しても移項してyの係数割る時元に戻る気がする

166 :132人目の素数さん:2018/05/09(水) 20:54:50.50 ID:gnIjlof6.net
>>161
KA+LB=0のがいい

167 :132人目の素数さん:2018/05/09(水) 23:11:52.44 ID:GEUtkttz.net
>>136 >>145 >>151 >>154

まとめると…

・楕円、双曲線の長さは初等函数では表せない。

・円、放物線の長さは初等函数(√とlog)で表わせる。

168 :132人目の素数さん:2018/05/09(水) 23:46:01.38 ID:gnIjlof6.net
>>167
>・円、放物線の長さは初等函数(√とlog)で表わせる。
放物線が?

169 :132人目の素数さん:2018/05/09(水) 23:58:44.81 ID:GEUtkttz.net
>>168

y = ax^2 とすると、

L(0,x) = ∫[0,x] √{1+(2ax ')^2} dx '

 = (1/2)x√{1+(2ax)^2} + (1/4a)log{2ax + √{1+(2ax)^2}]

170 :132人目の素数さん:2018/05/10(木) 07:10:25.01 ID:lMbFJOYg.net
【問題】
10から110までの数字の組み合わせの和で
10から110までのすべての整数を表現したいとき
用意しなければならない数字の最小の個数とその数字を述べよ。




お願いします。

171 :170:2018/05/10(木) 07:11:57.01 ID:lMbFJOYg.net
>>170
なお、組み合わせは2個でも3個でも、何個でも組み合わせて良い。

172 :132人目の素数さん:2018/05/10(木) 07:33:28.06 ID:58oVMDr7.net
>>170
数字なの?
数ではなくて!?

173 :132人目の素数さん:2018/05/10(木) 10:31:25.24 ID:SsvtRexk.net
>>170
10+10=20
はあり?

174 :132人目の素数さん:2018/05/10(木) 10:59:04.99 ID:lprgN9Zl.net
>>170

10+10 = 20がありなら
{10,11,12,13,14,15,16,17,18,19} の10個
無しなら
{10,11,12,13,14,15,16,17,18,19,20} の11個

175 :IQの低い人:2018/05/10(木) 13:29:02.37 ID:sToklkep.net
1,0でいいんじゃないの

176 :132人目の素数さん:2018/05/10(木) 13:40:08.31 ID:/57cBKqk.net
いや、1だけでいい

177 :132人目の素数さん:2018/05/10(木) 13:41:51.82 ID:/57cBKqk.net
175,176は、10が組み合わせ可能な数字の最小値であるという仮定を無視してしまったようだ

178 :132人目の素数さん:2018/05/10(木) 14:47:07.34 ID:NEWFwW7D.net
>>177
> 175,176は、10が組み合わせ可能な数字の最小値であるという仮定を無視してしまったようだ
数字って0123456789のことよ

179 :132人目の素数さん:2018/05/10(木) 16:25:13.36 ID:9tRstjmn.net
素数pを1つとり、p^nを3で割った余りをanとする。{an}の一般項を求めよ。

180 :132人目の素数さん:2018/05/10(木) 16:47:11.20 ID:swO+9t/h.net
>>166
へー

181 :132人目の素数さん:2018/05/10(木) 16:48:52.87 ID:swO+9t/h.net
>>161
高校数学では説明できないんでしょうか

182 :132人目の素数さん:2018/05/10(木) 16:50:12.60 ID:9tRstjmn.net
>>181
数学Vだと思うがロルの定理を使うと説明しやすい

183 :132人目の素数さん:2018/05/10(木) 16:57:21.09 ID:swO+9t/h.net
>>182
微分積分勉強し直して出直してきます。
とりま頭に入れて問題とばそうかな

184 :132人目の素数さん:2018/05/10(木) 17:09:16.92 ID:IRATbn/s.net
>>181
f(x,y)=0 と g(x,y)=0の交点は、f(x,y)=0 かつ g(x,y)=0を満たす点(x0,y0), (x1,y1), ... であり
f(x0,y0)=0、g(x0,y0)=0 などを満たす。

よって、f(x0,y0) + kg(x0,y0) = 0 などとなるので、f(x,y)+kg(x,y)=0 は、必ず f(x,y)=0 , g(x,y)=0の
全ての交点を通る。

185 :132人目の素数さん:2018/05/10(木) 19:21:06.15 ID:9tRstjmn.net
a,bは0でない実数とする。
双曲線(x^2/a^2)-(y^2/b^2)=1の、t≦x≦t+kの区間の長さをL(t,k)とする。

(1)L(t,k)が定義できるようなtの範囲を定めよ。

(2)tは(1)の範囲にあり、また正とする。lim[k→∞] {L(t,k)/(αk)} = 1となるαが存在することを示せ。

186 :132人目の素数さん:2018/05/10(木) 20:08:10.67 ID:9tRstjmn.net
2^n+1(nは2以上の自然数)の形で表される自然数は、いくつかの相異なる素数の和で表せることを示せ。

187 :132人目の素数さん:2018/05/10(木) 20:55:54.68 ID:R9xe/nJK.net
>>180
KA+(1-K)B=0

188 :132人目の素数さん:2018/05/11(金) 00:38:27.38 ID:SdgBfY6R.net
>>179

 p≡1 (mod 3) のとき、a_n = 1
 p≡2 (mod 3) のとき、a_n = 2 (n:奇数)、a_n = 1(n:偶数)
 p=3 のとき、a_n = 0

>>185

(1) 任意の実数

(2)
 y = ±(b/a)√(aa-xx) (両分枝の合併)
 漸近線 y = ±(b/a)x
 α = (2/a)√(aa+bb)

189 :132人目の素数さん:2018/05/11(金) 01:45:36.95 ID:hWM/gHo5.net
>>186
7以上の整数nは相異なる2個以上の素数の和で表せる。
∵n≦23では以下のように正しい。
7 = 2+5, 8=3+5, 9=2+7, 10=3+7,11=2+9,12=5+7,13=2+11,
14=3+11,15=2+13,16=5+11, 17=2+3+5+7, 18=7+11, 19=2+17,
20=3+17, 21=2+19, 22=3+19, 23=3+7+13
24以上の整数Nについてn<Nで成立するとしてn=Nとする。
x以下の素数の数をπ(x)とおくと
x≧17に対しx/logx<π(x)
x≧1に対し<1.25506x/log(x)
https://ja.wikipedia.org/wiki/%E7%B4%A0%E6%95%B0%E8%A8%88%E6%95%B0%E9%96%A2%E6%95%B0
また容易にx≧11に対し
(x-7)/log(x-7)-1.25506(x/2)/log(x/2)>0
よってx≧24に対しπ(x-7)-π(x/2)>0。よってn/2<p<n-7を満たす素数が存在する。
帰納法の仮定より素数の集合PでΣ[q∈P]q = n-p<n/2となる。
またPの要素はすべてp未満である。よってn = p + Σ[q∈P]qとなりn=Nのときも正しいとわかった。
特にn≧3にたいし2^n+1は相異なる2個以上の素数の和で表せる。
また2^2+1=2+3。

190 :132人目の素数さん:2018/05/11(金) 01:48:38.09 ID:BJQnbXOk.net
>>189
訂正。14行目。
×:n/2<p<n-7
○:n/2<p≦n-7

191 :132人目の素数さん:2018/05/11(金) 02:01:18.43 ID:dn9lA8yy.net
計算ミスったorz。やり直し。

>>186
7以上の整数nは相異なる2個以上の素数の和で表せる。
∵n≦24では以下のように正しい。
7 = 2+5, 8=3+5, 9=2+7, 10=3+7,11=2+9,12=5+7,13=2+11,
14=3+11,15=2+13,16=5+11, 17=2+3+5+7, 18=7+11, 19=2+17,
20=3+17, 21=2+19, 22=3+19, 23=3+7+13, 24=5+19
25以上の整数Nについてn<Nで成立するとしてn=Nとする。
x以下の素数の数をπ(x)とおくと
x≧17に対しx/logx<π(x)
x≧1に対し<1.25506x/log(x)
https://ja.wikipedia.org/wiki/%E7%B4%A0%E6%95%B0%E8%A8%88%E6%95%B0%E9%96%A2%E6%95%B0
また容易にx≧25に対し
(x-7)/log(x-7)-1.25506(x/2)/log(x/2)>0
よってx≧25に対しπ(x-7)-π(x/2)>0。よってn/2<p<n-7を満たす素数が存在する。
帰納法の仮定より素数の集合PでΣ[q∈P]q = n-p<n/2となる。
またPの要素はすべてp未満である。よってn = p + Σ[q∈P]qとなりn=Nのときも正しいとわかった。
特にn≧3にたいし2^n+1は相異なる2個以上の素数の和で表せる。
また2^2+1=2+3。

192 :132人目の素数さん:2018/05/11(金) 03:14:45.17 ID:GCL6tPR6.net
スレ汚しすまんorz。再挑戦

12以上の整数nは相異なる2個以上の素数の和で表せる。
∵n≦39では以下のように正しい。

12=5+7,13=2+11,14=3+11,15=2+13,16=5+11, 17=2+3+5+7, 18=7+11, 19=2+17,
20=3+17, 21=2+19, 22=3+19, 23=3+7+13, 24=5+19, 25=2+23, 26=3+23, 27=3+5+19,
28=5+23, 29=3+7+19, 30=7+23, 31=2+29, 32=3+29, 33=2+31, 34=3+31, 35=5+7+23,
36=5+31, 37=3+5+29, 38=7+31, 39=2+37。

40以上の整数Nについてn<Nで成立するとしてn=Nとする。
x以下の素数の数をπ(x)とおくと
x≧17に対しx/logx<π(x)
x≧1に対し<1.25506x/log(x)
https://ja.wikipedia.org/wiki/%E7%B4%A0%E6%95%B0%E8%A8%88%E6%95%B0%E9%96%A2%E6%95%B0
また容易にx≧40に対し
(x-12)/log(x-12)-1.25506(x/2)/log(x/2)>0
よってx≧40に対しπ(x-12)-π(x/2)>0。よってn/2<p≦n-12を満たす素数が存在する。
帰納法の仮定より素数の集合PでΣ[q∈P]q = n-p<n/2となる。
またPの要素はすべてp未満である。よってn = p + Σ[q∈P]qとなりn=Nのときも正しいとわかった。

193 :132人目の素数さん:2018/05/11(金) 03:38:05.23 ID:SdgBfY6R.net
>>189-192

http://ja.wikipedia.org/wiki/ゴールドバッハの予想
http://ja.wikipedia.org/wiki/弱いゴールドバッハ予想

194 :132人目の素数さん:2018/05/11(金) 03:50:10.98 ID:AP8YngWJ.net
>>193
そのなかのどれかつかってもっとうまく示せる?
“異なる”と”2個以上”という縛りがあるからサラッと処理する方法思いつかんかったんだけど?

195 :132人目の素数さん:2018/05/11(金) 03:59:37.57 ID:0HlT8DZW.net
>>161
>A+KB=0でなぜB以外の全ての直線を表せるのだろ
A、Bが直線の式(2x+3y-5とか)を示しているんだったら、全ての直線を表すことはできないぞ

A+KB=0が表すのは、A=0とB=0の交点を通る直線のうちで、B以外のすべての直線だ。

196 :132人目の素数さん:2018/05/11(金) 07:45:25.94 ID:j9aOBqih.net
>>191
>またPの要素はすべてp未満である。

197 :132人目の素数さん:2018/05/11(金) 07:46:45.87 ID:j9aOBqih.net
>>195
>A+KB=0が表すのは、A=0とB=0の交点を通る直線のうちで、B以外のすべての直線だ。
KA+(1-K)B=0が表すのは、A=0とB=0の交点を通るすべての直線だ。

198 :132人目の素数さん:2018/05/11(金) 08:30:10.47 ID:lqFhgDwc.net
極方程式r=e^(-θ)で表される曲線の、y≧0の部分をCとする。またC上のy座標が0である点のうち、(1,0)でないものをBとする。
xy平面上を速さ1で動く点Pは、点A(2,0)から出発してCの外部かつy>0の領域を通り、C上の点Sに到達して、そこからC上を通って点Bに至る。
点Pが最も早く点Bに至るように、点Sの位置を定めよ。

199 :132人目の素数さん:2018/05/11(金) 09:22:02.18 ID:mYebBGTa.net
>>196
n/2<p≦n-12

200 :132人目の素数さん:2018/05/11(金) 11:21:19.76 ID:OFsS5uwl.net
>>197
A-B=0 が表せない。

201 :132人目の素数さん:2018/05/11(金) 13:48:25.25 ID:FtejLL1m.net
>>179
mod 3で
p≡1のときa_n=1
p≡2のときa_n=(3-(-1)^n))/2
p=3のときa_n=0

202 :132人目の素数さん:2018/05/11(金) 14:22:49.94 ID:FtejLL1m.net
>>157
f(a,b)=0, g(a,b)=0を満たす点(a,b)が存在するとする。
f(a,b)=0を満たす(a,b)はもちろんグラフf(x,y)=0上の点。
g(a,b)=0を満たす(a,b)はもちろんグラフg(x,y)=0上の点。
つまり(a,b)はf(x,y)=0, g(x,y)=0の交点である。

さて、実数α,β(いずれかは0でないとする)について、h(x,y)=αf(x,y)+βg(x,y)=0で表されるグラフを考える。
αf(a,b)+βg(a,b)=α*0+β*0=0より、(a,b)は(α,βによらず)グラフαf(x,y)+βg(x,y)=0上の点である。
もちろん、h(x,y)=0が(a,b)を通るあらゆる線を表すわけではない。

α≠0のとき、h(x,y)=0⇔f(x,y)+(β/α)g(x,y)=0⇔f(x,y)+kg(x,y)=0
α=0のときβ≠0で、h(x,y)=0⇔βg(x,y)=0⇔g(x,y)=0
よって、h(x,y)=0は次の2式として表せる。
f(x,y)+kg(x,y)=0
g(x,y)=0

(a,b)が複数存在する(つまりf(x,y)=0, g(x,y)=0の交点が2つ以上ある)場合も、全ての交点について上記の議論が成り立つ。
つまり、全ての交点はf(x,y)+kg(x,y)=0やg(x,y)=0上の点である。

203 :132人目の素数さん:2018/05/11(金) 15:05:29.99 ID:sUzPr1Ik.net
>>198
誰かこれを教えてください
最短経路の問題ですが折れ線でないのでわかりません

204 :132人目の素数さん:2018/05/11(金) 15:52:09.93 ID:sylrZDp2.net
>>203
問題を画像で上げろ

205 :132人目の素数さん:2018/05/11(金) 16:11:28.11 ID:MgeDrhnB.net
接戦引けばいいだけじゃないの?

206 :132人目の素数さん:2018/05/11(金) 16:16:37.23 ID:sUzPr1Ik.net
>>205
接線引いてC上通るのが最短経路?
多分そうかも知れないけど微分使わず説明できる?

207 :132人目の素数さん:2018/05/11(金) 16:17:21.46 ID:MgeDrhnB.net
変分原理でも使うか?

208 :132人目の素数さん:2018/05/11(金) 16:23:25.85 ID:MgeDrhnB.net
x軸とのなす角θとして道のりをθで表して微分すればいいだけだよ。

209 :132人目の素数さん:2018/05/11(金) 16:36:59.32 ID:sUzPr1Ik.net
互いに素な自然数a,bと3以上の素数pに対して、(a+bi)^pは実数でないことを示せ。
ただしiは虚数単位である。

210 :132人目の素数さん:2018/05/11(金) 16:45:53.09 ID:0yS46dEA.net
>>206
微分つかわんのはしんどいやろ。
長さの合計をLとして
dl = 接線方向への単位ベクトル + A方向への単位ベクトル
これが法線ベクトルと平行になる点が答え。つまりA方向へのベクトルが接線と平行のとき。

211 :132人目の素数さん:2018/05/11(金) 16:48:54.82 ID:l+WLgHA8.net
ID:sUzPr1Ikは高校生かw

212 :132人目の素数さん:2018/05/11(金) 16:52:18.28 ID:MgeDrhnB.net
にこう定理で解けるやろ

213 :132人目の素数さん:2018/05/11(金) 17:04:40.86 ID:0yS46dEA.net
>>209
α=a+bi, θ=arctan(b/a)とおいてmθ∈πNと仮定する。
とくにmとしてこれを満たす最小の自然数をとる。
exp(iθ) ∈ Q(α,√(a^2+b^2))により
φ(m)=[Q(exp(iθ))]≦[Q(α,√(a^2+b^2)):Q]≦4。
∴φ(m)≦4
∴m = 1,2,3,4,5,6,8,12
このなかでtan(θ)=b/aが有理数であるのはm=4のみ。
とくに(a+bi)^pが実数となるのはpが4の倍数のときのみ。

214 :132人目の素数さん:2018/05/11(金) 17:11:43.54 ID:sUzPr1Ik.net
>>211
高2です、数V一通り終わって受験対策とかやってます

215 :132人目の素数さん:2018/05/11(金) 17:12:13.21 ID:yjsU5oy7.net
複素数平面上で、右の図のように、異なる3点O(0)、A(α)、B(β)を頂点とする△OABの外側に、辺OA、辺OBをそれぞれ斜辺とする直角二等辺三角形OAC、OBDを作る。このとき、辺ABの中点をMとするとCM=DM、CM⊥DMであることを複素数を用いず証明せよ。
https://imgur.com/a/oKyaclT
写真貼れてるかどうか心配ですがよろしくお願いします。

216 :132人目の素数さん:2018/05/11(金) 17:13:23.79 ID:x4I+N5Vk.net
>>214
受験勉強やれよ、入試に落ちるぞ

217 :132人目の素数さん:2018/05/11(金) 17:33:32.98 ID:MgeDrhnB.net
>>209
0<k<p では  pCk = 0 mod p より
 (a+bi)^p = a^p +b^p i^p mod p
p>2 より、pは奇数。よって、(a+bi)^pは実数ではない。

218 :132人目の素数さん:2018/05/11(金) 17:45:10.62 ID:c/H/a+yG.net
井山裕太氏が囲碁の世界には進まず、普通に勉強してたら、東大理Vに首席で合格できたと思いますか?

219 :132人目の素数さん:2018/05/11(金) 17:52:12.43 ID:MgeDrhnB.net
>>217 これはダメだね。

220 :132人目の素数さん:2018/05/11(金) 18:35:42.29 ID:/24RCwpQ.net
>>174
さんきゅー
仕事がはかどります!

あなたの回答は我が国のGDPに
これから貢献することになるでしょう。

221 :132人目の素数さん:2018/05/11(金) 18:39:03.80 ID:8AeqVxxw.net
しょうもない仕事

222 :132人目の素数さん:2018/05/11(金) 19:11:01.74 ID:fMewDKvN.net
>>215できたけど全部文字に起こす気にならん。図って偉大だなぁ

223 :132人目の素数さん:2018/05/11(金) 19:13:51.64 ID:zUp9NIGk.net
問題
数直線全体で定義される何回でも微分できる関数f(x)で次の条件を満たすものを見つけよ:
ある1でない正定数αと多項式Pが存在して、数直線全体でf(αx)=P(f(x))を満たす。
例. cos(3x)=4cos^3(x)-3cos(x).

224 :132人目の素数さん:2018/05/11(金) 19:28:17.91 ID:c/H/a+yG.net
無と全が戦ったらどっちが勝ちますか?

225 :132人目の素数さん:2018/05/11(金) 20:11:52.89 ID:sSFRGylu.net
>>222
全部文字に起こすのが面倒くさいなら方針だけでも教えていただけますか。

226 :132人目の素数さん:2018/05/11(金) 20:57:57.47 ID:2MTeB4Yq.net
中3の中間試験の因数分解お願いします

8a^2b-2a+4a^2c-c


…問題間違えてないですか?これ

227 :132人目の素数さん:2018/05/11(金) 22:05:55.75 ID:sylrZDp2.net
>>226
http://www.wolframalpha.com/input/?i=factor+8a%5E2b-2a%2B4a%5E2c-c

228 :132人目の素数さん:2018/05/11(金) 22:09:16.40 ID:fMewDKvN.net
>>225
O中心にCDMを2倍に拡大した点をC’D’M’、ベクトルOX=ベクトルD’M’となるようにXをとると△AM’C’が△AXOをA中心に90度回したものになる

229 :132人目の素数さん:2018/05/11(金) 22:29:08.12 ID:s715UC9R.net
>>226
 8a^2b-2b+4a^2c-c  の誤植か?

230 :132人目の素数さん:2018/05/11(金) 23:17:17.69 ID:2MTeB4Yq.net
>>229
ありがとうございます

たぶんそうですよね!
与式=(2a-1)(2a+1)(2b+1)
かー

231 :132人目の素数さん:2018/05/11(金) 23:26:09.50 ID:7F80ojQi.net
>>230
最後油断したね?

232 :132人目の素数さん:2018/05/11(金) 23:52:41.19 ID:sUzPr1Ik.net
次の式を因数分解せよ。
a^3+b^3+c^3-4n(ab+bc+ca)+abc

233 :DJgensei artchive gemmar:2018/05/12(土) 11:57:24.23 ID:pKtCKnP+.net
因数分解って、稼働区域のパターンのことだから、動きを重点にしてね。

234 :132人目の素数さん:2018/05/12(土) 20:05:54.07 ID:SCW0csPu.net
f(x)=x(1/x)の定義域を述べ、xを限りなく0 に近づけたときのf(x)の挙動を調べよ。

235 :132人目の素数さん:2018/05/12(土) 20:08:56.71 ID:oLpBza7h.net
>>234
ばかすか?

236 :132人目の素数さん:2018/05/12(土) 20:15:42.80 ID:PmaOoNNr.net
高校数学で f(x)=√(1-x^2) の定義域を求めよとかいうふざけた問題あるよね

237 :132人目の素数さん:2018/05/12(土) 21:04:16.27 ID:oLpBza7h.net
>>236
普通の問題

238 :132人目の素数さん:2018/05/12(土) 21:09:06.91 ID:LdhTEb90.net
複素平面全体かな?

239 :132人目の素数さん:2018/05/12(土) 21:10:52.32 ID:SCW0csPu.net
>>232
これ誰もできないんすか?

240 :132人目の素数さん:2018/05/12(土) 21:16:42.87 ID:1opnZLPA.net
>>239
ペアノ算術を含む任意の無矛盾な公理系に対し、あるモデルM,Nおよび論理式φが存在して、M|=φかつN|≠φとできることを示せ、という問題がわかりません

よろしくお願いします

241 :132人目の素数さん:2018/05/12(土) 21:21:44.36 ID:1opnZLPA.net
>>239
本当にわからないので、なるべく早く回答いただけるとありがたいですね

242 :132人目の素数さん:2018/05/12(土) 21:36:57.99 ID:oLpBza7h.net
>>241
分からないんですねw

243 :132人目の素数さん:2018/05/12(土) 21:38:35.79 ID:1opnZLPA.net
回答がありませんね


まさかとは思いますが、わからないんですか?

244 :132人目の素数さん:2018/05/12(土) 21:48:36.62 ID:PmaOoNNr.net
劣等感婆さん久しぶり!

245 :132人目の素数さん:2018/05/12(土) 21:57:06.02 ID:eiK+3dSt.net
わからなあなぁ

246 :132人目の素数さん:2018/05/12(土) 21:58:04.16 ID:b2J32MDf.net
閉リーマン面、C上1変数代数関数体、C上非特異射影代数曲線にはそれぞれ対応がある(反変圏同値?)らしいですが、
コンパクトn次元複素多様体、C上n変数代数関数体、C上非特異n次射影代数多様体も同様の対応が成り立つのでしょうか?

247 :132人目の素数さん:2018/05/12(土) 22:32:03.07 ID:eiK+3dSt.net
>>246
関数体からC上非特異n次射影代数多様体へは広中の定理でいけそうだけどコンパクトn次元複素多様体へいけるかなぁ?
畑違いなので自身ないけど。
たしか代数的に限ってもプロジェクティブでないのが作れるって聞いたことあるからダメな気がする。
ハーツホーンの練習問題にあった記憶が……

248 :132人目の素数さん:2018/05/12(土) 22:34:18.13 ID:SCW0csPu.net
空間の円C:x^2+y^2=1,z=0を底面とし、点(0,0,1)を頂点とする円錐面のうち、y≧0の部分をKとする。
また、Cのy≦0の部分をC'とする。
C'上を点Pが動くとき、A(0,1,tan75°)とPを結ぶ直線がKと交わる点Qの軌跡を図示せよ。
ただしKの側面の展開図上に図示すること。

249 :132人目の素数さん:2018/05/12(土) 22:36:39.66 ID:ERQyPxVg.net
>>212 >>217

 (a + bi)^p = {a^p - pC2・a^(p-2)・b^2 + ……} + {pC1 a^(p-1)・b - pC3・a^(p-3)・b^3 + ……}i

 bがpの倍数ならば、aはpの倍数ではない。
b = B・p^m となる最大の自然数をmとする。
 
虚数部は
 p・a^(p-1) B・p^m - p(p-1)(p-2)/6・a^(p-3)・B^3・p^(3m) + …… ≡ a^(p-1)・B・p^(m+1)
 ≠ 0  (mod p^(3m+1))
よって、(a+bi)^p は実数ではない。

250 :132人目の素数さん:2018/05/12(土) 22:49:06.21 ID:eiK+3dSt.net
>>248

> 図示せよ。

どないせぇっちゅうねん

251 :132人目の素数さん:2018/05/12(土) 22:49:33.56 ID:1opnZLPA.net
>>248
ある無矛盾な公理系τの任意のモデルに対してある論理式φが常に真となるならば、τからφがLKにおいて証明可能となることを示せ、という問題がわかりません

252 :132人目の素数さん:2018/05/12(土) 22:59:22.56 ID:rulsrvzO.net
Σ[n=-∞,∞]1/((ni-a)(ni-b))

を求めてください
iは虚数単位で、a,bは実数で、a≠bです
aもbも0出ない時と、どちらかが0の時の場合の両方を求めてくださるとありがたいです

253 :132人目の素数さん:2018/05/12(土) 23:18:57.44 ID:Kac7O/0/.net
>>243
何をしたらよいのか、わからんなあ

254 :132人目の素数さん:2018/05/12(土) 23:38:12.32 ID:xLUYWUsP.net
>>232
因数分解されるとしたら
(aについて0次)(aについて3次) または (aについて1次)(aについて2次)
となるしかない。

前者の場合
 f(b,c)*(g(b,c)a^3+(aについて2次以下))
という形だが、a^3 の係数を考えると f(b,c) が定数でなければならず不適。

後者の場合
上と同様に a^3 の係数について考えれば
 (a+f(b,c))(a^2+g(b,c)a+h(b,c))
の形に分解できるはず。
定数項を比較して
 f(b,c)h(b,c)=b^3+c^3-4nbc

ここで b^3+c^3-4nbc の因数分解を考える。
もし因数分解されるとしたら、b^3 の係数が 1 なので、上と同様に
(b+F(c))(b^2+G(c)b+H(c))
の形に分解できるはず。
定数項を比較して
 F(c)H(c)=c^3
よって、F(c) は実数 k を用いて k, kc, kc^2, kc^3 のいずれかの形。
このとき、b^3+c^3-4nbc に b=-F(c) を代入すれば (c によらず) 0 になるはずだが、
得られた F(c) の候補のどれを代入してもそうならないので矛盾。
したがって b^3+c^3-4nbc は既約。

f(b,c)h(b,c)=b^3+c^3-4nbc より、f(b,c) は定数または k(b^3+c^3-4nbc) の形。
このとき、a^3+b^3+c^3-4n(ab+bc+ca)+abc に a=-f(b,c) を代入すれば (b,c によらず) 0 になるはずだが、
得られた f(b,c) の候補のどれを代入してもそうならないので矛盾。
したがって a^3+b^3+c^3-4n(ab+bc+ca)+abc は既約。

本当に高校生なのかどうかは知らないけど、あまり大人を試すような真似をするんじゃありませんよ

255 :132人目の素数さん:2018/05/12(土) 23:39:14.03 ID:/fKwCW3Q.net
wolframalpha に聞け
とだけ言っとけ

256 :132人目の素数さん:2018/05/12(土) 23:44:02.93 ID:ERQyPxVg.net
>>252
π{coth(πb) - coth(πa)}/(a-b)

(解)
1/{(a-n・i)(b-n・i)} = {1/(b-n・i) - 1/(a-n・i)}/(a-b),
1/{(a+n・i)(b+n・i)} = {1/(b+n・i) - 1/(a+n・i)}/(a-b),
辺々足したのち、次を使う。
Σ[n=1,∞] {1/(a-n・i) + 1/(a+n・i)} + 1/a
= Σ[n=1,∞] 2a/{(a-n・i)(a+n・i)} + 1/a
= π coth(πa),

a→0 のときは、n=0の項が発散しそうな希ガス…

257 :132人目の素数さん:2018/05/12(土) 23:59:07.49 ID:AcZ8M7Y5.net
>>252
どっちも0でないときは
Σ[n=-∞,∞]1/((ni-a)(ni-b))
=1/(ai-bi)Σ[n=-∞,∞](1/(n+ai)-1/(n+bi))
=1/(ai-bi)Σ[n=1,∞](1/(n+ai)+1/(-n+ai))
-1/(ai-bi)Σ[n=1,∞](1/(n+bi)+1/(-n+bi)) + 第0項
=-π/(ai-bi)cotπai + π/(ai-bi)cotπbi - 1/(ai-bi)(1/ai-1/bi) + 第0項
https://ja.wikipedia.org/wiki/%E4%B8%89%E8%A7%92%E9%96%A2%E6%95%B0%E3%81%AE%E9%83%A8%E5%88%86%E5%88%86%E6%95%B0%E5%B1%95%E9%96%8B
どっちか0のときは第0項が計算不能。

258 :132人目の素数さん:2018/05/13(日) 00:00:21.16 ID:iNO2x29a.net
かぶったorz

259 :132人目の素数さん:2018/05/13(日) 01:58:35.15 ID:mSJNHrCr.net
>>250
(001)からの距離を与えたら?

260 :132人目の素数さん:2018/05/13(日) 01:58:52.74 ID:mSJNHrCr.net
>>251
恥を知らないのですねw

261 :132人目の素数さん:2018/05/13(日) 02:09:45.41 ID:I2s0crdy.net
>>256
それもそうでした!
ありがとうございます!

262 :132人目の素数さん:2018/05/13(日) 02:52:46.02 ID:cuL3jHY3.net
>>259
試しにやってみたらこれ問題間違ってないか?
出せなくはないけど(0,1,tan 75°)が全然行きてない。
頂点が(0,0,0)でCとC’の境目がz=0ならちょっといい感じだけど。
z座標の設定変えてAの座標とか境目の座標そのままにしちゃったんじゃない?
2円錐の共通域出すだけのしょうもない作業のわりには数値うるさすぎて下らない。

263 :132人目の素数さん:2018/05/13(日) 02:56:42.81 ID:dQIRZ6zE.net
>>4

nが奇数のとき
 f(x) = {x(1-x)}^{(n-1)/2} (2x-1),
 Max{ |f(x)| ; 0≦x≦1 } = (1/2)^(n-1) √{(n-1)^(n-1) / (n^n)}, x = 1/2 ± 1/(2√n),

nが偶数(n≧4)のとき
 f(x) = {x(1-x)}^(n/2 -1) (2x-1)^2,
 Max{ f(x) ; 0≦x≦1 } = (1/2)^(n-3) √{(n-2)^(n-2) / (n^n)}, x = 1/2 ± 1/√(2n),

nが偶数(n≦4)のとき
 f(x) = {x(1-x)}^(n/2),
 Max{ f(x) ; 0≦x≦1 } = (1/2)^n, x = 1/2,

エレガントな解答スレ2-813

264 :132人目の素数さん:2018/05/13(日) 05:27:32.19 ID:9cj8IIRg.net
1,2,3 の各数字が1つだけ書かれたカードが袋の中に大量にある。
袋からカードを1枚引き、そのカードに書かれた数字を記録する操作を繰り返す。どの数字のカードを引くかは同様に確からしいとする。
kを自然数とする。

(1)操作を繰り返し、記録された数字の和がちょうどkになるか、k+1またはk+2になった時点で操作を終了する。和がkになる確率P(k)をkで表せ。

(2)lim[k→∞] P(k) を求めよ。

265 :132人目の素数さん:2018/05/13(日) 05:46:15.31 ID:e0GLK8FT.net
Gを位数nの有限群、dをnの約数とするとき、x^d=1をみたすxがd個より多くなる例を教えて下さい

266 :132人目の素数さん:2018/05/13(日) 07:57:55.69 ID:mBXeE/BU.net
>>264
(1)
p(-2) = p(-1) = 0, p(0) = 1と定めておいて
p(k+3) = (1/3((p(k+2)+p(k+1)+p(k))
(2)
1/3

267 :132人目の素数さん:2018/05/13(日) 08:00:35.82 ID:GygA6/J7.net
>>265
Gを位数2の巡回群の2個の直積として
位数が2の約数の元の数=4>2

268 :132人目の素数さん:2018/05/13(日) 08:38:00.31 ID:h0T9Njyo.net
>>266
1/6 orz

269 :132人目の素数さん:2018/05/13(日) 08:47:41.18 ID:ZQnefJyn.net
>>266
1/2 orz

270 :132人目の素数さん:2018/05/13(日) 10:03:17.57 ID:brauS78B.net
最小二乗法について教えてください。

(x1, y1), (x2, y2), …, (xn, yn)とする、n個のxとyの値が分かっているペアがあります。これらが以下の方程式

y = a * {sinh(bx)}^c

を満たす場合、最小二乗法を使って係数a, b, cを求めたいです。

どうにもうまく求められなかったため、分かる方求め方を教えてください。

また、最小二乗法ではない方法でなら求められるのであれば、その方法でもokです。

よろしくおねがいします。

271 :132人目の素数さん:2018/05/13(日) 10:56:45.38 ID:9cj8IIRg.net
正方形ABCDがあり、4点A,B,C,Dはこの順に反時計回りに並んでいる。
辺CDを1:3に内分する点をE、線分EAを1:4に内分する点をF、BCの中点をMとする。
このとき、∠MAE=∠FOEを示せ。

272 :132人目の素数さん:2018/05/13(日) 11:07:17.86 ID:fpkwHaKc.net
O?

273 :132人目の素数さん:2018/05/13(日) 11:12:12.65 ID:9cj8IIRg.net
>>271
訂正
O→M

274 :132人目の素数さん:2018/05/13(日) 12:13:20.86 ID:dQIRZ6zE.net
>>271 >>273

AB:BM = MC:CE
∠B = ∠C = 90°
∴ 儁AB ∽ 僞MC
MA:ME = AB:MC = BM:CE = 2:1
∠AME = 90°
Mから対辺AEに垂線MHを下す。
儁EH ∽ 僊MH ∽ 僊EM
∴ AH:HM = HM:HE = AM:ME = 2:1
∴ AH:HE = 4:1
∴ H = F
∴ ∠MAE = ∠HME = ∠FME

275 :132人目の素数さん:2018/05/13(日) 12:15:34.32 ID:mSJNHrCr.net
>>266
>p(k+3) = (1/3((p(k+2)+p(k+1)+p(k))
q(k+2)=p(k+3)-p(k+2)=-2/3(p(k+2)-p(k+1))-1/3(p(k+1)-p(k))=-2/3q(k+1)-1/3q(k)
3q(k+2)+2q(k+1)+q(k)=0
3t^2+2t+1=0
t=(-1±i√2)/3=t±
q(k)=A(t+)^k+B(t-)^k
p(k)-p(0)=A(1-(t+)^(k+1))/(1-t+)+B(1-(t-)^(k+1))/(1-t-)
p(∞)-1=A/(1-t+)+B/(1-t-)=A/(5/3+t-)+B/(5/3+t+)=(A(5/3+t+)+B(5/3+t-))/(5/35/3+(t++t-)+t+t-)=(5/3q(0)+q(1))/(25/9-2/3+1/3)=(5/3(p(1)-p(0))+(p(2)-p(1)))/(22/9)=(p(2)+2/3p(1)-5/3p(0))/(22/9)=(4/9+2/31/3-5/3)/(22/9)=(4+2-15)/22=-9/22
p(∞)=1-9/22=11/22=1/2

276 :132人目の素数さん:2018/05/13(日) 12:16:38.80 ID:mSJNHrCr.net
>>275
>p(∞)=1-9/22=11/22=1/2
p(∞)=13/22

277 :132人目の素数さん:2018/05/13(日) 12:34:54.03 ID:dQIRZ6zE.net
>>267
 Klein の vierer Gruppe(V)

278 :132人目の素数さん:2018/05/13(日) 12:37:29.56 ID:mSJNHrCr.net
>>275
>(5/35/3+(t++t-)+t+t-)=
(5/35/3+5/3(t++t-)+t+t-)=2
p(∞)=1-9/2222/91/2=1/2

279 :132人目の素数さん:2018/05/13(日) 12:39:56.60 ID:mSJNHrCr.net
>>269
むしろなぜこの値なのか
簡単な理由がありそうな

280 :132人目の素数さん:2018/05/13(日) 12:53:34.46 ID:fpkwHaKc.net
>>271
△ABMと△MCDが相似で相似比は2:1。
MからAEに垂線の足Gを下ろしたとき△AME、△AGM、△EGMは相似で辺の比は1:2:√5。特にAG:GE=4:1でF=G。

281 :132人目の素数さん:2018/05/13(日) 13:20:40.90 ID:9cj8IIRg.net
xy平面の点A(1,1)を通る直線と円C:x^2+y^2=1が交点を持つとき、その交点のAに近い方をPとする(ただ一つの交点を持つ場合はそれをPとする)
また、この直線上のAから見てPの側に点Qをとり、AP・AQ=kとなるようにする。ここでkは正の定数である。

(1)点Qが動いてできる曲線Kにより、円Cの内部が面積が等しいように二分される場合のkの値を求めよ。

(2)(1)のとき、KとCとの2交点をそれぞれS,Tとする。sin(∠SAT)≧(i/10)となる最大の整数を求めよ。

282 :132人目の素数さん:2018/05/13(日) 13:56:16.96 ID:XAMB7xRz.net
0.999…=1 の説明ってさ、「実数は連続であるから」でいいよね?

283 :132人目の素数さん:2018/05/13(日) 14:18:16.41 ID:NMjJwVYY.net
連続をどのような意味で使っているか、疑問ではあるが、一言で言うなら、小数の表現の性質。
同じ値に対し、複数の表現方法があるのに、この事実を知らず、不思議がっている人がいるだけ。

284 :132人目の素数さん:2018/05/13(日) 14:40:09.37 ID:ig+KKpxF.net
まずは無限小数の定義を考えるところから
それさえわかれば上に有界な単調増加だから収束することは実数の連続性
収束値が1になることは明らか

285 :132人目の素数さん:2018/05/13(日) 16:08:15.61 ID:9cj8IIRg.net
>>283
10進法で1を表現する方法は1と0.9999...以外にありますか?

286 :132人目の素数さん:2018/05/13(日) 16:55:37.57 ID:mSJNHrCr.net
>>285
ない

287 :132人目の素数さん:2018/05/13(日) 17:02:04.33 ID:1Hs4Nvmc.net
1.0000... は?

288 :132人目の素数さん:2018/05/13(日) 17:03:43.23 ID:bjhAtIi0.net
0.111111111111111111111111111111111111111111111111・・・・・・・・・・・・・・X9

289 :132人目の素数さん:2018/05/13(日) 17:17:53.34 ID:mSJNHrCr.net
>>287
それ許すなら無限にあるわけだが

290 :132人目の素数さん:2018/05/13(日) 17:40:44.73 ID:1Hs4Nvmc.net
>>289
形式的計算で 2-0.99999…=1.00000… だから
0.99999…と1.00000…はそれぞれ下からと上から近づく2パターンの表現
それ以外の無数の表現の例はどんなのがあるの?

291 :132人目の素数さん:2018/05/13(日) 17:52:41.25 ID:mSJNHrCr.net
1.0

292 :132人目の素数さん:2018/05/13(日) 17:53:24.41 ID:mSJNHrCr.net
>>290
>上から近づく


293 :132人目の素数さん:2018/05/13(日) 18:01:06.42 ID:1Hs4Nvmc.net
>>292
0.999…=lim[ε→-0](1+ε)
1.000…=lim[ε→+0](1+ε)

294 :132人目の素数さん:2018/05/13(日) 18:20:06.25 ID:1Hs4Nvmc.net
>>291
例ありがとう。

でも有限で0が終わったら厳密に1に等しいけど
無限に0が続く場合それが1に等しいことは自明ではない気がする

295 :132人目の素数さん:2018/05/13(日) 18:34:16.99 ID:9cj8IIRg.net
次の性質(A)を持つ立体は存在しないことを示せ。

(A)どのような平面で切っても、切断面の面積は常に同じ値をとる。

296 :132人目の素数さん:2018/05/13(日) 18:36:47.36 ID:mSJNHrCr.net
>>293
何それ?

297 :132人目の素数さん:2018/05/13(日) 18:37:30.46 ID:mSJNHrCr.net
>>294
>無限に0が続く場合
とはどういう定義か考えてないの?

298 :132人目の素数さん:2018/05/13(日) 18:50:56.10 ID:NsIUdaFs.net
z=cosθ+i sinθのとき
⑴2cosθ=z+1/zを示せ
⑵2 cosnθ=z^n+1/z^nを示せ(分母にだけn)
⑶3z^4-z^3+2z^2-z+3=0のとき
 a, 6cos2θ-2cosθ+2=0
b. 方程式の4つの実数ではない解を示せ

この問題たちがわからない、だれか部分部分でもいいので教えてください

299 :132人目の素数さん:2018/05/13(日) 19:12:07.48 ID:gywqs905.net
>>298
3.(a)意味ぷー
(b)3z^4-z^3+2z^2-z+3=(3z^2-4z+3)(z^2+z+1)と2次の判別式。

300 :132人目の素数さん:2018/05/13(日) 19:25:43.07 ID:NsIUdaFs.net
>>299 thx ごめん、3のaは3z^4-z^3+2z^2-z+3=0を6cos2θ-2cosθ+2=0の形に変形できることを示せ、でしたm(_ _)m

301 :132人目の素数さん:2018/05/13(日) 19:53:28.92 ID:zoXfCvTV.net
>>300
そんなこと成り立たんでしょ?3z^2-4z+3=0の解は
z=(1/3)(2±√5i)なので絶対値は1とは限らん。

302 :132人目の素数さん:2018/05/13(日) 19:54:31.64 ID:ig+KKpxF.net
(1)オイラーの公式
(2)オイラーの公式

303 :132人目の素数さん:2018/05/13(日) 21:13:45.97 ID:w1azPazB.net
>>300>>301
寝ぼけてた。>>301
>z=(1/3)(2±√5i)
これ絶対値1やね。でも絶対値が全部1であること示すよりz+1/z=tとおいて
与式⇔3(t^2-2)-t+2=0⇔t=4/3,-1
を出してz+1/z=-1とz+1/z=4/3とく方が早い。誘導どうりやった方がしんどいクソ誘導は無視すべし。

304 :132人目の素数さん:2018/05/13(日) 21:36:55.44 ID:dQIRZ6zE.net
>>298 >>300

(b) 3zz -z +2 -1/z +3/zz = 3(z+1/z)^2 - (z+1/z) - 4 = {3(z+1/z) -4} {(z+1/z) +1},

 z+1/z = -1 から z = (-1±i√3)/2 = e^(±i(2π/3)),

 z+1/z = 4/3 から z = (2±i√5)/3,

305 :132人目の素数さん:2018/05/14(月) 00:10:09.98 ID:ySndPn+E.net
https://i.imgur.com/Bebbdo4.jpg

306 :132人目の素数さん:2018/05/14(月) 00:13:38.70 ID:N5/oSqy7.net
>>267
>>277
ありがとうございます

307 :132人目の素数さん:2018/05/14(月) 00:14:59.96 ID:N5/oSqy7.net
もう一つ質問なのですが整域上の次数nの多項式で根をn個以上持つものはありますか?

308 :132人目の素数さん:2018/05/14(月) 00:25:51.14 ID:s90IxSDn.net
>>256
ちなみにですけど、この無限級数がπcoth(πa)の形で表される事ってどのように導出しましたか?

309 :132人目の素数さん:2018/05/14(月) 00:31:49.43 ID:cV/gIJVZ.net
>>307
四元数でx^2=-1とか

310 :132人目の素数さん:2018/05/14(月) 00:41:46.40 ID:V8xenapF.net
整域上のn次多項式は、その整域の商体上のn次多項式

311 :132人目の素数さん:2018/05/14(月) 00:42:29.22 ID:qdNB/X3F.net
>>307>>309
整域って言ったら普通は可換性を仮定すると思う。
そうすると存在しない。

312 :132人目の素数さん:2018/05/14(月) 00:44:19.98 ID:BPMfd3hq.net
>>281

交点のAから遠いほうの交点を P~ とする(ただ一つの交点を持つ場合はそれを P=P~ とする。)
方べきの定理より、AP・AP~ = 1,
題意より、AQ = k・AP~
∴ Kは、円C(x>0 かつ y>0 の部分を除く)を、Aを中心としてk倍したもの。
∴ Kは、中心が (1-k,1-k) 半径がkの円(x>1-k かつ y>1-k の部分を除く)
 (x-1+k)^2 + (y-1+k)^2 = k^2,

(1) k = 0.728967367687286

(2) ∠SOT /2 = α とおく。
 cosα = (3-k)/√8 = 0.802931287302128
 sinα = 0.596071596262855
 tan(∠SAT /2) = sinα・(√8)/(1+k)
 ∠SAT = 1.54560093958281 < π/2
 sin(∠SAT) = 0.99968261302211979

相変わらずセンスのない作問者…

313 :132人目の素数さん:2018/05/14(月) 00:47:19.25 ID:N5/oSqy7.net
ありがとうございます
では整域を可換環に置き換えると存在するでしょうか?

314 :132人目の素数さん:2018/05/14(月) 00:57:39.14 ID:rt0PhzAS.net
線形代数ってなんですか?
何が線形なんですか?
a11 a12
a21 a22
☝これの何が線形なんですか

315 :132人目の素数さん:2018/05/14(月) 00:57:47.28 ID:V8xenapF.net
>>313
Z/(4)上の2x^2+2x=0

316 :132人目の素数さん:2018/05/14(月) 01:05:05.51 ID:cHwVvG6S.net
>>314
難しいこと考える前に計算できるようにしときましょう

317 :132人目の素数さん:2018/05/14(月) 01:10:24.42 ID:V8xenapF.net
>>314
一次方程式の解法理論を整理したものが線形代数の始まり。

318 :132人目の素数さん:2018/05/14(月) 01:10:49.92 ID:N5/oSqy7.net
>>315
ありがとうございます

319 :132人目の素数さん:2018/05/14(月) 01:38:22.71 ID:V8xenapF.net
零因子のある可換環で多項式関数値が零因子になるような理論はまだ未整備か?
>>315では x^2+x の x に対象としている環のどの元を代入しても零因子、もしくは0になる例

320 :132人目の素数さん:2018/05/14(月) 02:03:39.99 ID:lmZSS4qL.net
>>312
数値汚くて計算するだけだった?
すまん

321 :132人目の素数さん:2018/05/14(月) 02:19:05.29 ID:jdLJYi70.net
>>305
これどこの問題?

322 :132人目の素数さん:2018/05/14(月) 02:19:57.04 ID:jdLJYi70.net
>>320
自作問題はよそでやってくれ

323 :132人目の素数さん:2018/05/14(月) 02:34:46.08 ID:dKKInaV2.net
6^10,000の頭16桁の数字を教えてください

324 :132人目の素数さん:2018/05/14(月) 02:38:15.96 ID:F8vZ30dB.net
1つの群には複数の環構造が入りますか?
つまりA,Bを環としてAとBが加法群として同型でも環としては同型でないことはありますか?

325 :132人目の素数さん:2018/05/14(月) 02:44:07.84 ID:V8xenapF.net
そんな程度なら、片一方を0環にすればいいだけの話だろ。
も少し建設的な問題を設定せよ。

326 :132人目の素数さん:2018/05/14(月) 03:26:12.87 ID:Sbvhabun.net
>>323
3254646585493662

327 :132人目の素数さん:2018/05/14(月) 04:45:06.25 ID:N5/oSqy7.net
>>319
ありがとうございます
最近勉強を始めたばかりで殆ど何も知りません
それについて詳しく書かれている本はありますか?(もしくは通常数行で済ませてしまうようなことでしょうか?)

328 :132人目の素数さん:2018/05/14(月) 04:54:51.17 ID:vWTDt18w.net
>>325
環は単位的なものを想定していました

329 :132人目の素数さん:2018/05/14(月) 10:04:11.06 ID:IhdOpBbv.net
自分で作った問題で自分で解けないときならスレ違いではないと思うけどそれならそれでその旨は書いといてほしい。
そういうのは解けない、解けるにしてもドエライ解になってしまうかもしれないから。

330 :132人目の素数さん:2018/05/14(月) 10:10:40.17 ID:ox0+BiKo.net
>>305
左辺を微分すると↓
http://www.wolframalpha.com/input/?i=d(integrate%5B(x-t)+f(t),%7Bt,pi%2F2,x%7D%5D)%2Fdx
この式で x = Π/2 とすればもちろん値は 0 となるが
右辺を微分した式で x = Π/2 としても 0 にはならない

331 :132人目の素数さん:2018/05/14(月) 10:34:40.92 ID:C/cyQfU2.net
wolfram間違ってない?

332 :132人目の素数さん:2018/05/14(月) 10:43:53.74 ID:6uJ2QcNR.net
さすがのwolframも未定の関数が入ってる式は処理できないんだなぁ。

333 :132人目の素数さん:2018/05/14(月) 12:02:55.53 ID:dKKInaV2.net
>>326
ありがとうございます

334 :132人目の素数さん:2018/05/14(月) 15:06:51.16 ID:SGjgBc66.net
日本民法の父、穂積陳重の『法窓夜話』を現代語に完全改訳

法律エッセイの古典的名著が短編×100話で気軽に読めます
リライト本です。「なか見検索」で立ち読み頂けます。原版は
国立国会図書館デジタルコレクションで無料で読めます

法窓夜話私家版 (原版初版1916.1.25)
https://www.amazon.co.jp/dp/B07BT473FB
(続)法窓夜話私家版 (原版初版1936.3.10)
https://www.amazon.co.jp/dp/B07BP9CP5V

335 :132人目の素数さん:2018/05/14(月) 16:04:25.17 ID:F+1vy2oQ.net
あるところを境に全てが0になる規則性をもった数列って存在しますか?
あるとしたらどんな一般項になるんでしょう
一応自分が考えた案としては、前の項+前の項×なんらかの数列(例えばマイナスから始まる奇数項)なんですが、これの一般項の求め方がわかりません
教えて下さい

336 :132人目の素数さん:2018/05/14(月) 16:46:26.29 ID:n2mfaafD.net
東京大学理学部数学科に入りたい。

337 :132人目の素数さん:2018/05/14(月) 18:12:16.53 ID:SJeCUFf8.net
すみません、わからない問題というか、質問なんだけど

「あなたは自分にとって∫f(x)dxにおけるdxだ」

と言われた場合のdxって何ですか?
たぶん告白だろうとは思うんですが、きちんと意味をつかんでから返事がしたいので
文系の自分にはさっぱりです

338 :132人目の素数さん:2018/05/14(月) 19:12:06.43 ID:VRvJuuxP.net
>>337

まらん

339 :132人目の素数さん:2018/05/14(月) 19:28:25.94 ID:gmAMWTBM.net
>>317
ああ、それはそうですね
寝ぼけたこと書いてました

340 :132人目の素数さん:2018/05/14(月) 19:39:16.54 ID:lmZSS4qL.net
>>335
a1が非負
a_(n+1)=a_(n)-[√(a_n)]

341 :132人目の素数さん:2018/05/14(月) 20:39:16.96 ID:XUjOMsY1.net
質問です。
任意の自然数nに対して、次の不等式が成り立つことを示せ。
2^(n+2)>n^2
という問題なのですがこれって任意の自然数じゃなくてn≧3のときですよね?

342 :270:2018/05/14(月) 20:39:23.72 ID:7Rk7XC5X.net
>>270もお願いします。

343 :132人目の素数さん:2018/05/14(月) 20:40:53.69 ID:7Rk7XC5X.net
>>341

いや、どう見てもn=1, n=2でも成り立つだろ

344 :132人目の素数さん:2018/05/14(月) 21:06:09.18 ID:XUjOMsY1.net
>>343
すいません。解けましたありがとうございます

345 :132人目の素数さん:2018/05/14(月) 22:26:03.02 ID:xg2RAmH0.net
⑴zとwが複素数(x+yi)で、w=1/1-z, (絶対値z)^2=1のとき、wの実数部分xを求めよ
⑵zがcisθのときz^2-1/z^2+1=i tanθを証明せよ
誰かこの二つお願いm(_ _)m
cisはr (cosθ+i sinθ)

346 :132人目の素数さん:2018/05/14(月) 22:34:06.26 ID:lmZSS4qL.net
>>345
zに代入して実部と虚部を計算するだけ

347 :132人目の素数さん:2018/05/14(月) 22:58:14.28 ID:48jxLhgY.net
>>345
(2)何かおかしいね。z^2-1/z^2が純虚数でそこに1足して純虚数になるはずない。

348 :132人目の素数さん:2018/05/14(月) 23:12:49.35 ID:s90IxSDn.net
>>308
お願いします

349 :132人目の素数さん:2018/05/14(月) 23:18:34.60 ID:VYiPAVkp.net
タイラー天気じゃないのかな?

350 :132人目の素数さん:2018/05/14(月) 23:22:48.86 ID:zLqYWFds.net
この問題を間違えたんですけど質問です
https://i.imgur.com/8KiXbIT.jpg


cos45°=1/√2のはずですけど(Googleを信用するなら)回答ではcos45°=√2/2と実質なっています
https://i.imgur.com/PSQ7V7y.jpg

一体なにが間違ってるんでしょう

351 :132人目の素数さん:2018/05/14(月) 23:35:38.99 ID:GuJhfaMX.net
間違ってません

352 :132人目の素数さん:2018/05/14(月) 23:35:47.06 ID:48jxLhgY.net
>>348
一般にはHadamardの因数分解定理だけど三角関数とかだと初等的な証明もある。Wikipediaにも載ってるはず。

353 :132人目の素数さん:2018/05/15(火) 00:01:14.07 ID:01U/3ytA.net
>>331-332
手計算でも同じ式になるが

354 :132人目の素数さん:2018/05/15(火) 00:49:59.50 ID:KmuIpfTz.net
>>345 >>347
(z^2-1)/(z^2+1)だな

355 :132人目の素数さん:2018/05/15(火) 01:27:17.00 ID:eFrawDDn.net
>>308 >>348

無限乗積表示(オイラー):
 sinh(πa) = πa・Π[n=1,∞] {1 + (a/n)^2} = πa・Π[n=1,∞] {1 + (a/n)i}{1 - (a/n)i},
対数をとってaで微分する。
 π coth(πa) = 1/a + Σ[n=1,∞] 2a/(aa+nn) = 1/a + Σ[n=1,∞] {1/(a-in) + 1/(a+in)},

356 :132人目の素数さん:2018/05/15(火) 02:23:17.12 ID:Omn+setj.net
zは複素数で、複素数平面上の単位円上を動く。
複素数wをw=z+(z")^2+2z"とするとき、wが動いてできる曲線で囲まれる領域の面積をSとする。
(zの共役複素数をz"と表した)

(1)S≧nをみたす最大の非負整数nを求めよ。

(2)nは(1)で求めた値とする。
S≧n+(i/4)をみたす最大の非負整数iを求めよ。

357 :132人目の素数さん:2018/05/15(火) 02:29:40.74 ID:11UbkqUX.net
天上神と東大史上最高の天才はどっちの方が賢いですか?

358 :132人目の素数さん:2018/05/15(火) 02:49:14.33 ID:yTfs4dgd.net
>>356
複素数の問題で虚数単位以外の意味で使う i が同時に出てきたらあかんだろ
そして出題したいだけならよそにスレを立ててやってくれ
その方があとで参照するときにも都合がよい

359 :132人目の素数さん:2018/05/15(火) 03:31:19.89 ID:RWV1I2yh.net
>>348 >>308

おそらく最も単純な方法:f(z)=πcot(πz)/(z^2+a^2)と置いて留数定理を用いると目的の級数が得られる。

おそらく最も初等的な方法:三角関数の2N倍角の公式と根と係数の関係より
cot(x) = (1/(2N))Σ[n=0,2N-1]cot((x+πn)/(2N))
= (1/(2N))[cot(x/(2N)) + Σ[n=1,N-1]{cot((x+πn)/(2N)) - cot((-x+πn)/(2N))}]
が成り立ち、N→∞とすると和のペア部はO(1/n^2)で絶対収束するので極限の交換ができて
cot(x) = 1/x + Σ[n=1,∞]{1/(πn+x) - 1/(πn-x)}
そしてx=aπiと置くと目的の級数が得られる。

360 :132人目の素数さん:2018/05/15(火) 04:02:58.65 ID:eFrawDDn.net
>>356

z = e^(i・2π/3),z = -1,z = e^(i・4π/3) で w = -2 となる。(3重点)

∴3つの単純閉曲線が w = -2 で交わったもの…

361 :132人目の素数さん:2018/05/15(火) 11:00:03.52 ID:/SFsFp0F.net
>>356
https://www.wolframalpha.com/input/?i=parametric+plot+(((cos+t)%2B(cos+2*t)%2B2*(cos+t)),(-(sin+t)%2B(sin+2*t)%2B2*(sin+t)))
https://www.wolframalpha.com/input/?i=int_0%5E(2*pi)%5BD%5B(-(sin+t)%2B(sin+2*t)%2B2*(sin+t))%5D+*+((cos+t)%2B(cos+2*t)%2B2*(cos+t))%5D

362 :132人目の素数さん:2018/05/15(火) 16:20:33.30 ID:/SFsFp0F.net
今日Wolfram君に教えてもらいました。

integral_0^∞ x^(-s) sin(x) dx = cos((π s)/2) Γ(1 - s) for 0<Re(s)<2…(1)
integral_0^∞ x^(-s) cos(x) dx = sin((π s)/2) Γ(1 - s) for 0<Re(s)<1…(2)

(1)でs=1のとき Dirichlet積分、s=1/2のときFresnel積分となかなかかっっちょええ公式。
Wolfram君は不定積分も教えてくれて確かに微分して元の積分核が出ることもx=0のとき-右辺になることもチェックはできます
…が、こんなん思いつくかボケ!んなもん不定積分なんかせんでもHankelの公式で一撃じゃ
…でもなかったorz。
なんか(1)の左辺をHankelの公式で計算すると(1)と(2)の左辺が混ざった形がでてきて切り離せない???
どなたか初等的な証明(=留数定理とかCauchyの積分公式とか級数展開とかまで)知ってます?orできます?
もしかしてこの積分なんか名前ついてます?ちなみに数学辞典には(1)の方はのってます。

363 :132人目の素数さん:2018/05/15(火) 16:21:39.54 ID:/SFsFp0F.net
Wolfram大先生の計算
https://www.wolframalpha.com/input/?i=int_0%5E(infty)%5Bx%5E(-s)sin(x)%5D
https://www.wolframalpha.com/input/?i=int_0%5E(infty)%5Bx%5E(-s)cos(x)%5D

364 :132人目の素数さん:2018/05/15(火) 16:26:55.58 ID:/SFsFp0F.net
>>362
ちょっとまちがった。やってみたのは(1)の右辺をHankelの表示で(1)の右辺で計算していくと
(https://ja.wikipedia.org/wiki/%E3%82%AC%E3%83%B3%E3%83%9E%E9%96%A2%E6%95%B0)
どうしても
(〜)∫(cosπs/2)(cos(x)/x^s+(sinπs/2)sin(x)/x^s)dx
の形になってそこから先がどうしたもんやら……

365 :132人目の素数さん:2018/05/15(火) 17:16:23.18 ID:eFrawDDn.net
>>356

 z = e^(it),  -π≦t≦π.
とおける。
 w = e^(it) + e^(-2it) +2e^(-it)
 u = Re{w} = 3cos(t) + cos(2t),
 v = Im{w} = -sin(t) -sin(2t),

s(t1,t2) = ∫[t1,t2] u '(t) v(t) dt = -∫[t1,t2] u(t) v '(t) dt

>>360 により3つに分ければ
s(-π,-2π/3) = 5π/6 -(3√3)/2 = 0.0199176666
s(-2π/3,2π/3) = 10π/3 + 3√3 = 15.6681279347
s( 2π/3, π) = 5π/6 -(3√3)/2 = 0.0199176666

S = s(-π,π) = 5π = 15.70796327

(1) n=15

相変わらず趣旨が分からない問題

366 :132人目の素数さん:2018/05/15(火) 19:02:23.40 ID:Omn+setj.net
一辺の長さが1の正四面体を平面で切ったときに出来る多角形全体からなる集合をSとする。
Sの要素のうち面積が最大である多角形からなる集合をTとする。

(1)Tに属する多角形はすべて合同であることを示せ。

(2)Tの要素の1つをkとする。以下の命題の真偽を判定せよ。
「kの外接円の半径は、Sに属する多角形の外接円の半径のうちで最大である」

367 :132人目の素数さん:2018/05/15(火) 19:10:27.52 ID:j0q7+E4u.net
以下の六つの中で、最も天才と呼ぶのに相応しいのはどれですか?

超絶天才数学者
超絶天才プログラマー
超絶天才画家
超絶天才ピアニスト
超絶天才建築家
超絶天才彫刻家

368 :132人目の素数さん:2018/05/15(火) 19:34:55.72 ID:NusI6DTd.net
不遇のサラリーマン

369 :132人目の素数さん:2018/05/15(火) 19:39:16.43 ID:hVzl3U0R.net
環の問題2つです.
次の証明が合っているか,教えてください.
よろしくお願いします.

[1]R:環とする.
   任意のRの元xについて,x^2=xが成り立つ
   ならば,
   任意のRの元x,yについて,xy=yxが成り立つ.
[証明]
  条件から
   (x+y)^2=x+y
   x^2+xy+yx+y^2=x+y
   xy=−yx.
  また,
   1=1^2=(−1)^2=−1.
  よって,
   xy=yx. □□

[2]R:環とする.
   任意のRの元xについて,x^3=xが成り立つ
   ならば,
   任意のRの元x,yについて,xy=yxが成り立つ.
[証明]
  条件から
   x^2−x=(x^2−x)^3=x^6−3x^5+3x^4−x^3=4(x^2−x)
   3(x^2−x)=0.
   (x^2−x)^2=x^4−2x^3+x^2=2(x^2−x)=−(x^2−x)
   {−(x^2−x)}^2=−(x^2−x)
  Rの元yについて,y^2=yをみたす元の集合をZ(R)とすると,
  Rの部分環になる.
   (x^2)^2=x^2,{−(x^2−x)}^2=−(x^2−x)
   x^2,−(x^2−x)は,Z(R)の元.
   x=x^2−(x^2−x)は,Z(R)の元.
   x^2=x 
   xy=yx. □□

370 :132人目の素数さん:2018/05/15(火) 19:54:52.75 ID:lcPiD9EL.net
>>367
彫刻家or建築家

371 :132人目の素数さん:2018/05/15(火) 20:04:13.05 ID:j0q7+E4u.net
>>370
理由を教えてください。

372 :132人目の素数さん:2018/05/15(火) 20:46:22.89 ID:uUBv6rUz.net
>>369
ええでえ

373 :132人目の素数さん:2018/05/15(火) 20:48:00.59 ID:NusI6DTd.net
>>371
めげずに頑張ってるから

374 :132人目の素数さん:2018/05/15(火) 20:50:01.16 ID:hVzl3U0R.net
>>372 さま
ありがとうございます.
助かりました.

375 :132人目の素数さん:2018/05/15(火) 20:57:40.48 ID:Omn+setj.net
>>366
ガチで渾身の出来だと思っております
宇宙の真理を暴く解答を期待しております

376 :132人目の素数さん:2018/05/15(火) 21:37:59.72 ID:uUBv6rUz.net
>>366
三角形か四角形
三角形の最大は表面の正三角形
四角形は三角形より狭い

377 :132人目の素数さん:2018/05/15(火) 21:59:58.14 ID:klxreP45.net
>>375
ペアノ算術を含む任意の無矛盾な公理系に対し、あるモデルM,Nおよび論理式φが存在して、M|=φかつN|≠φとできることを示せ、という問題がわかりません

378 :132人目の素数さん:2018/05/15(火) 22:00:48.16 ID:LRk6Hmj7.net
>>375
宇宙の真理とは?

379 :132人目の素数さん:2018/05/15(火) 22:26:26.17 ID:In+Nnt+U.net
>>375の宇宙がポリゴンで出来てるって事だろ

380 :132人目の素数さん:2018/05/16(水) 00:38:28.32 ID:hoGpnmIF.net
以下の関数方程式を解け。
f(x+1)+f(x)={1/(f(x+1)-f(x))}

381 :132人目の素数さん:2018/05/16(水) 00:39:15.17 ID:Yf3BPH11.net
しっかし誰も解けない難しい質問ばっかでつまんねえなぁ。
本当に「実際は解いている連中ばっか」状態になったこと一度もねえじゃんw
もっと簡単な質問してこい、脳みそウンコまみれの底辺層ども。

382 :132人目の素数さん:2018/05/16(水) 00:59:34.50 ID:Je9uJcpl.net
>>380
与式⇔f(x+1)^2 = f(x)^2
こんなん死ぬほど解あるやん。

383 :132人目の素数さん:2018/05/16(水) 01:03:33.69 ID:hoGpnmIF.net
>>382
その変形合ってる?

384 :132人目の素数さん:2018/05/16(水) 01:10:27.38 ID:hoGpnmIF.net
しょうがないなあ簡単な質問してやる。
これ解けません

一辺の長さが1の正方形の形をした折り紙がある。1つの角を平面の原点Oに重ね、Oから出る2辺をx軸とy軸の正の部分に重ねる。(1,0)にある角をAとする。
a>0とし、直線y=axに沿って、この折り紙のAを含む側を他方の側に重なるよう折り曲げる。
以下の問に答えよ。

(1)a=1/2,a=2のとき、それぞれ2枚の紙が重なる部分の面積を求めよ。

(2)一般のaに対し、2枚の紙が重なる部分の面積を求めよ。

385 :132人目の素数さん:2018/05/16(水) 01:11:16.97 ID:Je9uJcpl.net
>>369
>Rの元yについて,y^2=yをみたす元の集合をZ(R)とすると,
>  Rの部分環になる.
ここ証明できてない希ガス。和について閉じてる事は要証明じゃね?

386 :132人目の素数さん:2018/05/16(水) 01:11:33.94 ID:iJQo49v2.net
え?

387 :132人目の素数さん:2018/05/16(水) 01:14:44.80 ID:hoGpnmIF.net
こちらはより簡単

a,bを整数とする。
x^4+ax^+bが整数を係数とする2つの既約な多項式の積に因数分解できるとき、a,bが満たす必要十分条件を求めよ。

388 :132人目の素数さん:2018/05/16(水) 01:14:46.03 ID:Je9uJcpl.net
>>383
すまん。正確には
与式⇔f(x+1)^2 = f(x)^2、f(x)≠f(x+1)⇔f(x) = - f(x+1)
相変わらず死ぬほど解ある。

389 :132人目の素数さん:2018/05/16(水) 01:15:46.56 ID:hoGpnmIF.net
>>388
1忘れてない?

390 :132人目の素数さん:2018/05/16(水) 01:17:40.09 ID:hoGpnmIF.net
そろそろこのスレとお別れしたいが、大学受験面白い問題載ってる本を教えてくれ
難問がいい
あと、大数とチャートの難問集は読んで実際もう解いた
理科も理一の合格点取れるしあまり勉強することないんだわ

391 :132人目の素数さん:2018/05/16(水) 01:27:20.72 ID:Je9uJcpl.net
>>389
右辺は
1
―――――
f(x+1)-f(x)
じゃないの?

392 :132人目の素数さん:2018/05/16(水) 01:34:12.23 ID:T2lVwmX3.net
>>390
受験板か受サロで聞け

393 :132人目の素数さん:2018/05/16(水) 01:38:40.60 ID:/MdEF5MU.net
>>380
f(x)^2=g(x) とおけば
g(x+1)-g(x)=1
もし定義域が実数全体であるなら、十分小さい x で g(x)<0 となる。
さらにもし関数 f(x) が実数値であると仮定するなら g(x)=f(x)^2≧0 より矛盾。
定義域をいじったり複素数値を許せば死ぬほどある

394 :132人目の素数さん:2018/05/16(水) 01:44:22.11 ID:iriy4b81.net
>>369

〔参考書〕
数セミ増刊「数学の問題」第(1)集、日本評論社 (1977) No.72
数セミ増刊「数学の問題」第(2)集、日本評論社 (1978) No.60 & 増補
N.Jacobson: "Structure of rings" Amer. Math. Soc. (1964) の 10章、§1

〔一般化〕
任意の x∈R に対して自然数 n(x) >1 があって x^n(x) = x ならば Rは可換。(ベキがxにより異なってもおk)
0以外のベキ零元をもたない有限環は可換。

〔参考文献〕
N. Jacobson: Annals of math., 2nd series, 46(4), p.695-707 (1945/Oct)
 "Structure theory for algebraic algebras of bounded degree"
  http://www.jstor.org/stable/1969205

A. Forsythe & N. H. McCoy: Bull. Amer. Math. Soc., 52(6), p.523-526 (1946)
 "On the commutativity of certain rings"
 http://pdfs.semanticscholar.org/4eb0/7131867b9883b7aebd93a5ed74db74824a14.pdf

395 :132人目の素数さん:2018/05/16(水) 01:45:11.84 ID:Je9uJcpl.net
>>389
ああ、1忘れてた。orz

396 :132人目の素数さん:2018/05/16(水) 02:00:21.58 ID:Je9uJcpl.net
非可換環論はマイナーやからねぇ。

397 :132人目の素数さん:2018/05/16(水) 02:00:54.87 ID:iriy4b81.net
>>380

x = [x] + {x},

f(x) = √([x] + h({x})),

h(x) = f(x)^2 (0≦x<1)

398 :132人目の素数さん:2018/05/16(水) 02:06:55.33 ID:Je9uJcpl.net
>>397
なにそれ?

399 :132人目の素数さん:2018/05/16(水) 03:22:17.35 ID:iriy4b81.net
>>398

(゚Д゚)ハァ?

トリビアの種
http://www.youtube.com/watch?v=8dD0Phsiz0w

400 :132人目の素数さん:2018/05/16(水) 03:39:08.52 ID:Je9uJcpl.net
>>366
>Sに属する多角形の外接円の半径のうちで最大である

って外接円持たないSの要素はどうするん?

401 :132人目の素数さん:2018/05/16(水) 04:33:38.95 ID:Bv6MQJ5v.net
東大の入試問題作りました
良問だと思います

第一問
一辺の長さが10の正四面体ABCDがある。
辺AB上にAP=kとなる点Pを、辺AC上にAQ=mとなる点Qをとる。ただしk,mは10より小さい正整数である。
このとき、△PQDの面積が整数となる(k,m)の組が存在するか、結論と理由を述べよ。

402 :132人目の素数さん:2018/05/16(水) 04:41:27.32 ID:Bv6MQJ5v.net
東大の入試問題作りました
良問だと思います

第ニ問
pを素数、m,nを1≦m≦p-1、1≦n≦p-1を満たす正整数とする。
このとき、二項係数の比
(pCm)/(pCn)……(A)
を既約分数の形で表わせ。二項係数を用いて表してよい。

403 :132人目の素数さん:2018/05/16(水) 04:53:42.08 ID:Bv6MQJ5v.net
東大の入試問題作りました
良問だと思います

第三問
x軸およびy軸の正の部分にそれぞれ点P,Qがあり、PQ=1を満たすように動く。
座標平面の原点をOとし、△OPQの内接円をCpqとする。また、Cpqの周および内部の領域をDpqとする。

(1)点Pが(1,0)に限りなく近づくとき、Cpqの中心はどのような点に限りなく近づくか。同様に、Pが(0,0)に限りなく近づく場合はどうか。

(2)PQが動くとき、座標平面上でDpqに含まれうる領域の面積を求めよ。
ただしPが(0,0)および(1,0)に一致する場合は、(1)で求めた点をDpqとせよ。

404 :132人目の素数さん:2018/05/16(水) 05:01:23.77 ID:iriy4b81.net
>>369 [2]

以下、第(2)集 No.60 からのコピペ  >>394

 xx・xx = x・x^3 = x・x つまり xx はベキ等。

〔補題〕 xxyy = yyxx,  ……(6)

xx=X,yy=Y はベキ等だから、
 X -Y = (X-Y)^3 = X^3 -Y^3 -XYX +YXY = X -Y -XYX +YXY,
 XYX = YXY,
 XY = (XY)^3 = (XYX)(YXY) = (YXY)(XYX) = (YX)^3 = YX,

 (xy)^2 = xx(xy)^2 = (xy)^2・xx,
 xy = (xy)^3 = (xy)^3・xx = (xy)xx … (7)

 (xy)^2 = (xy)^2・yy = yy(xy)^2 
 xy = (xy)^3 = yy(xy)^3 = yy(xy) …… (8)
xとyを入れ替えて
 yx = xx(yx) …… (9)
(8)*(7)
 (xy)^2 = yy(xy)・(xy)xx = y(yx)^3・x = y(yx)x = yy・xx = xx・yy,
xとyを入れ替えて
 (xy)^2 = (yx)^2,
 xy = (xy)^3 = (xy)(yx)^2 = x(yyxy)x = x(xy)x = yx …… (10)

405 :132人目の素数さん:2018/05/16(水) 05:45:28.63 ID:qlzfKH7q.net
>>362 >>364

Γ(s)の定義式を積分路変更することで容易に導けます
(Hankel の公式からでも導出可能ですが、リーマン面で考えないといけないので少し面倒になります)

定義式:Γ(1-s)=∫[0,∞] z^(-s) e^(-z) dz において積分路を実軸から虚軸に変更する
(厳密には半径rとRの1/4円弧と実軸と虚軸上の線分を結ぶ閉曲線を考えr→0,R→∞とする)と
Γ(1-s)=∫[0,∞] (it)^(-s) e^(-it) d(it)
=∫[0,∞] i^(1-s) t^(-s) (cos t -isin t) dt
となって、この式の両辺に i^(-1+s)=e^(πi(-1+s)/2)を乗じれば直ちに(1),(2)式が得られます
また(1)式が0<s<2でも成り立つことはs平面での解析接続より明らか

406 :132人目の素数さん:2018/05/16(水) 06:19:18.60 ID:/WNgShhb.net
>>402
>を既約分数の形で表わせ。二項係数を用いて表してよい。
なにこれ?

407 :132人目の素数さん:2018/05/16(水) 06:24:09.12 ID:Muwlg8wF.net
>>405
おお、すばらしい!thx!

408 :132人目の素数さん:2018/05/16(水) 06:54:49.93 ID:iriy4b81.net
>>384
しょうがないから軽く解くか…

 y=ax (これは単位円の直径)に関してAと対称な点をA'とする。
 A' = ((1-aa)/(1+aa),2a/(1+aa))も単位円周上にある。
 0<a<1 では Aを含む側 ⊂ 反対側
 a>1 では Aを含む側 ⊃ 反対側
よって
 (1/2)min{a,1/a}

409 :132人目の素数さん:2018/05/16(水) 07:45:09.56 ID:hMthHlx8.net
じゃ>>401
面積は1/4√(4(k^2-10k+100)(m^2-10m+100)-(km-10k-10m+200)^2)。
√のなかはmod 5で3k^2m^2に合同。よって平方数になるにはどっちか5。
k=5としてよい。
面積は5/4√(15m^2-20m+700)。
さらにmは5でないとだめ。
でも√のなかは24375。
奇数なのでアウト。

410 :132人目の素数さん:2018/05/16(水) 07:53:54.90 ID:NcyWMFku.net
>>404
x^4=xだとかに一般化は出来るの?

411 :132人目の素数さん:2018/05/16(水) 07:57:56.87 ID:lbcFBVcw.net
>>403
(1)略
(2)内接円を固定して斜辺をうごかすときlog(cot(x))が下に凸よりOP・OQが最小となるのはOP=OQのとき。
よって条件内で内接円の半径が最大となるのはOP=OQ=1のとき。以下ry

412 :132人目の素数さん:2018/05/16(水) 07:58:58.32 ID:lbcFBVcw.net
>>402は意味が……???

413 :132人目の素数さん:2018/05/16(水) 08:19:58.40 ID:mSOZV66q.net
https://light.dotup.org/uploda/light.dotup.org526231.png.html
解法1,2でどこがどう違うのか解説頼む

414 :132人目の素数さん:2018/05/16(水) 08:23:39.86 ID:ej8w+lSP.net
わかりません。

415 :132人目の素数さん:2018/05/16(水) 08:34:10.43 ID:Yf3BPH11.net
>>413
•dotupはリンク先からさらにリンクに飛ばなければならないのでなるべく使わないようにしましょう
•パソコンで数式書く暇があるなら自分で調べましょう

416 :132人目の素数さん:2018/05/16(水) 08:41:37.04 ID:IdiwIaZW.net
1日 a リットルずつ供給される水道で1日 b リットルまで到達させている
b リットルまで x 倍の時間で到達させたい場合、時間あたりの供給量を y 倍すれば良い

xとyの関係を式で表すとどうなりますか?

417 :132人目の素数さん:2018/05/16(水) 08:49:02.25 ID:mSOZV66q.net
>>415
タイトル100万回見直せば如何?

418 :132人目の素数さん:2018/05/16(水) 08:54:33.64 ID:Yf3BPH11.net
>>417
へー、てことは、わからないんですね(笑)

419 :132人目の素数さん:2018/05/16(水) 09:16:07.30 ID:c+HnAvYR.net
xについての不定積分にθが現れているという点で解法2の答え方は不十分
x=tanθとおいたのだからcosθ=(1+x^2)^(-1/2)を用いて変形すれば解法1の答えと一致する

420 :132人目の素数さん:2018/05/16(水) 10:54:39.34 ID:VoYLmZd9.net
東大だからっていわゆる難問にしてないだろうな?

421 :イナ :2018/05/16(水) 10:55:23.89 ID:njbpuZLY.net
/_/_/人人_/_/_/_
/_/_(_^_)/_/_/_
/_/_(_)_)/_/_/_
/_/_( (`_)/_/_/_
/_/_(_っ-┓_/_/_
/_/_◎゙┻υ◎゙/_/_/_/_/_/_/_/_/_/_/_/_/キコキコ……。>>350分母を有利化すると見た目が違ってきます。髪の毛が風でうしろになびくと頭頂部の見た目が変わるでしょ。

422 :132人目の素数さん:2018/05/16(水) 11:21:08.44 ID:hpTFF7qj.net
画像中央の行にある式の波線部分がどのような指数計算をして出したのか途中式を交えて教えてくだされば幸いです
二項定理そのものはわかりますがこの指数計算がわからず質問しました
よろしくお願いします
https://i.imgur.com/y5krm7F.jpg

423 :132人目の素数さん:2018/05/16(水) 12:19:58.64 ID:ej8w+lSP.net
眠い

424 :132人目の素数さん:2018/05/16(水) 13:11:47.77 ID:phdn5f0F.net
422
(x^2)^k (2/x)^(10-k)=x^(2k) 2^(10-k) x^(-(10-k))
=x^(2k) 2^(10-k) x^(-10+k)=2^(10-k) x^(2k) x^(-10+k)
=2^(10-k) x^(2k-10+k)=2^(10-k) x^(3k-10)

425 :132人目の素数さん:2018/05/16(水) 13:31:06.76 ID:T2lVwmX3.net
>>413>>415
リンクの貼り方が悪い
https://light.dotup.org/uploda/light.dotup.org526231.png
と .html を消せばちゃんと見れる

426 :413:2018/05/16(水) 15:11:17.93 ID:mSOZV66q.net
>>419
なるほど
そういう事か
3Q

427 :132人目の素数さん:2018/05/16(水) 15:22:50.26 ID:hpTFF7qj.net
>>424
ありがとうございます

428 :132人目の素数さん:2018/05/16(水) 16:49:16.19 ID:iriy4b81.net
>>410

指数nが偶数のとき
 (-x)(-x) = xx,
 の両辺を n/2 乗して
 -x = (-x)^n = x^n = x,
 また
 x + xx + … + x^(n-2) ∈ Z(R)

n=2 のとき(ベキ等環)
 xy + yx = (x+y)^2 -x^2 -y^2 = (x+y) -x -y = 0
 ゆえ反可換、可換。

n=4 のとき
 x + xx ∈ Z(R) より
 x(xy+yx) = (xy+yx)x,
 xxy = yxx,
 ここで x→xx とすれば
 xy = x^4・y = y・x^4 = yx,

n=6 のとき
 n=2 に帰着する。

n=4〜8は証明できたらしい。(淡中忠郎教授)

429 :132人目の素数さん:2018/05/16(水) 17:08:33.77 ID:GzTDB8cM.net
まぁだからどやねんというツッコミが絶えないんだよなぁ、>非可換環ろん

430 :132人目の素数さん:2018/05/16(水) 18:54:03.69 ID:X5WFdbcF.net
東大の入試問題作りました
良問だと思います

第四問
iは虚数単位、p,qは実数とする。
数列{a_n}を以下のように定める。
a_1=1、a_2=i、
a_(n+2)=pa_(n+1)+qa_n

(1)すべてのnに対し|a_n|=1となるために、p,qが満たすべき必要十分条件を求めよ。

(2)すべての点Pn(a_n)が同一円周上にあるために、p,qが満たすべき必要十分条件を求めよ。

431 :132人目の素数さん:2018/05/16(水) 19:25:30.56 ID:Kj+fSsL+.net
急にレベル下がった感が……

432 :132人目の素数さん:2018/05/16(水) 19:39:58.76 ID:X5WFdbcF.net
東大の入試問題作りました
良問だと思います

第五問
空間の2点(0,0,0)と(1,0,0)を直径の両端とする円をC1、2点(0,0,1)と(1,0,1)を直径の両端であとする円をC2とする。
C1とC2を底面とする円柱を曲面z=x^2によって2つの領域に切り分けるとき、領域の体積比V1:V2を求めよ。

433 :132人目の素数さん:2018/05/16(水) 19:40:36.33 ID:i0JFdzw4.net
東大の入試問題を作りました
良問だと思います

ある無矛盾な公理系τの任意のモデルに対してある論理式φが常に真となるならば、τからφがLKにおいて証明可能となることを示せ

434 :132人目の素数さん:2018/05/16(水) 19:49:29.84 ID:X5WFdbcF.net
東大の入試問題作りました
良問だと思います

第六問
a,bは正の実数とし、xの関数f(x)をf(x)=ax-b+e^(-x)と定める。相異なる実数p,qに対して、f(p)とf(q)の大小を比較せよ。

435 :132人目の素数さん:2018/05/16(水) 20:19:59.92 ID:0FC1yzkn.net
スレタイも読めないんじゃね

436 :132人目の素数さん:2018/05/16(水) 20:27:54.26 ID:+HHSIr+p.net
東大の入試問題を作りました
良問だと思います

なぜ劣等感婆は数学板、物理板を荒らすのか?
400字以内で回答せよ。キイワードは必須とする。

キイワード
素人、劣等感、自演

437 :132人目の素数さん:2018/05/16(水) 21:08:19.70 ID:MPlRBdj8.net
>>413
結論はどちらも正しい。

2つめはθをxに直せば、θ→arctan xであるが
cos (arctan x)を調べたら、(1+x^2)の(-1/2)乗と等しいので
上と一致する

438 :132人目の素数さん:2018/05/16(水) 21:17:32.79 ID:zoVvkzUT.net
ここの回答者って、簡単な問題だとすでに回答がついていても同じ回答つけるんですな

439 :132人目の素数さん:2018/05/16(水) 21:21:54.48 ID:IWAY7ub/.net
まぁ、第4問以降は急に普通の受験数学のレベルだからなぁ。

440 :132人目の素数さん:2018/05/16(水) 21:25:12.26 ID:+HHSIr+p.net
東大の入試問題を作りました
良問だと思います

なぜ劣等感婆は基礎論を勉強したのに予備校をくびになったのか?
400字以内で回答せよ。

441 :132人目の素数さん:2018/05/16(水) 21:32:08.66 ID:GzTDB8cM.net
東大入試作ってるのんが劣等感婆?

442 :270:2018/05/16(水) 23:00:10.70 ID:1o6QYqoa.net
>>270は無理でしょうか……

443 :132人目の素数さん:2018/05/17(木) 02:04:16.25 ID:3t2VcDet.net
工夫すればエレガントに解けるそうですが
教えてください。

立方体ABCD-EFGHと1~10までのカードが1枚ずつある。
このカードの中から8枚選び、各頂点に1枚ずつ割り当てる。このとき、
”4つの頂点のカードの数字の和が偶数となる”ような面がちょうど3つ存在する確率を求めよ。

444 :132人目の素数さん:2018/05/17(木) 02:27:49.32 ID:1BF/qMvl.net
>>439
第1問は面積を表すまでは簡単ですがその先で余りに着目できないと詰みです
C***

第2問はpCiが全てpで割り切れることを知らないと手がつけられません。その後のpCkとpCmの処理も難しく、6問中の最難問です
D#

第3問は円板の通過領域の把握がやや難しいです。面積計算は平易です
C***

第4問は6問中最も易しいです
B**

第5問も易しいですが、円柱をタテに切らないと計算が面倒になります
B**

第6問は意外に難しいです。場合分けや論証に手こずると思います
C****

445 :132人目の素数さん:2018/05/17(木) 02:34:09.45 ID:1BF/qMvl.net
>>439
第4問と第5問は完答必須。ここで20*2の40点は確保したい

第1問と第3問はあわせて1題分以上の点数を取りたい。20点程度を目標

第6問は手数がかかる上に緻密さも必要で、後回しにしたい
第2問は出来るところまでで残りは捨てたほうがいい
合計で1題ブン取れれば上出来、20点弱が目標

合格者平均
理一65、理ニ55、理三80

446 :132人目の素数さん:2018/05/17(木) 02:35:19.73 ID:1BF/qMvl.net
明日も東大の入試問題を上げます。
よろしくお願い致します。

447 :132人目の素数さん:2018/05/17(木) 02:39:01.75 ID:rInW77ym.net
>>443
エレガントかどうか知らんけど
奇数のカード数≡奇数の面数 (mod 2)
より奇数のカード数は奇数。
このとき常に奇数の面数は3。
∵奇数カード数が1なら自明(無視していいケースだが)
奇数カード数が3、隣接頂点に奇数カードが配置されるときは残り1枚の奇数カードをどこにおいても(実質2ケースしかない)奇数の面数は3。
奇数カード数が3、隣接頂点に奇数カードが配置されないときはどの2頂点も対角に配置できないから正三角形の頂点をなすように配置するしかなく、奇数の面数は3。
奇数カード数が5のときは奇数カードと偶数カードの配置を総入れ替えすれば奇数カード3のケースに帰着される。
以上により奇数カード数が奇数となることが条件。
確率は2×C[5,3]×C[5,5]/C[10,8]。

448 :132人目の素数さん:2018/05/17(木) 02:43:36.84 ID:rInW77ym.net
>>446
いつまで受験数学レベルやってんの?もう卒業したら?
もっと素晴らしい世界がひらけてるのに。
そのくらいの問題作れるなら次のステップに進む素地は整ってるんだから。

449 :132人目の素数さん:2018/05/17(木) 02:50:56.14 ID:1BF/qMvl.net
>>448
大学受験で大学数学使えないんで
それに実数とか極限とか当たり前のことを精密に議論するのって何が楽しいんですか
線形代数やっとけって先輩から言われましたが使いみちが分かりません。空間の点を回転したり対称移動できるとかつまらない

大学行ってから勉強します
今は遊びます

450 :132人目の素数さん:2018/05/17(木) 03:04:52.90 ID:FDCSht5h.net
>>270

b=b0 を固定する。
 
log(y) = log(a) + c・log|sinh(bx)|
より
 X = log(sin(bx)),Y = log(y)
とし、(X,Y)データを最小二乗法で直線回帰する。
 Y = log(a) + c・X
ただし、(a,c) は b0 に依存する。

次に、
Z = sinh^(-1){(y/a)^(1/c)}
とし、(x,Z)データを最小二乗法で直線回帰する。
Z = b・x + d,
ただし、(b,d)は(a,c)に依存する。

これを (a,b,c,d) が収束するまで繰り返す。

SCF (Self-consistent Field)

451 :132人目の素数さん:2018/05/17(木) 03:05:20.32 ID:FDCSht5h.net
>>270

b=b0 を固定する。
 
log(y) = log(a) + c・log|sinh(bx)|
より
 X = log(sin(bx)),Y = log(y)
とし、(X,Y)データを最小二乗法で直線回帰する。
 Y = log(a) + c・X
ただし、(a,c) は b0 に依存する。

次に、
Z = sinh^(-1){(y/a)^(1/c)}
とし、(x,Z)データを最小二乗法で直線回帰する。
Z = b・x + d,
ただし、(b,d)は(a,c)に依存する。

これを (a,b,c,d) が収束するまで繰り返す。

SCF (Self-consistent Field)

452 :132人目の素数さん:2018/05/17(木) 03:23:01.81 ID:FDCSht5h.net
>>403

 P(p,0) Q(0,q) p>0,q>0 とする。
 題意より pp+qq=1
 Cpq の中心 (r,r) ここに r = (p+q-√(pp+qq))/2 = (p+q-1)/2,
(1) p→1 のとき q→0,r→0
  p→0 のとき q→1,r→0

(2) は >>411

453 :132人目の素数さん:2018/05/17(木) 03:56:27.34 ID:FDCSht5h.net
>>430

a_{n+2} = p・a_{n+1} + q・a_n,
より
|a_{n+2}|^2 = a_{n+2}・a_{n+2}~
= (p・a_{n+1} + q・a_n) (p・a_{n+1}~ + q・a_n~)
= pp|a_{n+1}|^2 + pq・Re(a_{n+1}・a_n~) + qq|a_n|^2,

(1) pp + qq = 1,pq = 0,
  (p,q) = (0,±1) (±1,0)

454 :132人目の素数さん:2018/05/17(木) 03:57:03.37 ID:EpQu68X7.net
毎日外からうるさい。誹謗しかできない卑怯者は黙ってろ。

455 :132人目の素数さん:2018/05/17(木) 04:54:16.25 ID:FDCSht5h.net
H大R学部K学科だな。

456 :132人目の素数さん:2018/05/17(木) 07:28:57.67 ID:1xjVKUsZ.net
広島大学理学部化学科とかですかね

457 :132人目の素数さん:2018/05/17(木) 09:29:42.08 ID:61Rpx63F.net
>>449
会話できたのか
ならなぜ散々スレ違いだと言われているのを頑なに無視するのか

458 :132人目の素数さん:2018/05/17(木) 10:39:15.71 ID:qdvcO8MT.net
>>449

精密に議論するのは、雑な議論で答えだけ出るのと比べて結局同じ答えしか出てないからつまらなく感じるかもしれないけど、それはより深い世界へと進むための準備で大切な事だよ。
今の日本だと “受験” という人参をぶらさげて他の人が出せない答えがどれだけだせるか?という競争の中で勉強させられるから間違いやすいけど数学は決して “ほら、俺こんな問題もとけるぜ” って得意がるための道具じゃないよ。
高々受験数学レベルの問題のあたりを一生ウロチョロして終わるか、数学が真に “人生の友” と呼べる素敵な “学問” になるかの分かれ目だねぇ。

459 :132人目の素数さん:2018/05/17(木) 12:37:07.69 ID:0BPyzBl6.net
>>443 >>447
ちょい改善。
abcdefghをそれぞれABCD-EFGHに奇数カードのせたとき1,偶数カードのとき0をとる変数とする。
奇数面1、5個がないことを示せば良い。
ABCDのみが奇数面なら
a+b+c+d≡1 (mod2)。
一方他の面は偶数面だから
a+b+e+f≡0 (mod2) c+d+g+h≡0 (mod2) e+f+g+h≡0 (mod2)
の3つ足して
a+b+c+d≡0 (mod2)。
∴矛盾。
奇数面5個がないのは今の議論で右辺の0と1を入れ替えれば良い。

460 :132人目の素数さん:2018/05/17(木) 12:49:27.37 ID:wzusziGa.net
>>458
ある無矛盾な公理系τの任意のモデルに対してある論理式φが常に真となるならば、τからφがLKにおいて証明可能となることを示せ

精密な議論でよろしくお願いしますね
これがわからないということは、論理や証明とは何かが分かっていないことと同義です

461 :132人目の素数さん:2018/05/17(木) 13:11:56.89 ID:t8x6A37/.net
進歩が無い奴

462 :132人目の素数さん:2018/05/17(木) 13:23:27.09 ID:kGwb7h50.net
まぁ何も高いレベルに挑戦し続けることだけが数学ではないからね。
受験数学レベルを楽しむのもよし。数学的なレベルは問題じゃない。
それで “俺様ってすごいだろ” って事にしか数学を使えてないのが問題だねぇ。
基礎論の勉強も結局 “お前らはそこまでやってないだろ” って粋がりたいためにしか使えてない。完全性定理レベルでねぇ。

463 :132人目の素数さん:2018/05/17(木) 14:04:36.68 ID:Iir1USr+.net
>>460
回答がありませんね
何故でしょうか

464 :132人目の素数さん:2018/05/17(木) 14:13:29.35 ID:kGwb7h50.net
>>463
完全性定理が君の人生で読んだ一番難しい理論?そこで終わり?

465 :132人目の素数さん:2018/05/17(木) 14:15:29.34 ID:DyYiDvcL.net
それエーデルワイズの定理だろ、

466 :132人目の素数さん:2018/05/17(木) 14:15:59.53 ID:ItvLqMFX.net
>>463
なぜ基礎論を勉強したのに予備校を首になったのですか?

467 :132人目の素数さん:2018/05/17(木) 14:16:03.22 ID:Iir1USr+.net
>>464
わかるなら答えられるはずですね
答えないということは、わからないということですね

468 :132人目の素数さん:2018/05/17(木) 14:31:05.87 ID:bK8jl4eF.net
>>467
なので私はあなたより頭がよいと。満足できて良かったねえ。

469 :132人目の素数さん:2018/05/17(木) 14:36:52.12 ID:bK8jl4eF.net
>>465
そうなん?完全性定理じゃないの?基礎論の教科書なんてもう何年も開いてないから忘れてしまったorz

470 :132人目の素数さん:2018/05/17(木) 14:37:17.34 ID:Iir1USr+.net
わからないんですね(笑)

471 :132人目の素数さん:2018/05/17(木) 14:45:00.32 ID:8bFcDFwY.net
>>2

>>460
いつまでみっともないこと晒すかねこの人

472 :132人目の素数さん:2018/05/17(木) 14:59:15.60 ID:zdWSgKfO.net
今度の劣等感婆は無差別に噛み付く芸風に変えたのか

473 :132人目の素数さん:2018/05/17(木) 15:05:24.94 ID:bK8jl4eF.net
諭されて改める=負け
なんだろうなぁ。一生懸命に数学勉強して獲得したものがこれだけなのはちょっとかわいそうではある。
今の “他人を打ち負かしてナンボ” という受験至上主義のかわいそうな犠牲者ではある。

474 :132人目の素数さん:2018/05/17(木) 15:28:55.68 ID:Iir1USr+.net
わからない人が何か言ってますね

475 :132人目の素数さん:2018/05/17(木) 15:35:00.43 ID:0y+f26PG.net
馬鹿にされたの?

476 :132人目の素数さん:2018/05/17(木) 15:38:11.25 ID:RMm2CIYv.net
「ある無矛盾な公理系τの任意のモデルに対してある論理式φが常に真となるならば、τからφがLKにおいて証明可能である」という命題が証明可能であることを示せ

という問題が分かりません。教えてください

477 :132人目の素数さん:2018/05/17(木) 15:49:13.11 ID:0y+f26PG.net
ここでこの答えを書く人は居ません
分からない人は書けませんし、分かる人は書きません
なぜならあなたの文章は人を試しているように見えるからです(あなたの本心は関係ありません あなたの文章を他人がどうとらえるかが大事です)
あなたに認められるより問題が解けた事実の方がはるかに価値があるのです

478 :132人目の素数さん:2018/05/17(木) 15:55:17.88 ID:Iir1USr+.net
私はリーマン予想が解くことができましたが、ここには書きません
書けたという事実が大事だからです
フィールズ賞は論文提出しなくてももらえるんですかね

479 :132人目の素数さん:2018/05/17(木) 16:20:03.46 ID:PIxyPgar.net
余白が狭すぎる

480 :132人目の素数さん:2018/05/17(木) 16:28:26.70 ID:bK8jl4eF.net
>>478
もらえるに決まってるやん。待っとき。

481 :132人目の素数さん:2018/05/17(木) 16:29:39.43 ID:1BF/qMvl.net
一辺の長さが1の正五角形ABCDEと、正方形CDFGがある。ただし2つの図形は直線CDについて同じ側にある。
点Fはこの正五角形の内部、周上、外部のいずれに位置するか。

482 :132人目の素数さん:2018/05/17(木) 16:36:29.67 ID:Gniau8fI.net
自演が既知外レベル

483 :イナ :2018/05/17(木) 18:20:49.21 ID:l1vlYSML.net
>>481同じ側なんだから題意から五角形の中じゃん? てか、かっこいいね、
_
/\龍雄って名前。
\_/ ̄ ̄ ̄ ̄ ̄ ̄/\
 ̄| ̄∩∩ ∩∩ ̄\/|
_|((-_-)-_-)) / |
 ̄|`(っu~)U⌒U、/| |
]| ‖υυ~UU~‖ |/ \
_| ‖ □ □ ‖ /  /
 ̄\‖____‖/  /|
_________/||
 ̄ ̄ ̄ ̄ ̄ ̄ ̄ ̄‖| |/
] □  □`;,□ ‖| /
______;__‖|/_
________‖//_/_/_/_/_/_/_/_/_/_/_/_/_/_/_/_/_/_

484 :132人目の素数さん:2018/05/17(木) 18:53:54.51 ID:Kc1v/rUO.net
尋常じゃないくらい頭が悪いけど、東京大学理学部数学科に入りたい。

485 :132人目の素数さん:2018/05/17(木) 20:30:19.59 ID:374sM+up.net
>>477
>あなたに認められるより問題が解けた事実の方がはるかに価値があるのです
うむ

486 :132人目の素数さん:2018/05/17(木) 21:03:43.22 ID:bK8jl4eF.net
まぁ受験勉強というのが究極の功利主義やからなぁ。

487 :132人目の素数さん:2018/05/17(木) 21:08:32.20 ID:0QGsx+P0.net
√9= √(-3)^2 = -3
この解き方がダメな理由を妹に説明したいです
√9はプラスの数だよと言っても納得してもらえず困っています…力を貸してください…

488 :132人目の素数さん:2018/05/17(木) 21:24:45.18 ID:DyYiDvcL.net
間違いではない。

489 :132人目の素数さん:2018/05/17(木) 22:23:16.76 ID:nU2AsPvn.net
まぁ。本来は絶対値付けるからな

490 :132人目の素数さん:2018/05/17(木) 22:27:02.15 ID:nU2AsPvn.net
まぁ。平方根と√の違いを教えてやれ

491 :132人目の素数さん:2018/05/17(木) 23:13:58.68 ID:374sM+up.net
>>487
正しいだろって言ってやれよ
3=√3^2=√9=√(-3)^2=-3
これでいいだろって

492 :132人目の素数さん:2018/05/17(木) 23:19:30.80 ID:8TelVl60.net
↓この証明なんかおかしくない?
a,a'∈A
b,b'∈B
fは全射ゆえV(f)=B
よって∃[a∈A](b∈f(a))⇔b∈B
b∈f(a)⇔a∈f^-1(b)より
∃[a∈A](a∈f^-1(b))⇔b∈B
よってD(f ^-1)=B-@
fは写像ゆえ(b∈f(a)∧b'∈f(a))⇒b= b'
集合の相当の定義より(b∈f(a)⇔b∈f(a')) ⇒f(a)=f(a')
よって(b∈f(a)∧b∈f(a'))⇒f(a)=f(a')
fは単射ゆえf(a)=f(a')⇒a=a'
よって(b∈f(a)∧b∈f(a'))⇒a=a'
b∈f(a)⇔a∈f^-1(b)より
(a∈f^-1(b) ∧a'∈f^-1(b))⇒a=a'-A
@、Aより全単射の写像fの逆対応f ^-1は写像


特にこの部分とかめっちゃ飛躍してるよね
>fは写像ゆえ(b∈f(a)∧b'∈f(a))⇒b= b'
>集合の相当の定義より(b∈f(a)⇔b∈f(a')) ⇒f(a)=f(a')
>よって(b∈f(a)∧b∈f(a'))⇒f(a)=f(a')
>fは単射ゆえf(a)=f(a')⇒a=a'
>よって(b∈f(a)∧b∈f(a'))⇒a=a'

493 :132人目の素数さん:2018/05/17(木) 23:20:28.80 ID:5tSjyrNL.net
>>487
あなたの √ の説明が下手なだけ、或いは嘘を教えているのかも。

494 :132人目の素数さん:2018/05/17(木) 23:25:27.27 ID:5tSjyrNL.net
>>492
まず、4行目の  b∈f(a)  なんて記述が写像を理解していない証。
どう書かなければいけないかが分かったところで、もう一度質問を書き込んでみてね。

4行目で読む気が失せたので。

495 :132人目の素数さん:2018/05/17(木) 23:28:56.03 ID:girYMyxF.net
>>494
え?f(a)=bという表記じゃないと認めないってこと?本来はf(a)={b}なんだから別にいいと思うけど

496 :132人目の素数さん:2018/05/17(木) 23:30:01.76 ID:5tSjyrNL.net
>>495
違うよ。

497 :132人目の素数さん:2018/05/17(木) 23:33:00.23 ID:girYMyxF.net
>>496
何が違うの?対応って知ってる?

498 :132人目の素数さん:2018/05/17(木) 23:34:17.09 ID:5tSjyrNL.net
>>497
きみは b={b} と思っているわけね

499 :132人目の素数さん:2018/05/17(木) 23:42:52.53 ID:girYMyxF.net
>>498
対応による像が部分集合である以上定義域が始集合と等しく各像が1つの元から成る対応を写像としているわけだから写像による像も部分集合と認めざるを得ないよね
f(a)=bは慣習的表記に過ぎない

500 :132人目の素数さん:2018/05/17(木) 23:47:32.78 ID:5tSjyrNL.net
>>499
ナンセンス


きみがおかしな証明といっているその証明の対象である命題は、多分こういうことなのだろうと思う。

集合Aから集合Bへの全単射fが存在するとき、逆像対応 f^(-1);B→2^A は f の逆写像を定義していることを示せ。
ここに fの逆像対応f^(-1)とはb∈Bに対してf^(-1)(b)={a∈A|f(a)=b}で定まる対応である。

501 :132人目の素数さん:2018/05/17(木) 23:50:26.44 ID:girYMyxF.net
>>500
いや、俺は>>492じゃないよ

502 :132人目の素数さん:2018/05/17(木) 23:52:47.80 ID:5tSjyrNL.net
>>501
では、>>495以降のあなたの言説は妄言。

503 :132人目の素数さん:2018/05/17(木) 23:55:50.59 ID:Mrc1Smh5.net
一般的な(教科書等に書いてある)記述はf(a)=bまたはf({a})={b}なので、それに合わせるのは普通だと思うが

504 :132人目の素数さん:2018/05/17(木) 23:56:30.56 ID:5tSjyrNL.net
あらためて >>492 さん、 質問を書き直してください。

505 :132人目の素数さん:2018/05/18(金) 00:00:14.72 ID:AZta2T+Z.net
>>502
妄言ってアンタ…写像は対応の一種でしょうが

506 :132人目の素数さん:2018/05/18(金) 00:02:29.20 ID:phLq/mSI.net
>>503
写像 f(a)=b に対して f({a})={b} と書いてある教科書をご教示下さい。

写像f が集合間写像である汎写像を定義する、という類の説明は別の問題、ということでよろしく。。

507 :132人目の素数さん:2018/05/18(金) 00:02:53.55 ID:AZta2T+Z.net
まぁ別にこだわるところでもないからいいけどさ、もう俺は寝る

508 :132人目の素数さん:2018/05/18(金) 00:03:11.30 ID:phLq/mSI.net
>>505
対応の階梯をごっちゃにしてる?

509 :132人目の素数さん:2018/05/18(金) 00:09:00.29 ID:K2Dl9lJX.net
つーか>>492の証明は何かの教科書の証明の引き写し?
なんつー教科書?

510 :132人目の素数さん:2018/05/18(金) 00:12:59.11 ID:phLq/mSI.net
うむ。それが合理的な反問。

511 :132人目の素数さん:2018/05/18(金) 00:14:12.78 ID:m1kJBbKU.net
>>495
>え?f(a)=bという表記じゃないと認めないってこと?
認めないんじゃないの?

512 :132人目の素数さん:2018/05/18(金) 00:15:59.45 ID:m1kJBbKU.net
>>499
f({a})とf(a)とを近藤

513 :132人目の素数さん:2018/05/18(金) 00:16:30.09 ID:IChGANrB.net
√ は二課関数だから、どっちでもいい。

514 :132人目の素数さん:2018/05/18(金) 00:18:11.29 ID:6J8mIcZT.net
>>495ってこういうこと?
https://ja.m.wikipedia.org/wiki/対応_(数学)
定義
対応 f = (A, B, Gf) は、
「各元 a ∈ A に対して (a, b) ∈ Gf となるような b ∈ B が一つしかない(すなわち、A のどの元 a についても f(a) がただ一つの元からなる)」
という条件をみたすとき、部分写像(一意対応)という。特に D(f) = A(全域的)なとき写像と呼ばれる。
対応 f が(部分)写像であるとき、f(a) = {b} となることを f(a) = b と略記して、この元 b を a の像と呼ぶ。

515 :132人目の素数さん:2018/05/18(金) 00:23:03.44 ID:+W5C6ZEg.net
>>506
誤解を避けるために言っておくと、f({a})={b}という書き方をしても構わないという話な
このような表記が許されるということが分かる例という話なら、例えば集合位相入門(松坂)p.30だが

516 :132人目の素数さん:2018/05/18(金) 00:27:22.85 ID:K2Dl9lJX.net
おお、ほんと。wikipediaはそういう書き方なのか。ビックリ。
でもこれは一般的な記法ではないに一票。
この書き方したらいかんとは言わないけど使うなら一言但し書きいれないとだめじゃね。
てかwikipediaこんな特殊な書き方するなら一言入れないとダメな気がする。
もしかして俺のほうが少数派なのかもしれんけど。

517 :132人目の素数さん:2018/05/18(金) 00:29:48.54 ID:K2Dl9lJX.net
>>515
wikipediaのリファレンスも松阪先生の教科書だね。
この書き方のほうがメジャーなん?
初めて見る……

518 :132人目の素数さん:2018/05/18(金) 00:57:09.46 ID:7GRL3rM4.net
f(集合)=集合

こんなの像の定義そのものですよね
どれだけレベルが低いんですか、このスレッドは

519 :132人目の素数さん:2018/05/18(金) 04:05:37.85 ID:JUWy+p9g.net
天国と数学はどっちの方が凄いのでしょうか?

520 :132人目の素数さん:2018/05/18(金) 04:19:42.13 ID:539vwTx6.net
>>481

△DEFは2等辺△だから
∠DEF < 90゚ < 108゚ = ∠AED
∴ 点Fは∠AED の内部にある。 … (1)

CDの中点をMとすると、対称性より、
∴ 点F は∠AMD の内部にある。 … (2)

(1)(2)より、点F ⊂ ◇AMDE ⊂ ABCDE

521 :132人目の素数さん:2018/05/18(金) 05:20:32.37 ID:fyKum6I9.net
(1)nは2以上の正整数とする。n!は平方数にならないことを示せ。

(2)kは2以上の正整数とする。k個の連続した正整数の積は平方数にならないことを示せ。

522 :132人目の素数さん:2018/05/18(金) 05:49:12.03 ID:/qlvVszp.net
>>519
真面目に数学の勉強するのは地獄だって自覚表明みたいなこといつまで続けるつもりなの?

523 :132人目の素数さん:2018/05/18(金) 05:51:31.10 ID:fyKum6I9.net
p,qは整数とする。
a_1=1、a_2=2、a_(n+2)=pa_(n+1)+qa_nである数列{a_n}について、以下の問に答えよ。

(1)任意のiに対しa_(i+1)>a_iとなるために、p,qが満たすべき必要十分条件を求めよ。

(2)p,qは(1)の条件を満たすとする。さらに{a_n}が以下の条件[C]を満たすために、p,qが満たすべき必要十分条件を求めよ。
[C]どのような3以上の整数jに対しても、1≦mj<kj≦3である正整数mj,kjが存在して、{a_(j+kj)-a_(j+mj)}/(kj-mj)がa_j,a_(j+1),a_(j+2),a_(j+3)のいずれかに等しくなるようにできる。

524 :132人目の素数さん:2018/05/18(金) 06:20:27.53 ID:CvPbHZEO.net
>対応 f が(部分)写像であるとき、f(a) = {b} となることを f(a) = b と略記して、この元 b を a の像と呼ぶ。

この定義の仕方は初めて見たな。普通は f(a)=∪{b} (右辺は和集合の公理から定まる集合)
と定義しないか?これなら文字通り f(a)=b だよ。

525 :132人目の素数さん:2018/05/18(金) 07:13:56.88 ID:7GRL3rM4.net
↑これが数学板の実力です
専門板なのに異常にレベルが低い
せいぜい数学の少しできる高校生レベル


本当にレベル低すぎませんか?
この程度なんですか、数学板って

526 :132人目の素数さん:2018/05/18(金) 09:48:40.35 ID:CHfLk6wX.net
https://i.imgur.com/qGsNHA5.jpg
これの4つ目、ご教授お願いします

527 :132人目の素数さん:2018/05/18(金) 09:57:55.27 ID:tBeUSVjx.net
1/z

528 :132人目の素数さん:2018/05/18(金) 11:47:13.48 ID:dH1Oea0s.net
あざます
≠0の時点で察するべきだったか…

529 :132人目の素数さん:2018/05/18(金) 15:25:53.82 ID:fyKum6I9.net
2次方程式の実数解は座標平面上だとx軸と放物線の交点に対応します
そこで、虚数解を座標平面の何かに対応させることはできるんでしょうか?
虚数とはいえ、その値は放物線のパラメーターに依存するので、放物線や座標平面との図形的対応ができないかと考えています

530 :132人目の素数さん:2018/05/18(金) 16:28:07.52 ID:syEIqgPS.net
問(3)について
画像2枚目の解説にある0<k<b, 0<l<bから、なぜ-b<k-l<bが出てくるのか教えてください
よろしくお願いします
https://i.imgur.com/2CWMRG0.jpg
https://i.imgur.com/nzla0rY.jpg

531 :132人目の素数さん:2018/05/18(金) 16:31:59.91 ID:hI27LzA8.net
あガガイ

532 :132人目の素数さん:2018/05/18(金) 17:47:10.12 ID:syEIqgPS.net
>>530の質問内容の書き忘れです
なぜ0<k<b, 0<l<bという風にkとlの範囲が絞られているのかもお教え頂けますか
連投すみません

533 :132人目の素数さん:2018/05/18(金) 18:09:27.44 ID:wLf2fNpx.net
>>530
0<k<b
-b<-l<0
を辺々足しただけ

>>532
問題を100回読み直せ

534 :132人目の素数さん:2018/05/18(金) 18:17:04.13 ID:539vwTx6.net
>>526

(iv) u(x,y) = x/(xx+yy),
   u(x,0) = 1/x,
 ∴ u(x,y) = u(z,0) = 1/z,

535 :132人目の素数さん:2018/05/18(金) 18:27:13.33 ID:i54mOA7u.net
自作ボードゲーム市場に詳しい「ペンとサイコロ」というブログの
「ゲムマ2017秋・アンケート結果 第二弾:2016→2017年比較」の記事によると

ゲームマーケットに出品した人の半分が赤字で半分が黒字でちょうど半々だそうだ
50万以上の儲けが5%いるが逆に50万以上赤字なのも5%いる
そして初参加の人の7割が赤字なのに対して、ノウハウありや知名度や固定ファン層が居る
中堅サークル7割が黒字になってる
継続性とブランド力構築とノウハウが大事だという事だと思う
初参加の人は作る個数と需要を見極めツイッターやユーチューブでの宣伝がカギになる
最初は50〜100個ぐらいをいかに金かけないで作って売るかの勝負になる

これがゲムマ2016・2017年(初の二日開催)の販売数
https://cdn-ak.f.st-hatena.com/images/fotolife/r/roy/20171220/20171220211924.png
これが販売金額
https://cdn-ak.f.st-hatena.com/images/fotolife/r/roy/20171220/20171220212902.png
これがイベントでの利益
https://cdn-ak.f.st-hatena.com/images/fotolife/r/roy/20171220/20171220213109.png

536 :132人目の素数さん:2018/05/18(金) 20:46:20.13 ID:Q0zCCt8k.net
教えてください

xy平面上で、D={(x,y)| 3≦x≦5, 3≦y≦5}とする。
Dをx軸を中心に回転させた立体をD、Dをさらにy軸を中心に回転させた立体をE
とするとき、Eの体積を求めよ。

537 :132人目の素数さん:2018/05/18(金) 21:03:23.78 ID:wLf2fNpx.net
>>536
名古屋大2011に類題がある

538 :132人目の素数さん:2018/05/18(金) 21:50:21.55 ID:drDf2Sp5.net
1+1/2+1/2+1/3+1/3+1/3+1/4+1/4+1/4+1/4+1/5+1/5+1/5+1/5+1/5+.......の部分和を表す一般的な式はありますか?
発散することを示したいです

539 :132人目の素数さん:2018/05/18(金) 23:19:58.38 ID:uUybNQSy.net
a[n]=(2n+k^2+k)/(2k+2) ただし、k=Floor[(Sqrt[8n+1]-1)/2]

540 :132人目の素数さん:2018/05/18(金) 23:41:23.02 ID:m1kJBbKU.net
>>516
>おお、ほんと。wikipediaはそういう書き方なのか。ビックリ。
ウィキペディアが嘘

541 :132人目の素数さん:2018/05/18(金) 23:42:30.96 ID:m1kJBbKU.net
>>518
だから
f(集合)=集合
f(元)=元

542 :132人目の素数さん:2018/05/18(金) 23:46:55.55 ID:TmksB6AE.net
>>538
1/nが発散することの証明か?

543 :132人目の素数さん:2018/05/18(金) 23:54:22.76 ID:TmksB6AE.net
>>518
横からだけど、f(集合)=元はおかしいやろって話なんじぇね?

544 :132人目の素数さん:2018/05/19(土) 00:15:09.51 ID:oHNHehfx.net
515 名前:132人目の素数さん :2018/05/18(金) 00:23:03.44 ID:+W5C6ZEg
>>506
誤解を避けるために言っておくと、f({a})={b}という書き方をしても構わないという話な
このような表記が許されるということが分かる例という話なら、例えば集合位相入門(松坂)p.30だが


これのどこが、f(集合)=元なんですか?

545 :132人目の素数さん:2018/05/19(土) 00:19:02.80 ID:9nKwXEtV.net
>>540
基礎論とかでそういう導入をするのかもしれない。
一般的な定義と同値なら問題は無いし、嘘とまで言う必要はないだろう。

546 :132人目の素数さん:2018/05/19(土) 00:24:19.21 ID:+EZsmfjZ.net
一般的な定義はどんな感じなんですか?

547 :132人目の素数さん:2018/05/19(土) 00:27:47.79 ID:nPyNoqtI.net
アスペ婆さん

548 :132人目の素数さん:2018/05/19(土) 00:29:49.28 ID:D3NocoQb.net
この問題が分かりません。点QがL1とL2の間に入り込む場合や、L2の向こう側にある場合など、場合をどう分けたらいいでしょうか。

a,bを実数とする。
xy平面上の2点A(-5,-5),B(-5,5)を結ぶ線分から両端を除いたものをL1、2点C(5,-5),D(5,5)を結ぶ線分から両端を除いたものをL2とする。
点P(-9,3)からL1もL2も通らずに点Q(a,b)に至る最短経路の長さをa,bで表し、ab平面に図示せよ。

549 :132人目の素数さん:2018/05/19(土) 00:35:09.63 ID:NIi6yyUc.net
1+2/2+3/3+4/4 + ….. + n/n = n かな。

550 :132人目の素数さん:2018/05/19(土) 00:36:15.50 ID:+EZsmfjZ.net
>>547
わからないんですね

551 :132人目の素数さん:2018/05/19(土) 00:41:23.92 ID:9nKwXEtV.net
>>546
普通に数学する分には、それこそwikipediaの「写像」のページにあるように
>集合 A の各元に対してそれぞれ集合 B の元をただひとつずつ指定するような規則 f が与えられているとき、f を「始域または定義域A から終域 B への写像」といい
ぐらいで十分でしょ
ZFC公理とか気にする文脈なら>>514みたいにすればいい

552 :132人目の素数さん:2018/05/19(土) 00:47:37.72 ID:IVV+JcaX.net
一般的ではない自分好みの記述をしたければいくらでもやればいいとはおもうけど、それならそれでその旨明記せんとダメやろ。一応辞書なんだから。

553 :132人目の素数さん:2018/05/19(土) 00:52:23.24 ID:S5Pbjzeo.net
>>548
問題文おかしくね?
「最短経路の長さを a,b で表す」 は意味があるが
その後にいきなり 「ab平面に図示せよ」 は不自然だ

554 :270:2018/05/19(土) 00:53:13.24 ID:cYWcA9YX.net
>>450

ありがとうございます。やはり反復計算になってしまうのですね。

555 :132人目の素数さん:2018/05/19(土) 00:54:38.70 ID:+EZsmfjZ.net
>>551
なるほど
数学板でもそこまでレベルが低いとは思いませんでした
流石ですね

556 :132人目の素数さん:2018/05/19(土) 01:00:48.90 ID:kHKNlegy.net
君が数学板のレベルをあげるんだ!
人を試すのではなく自分を試せ!

557 :132人目の素数さん:2018/05/19(土) 01:08:16.43 ID:+EZsmfjZ.net
>>506
よく見たらこういうのもひどいですね
なんですか汎写像って

558 :132人目の素数さん:2018/05/19(土) 01:14:42.12 ID:xGV7D22I.net
よく気付いたね。ほめてあげる。
この際だから、今回の文脈に合うように「汎写像」の定義を作ってみたらどう。

559 :132人目の素数さん:2018/05/19(土) 01:15:05.08 ID:+EZsmfjZ.net
あと、ちなみにですけどウィキペディアの集合とか写像云々って基本的には素朴集合論ですからね

ここの回答者は、基礎論どころか普通の集合論すら分かっていないということが判明してしまいましたね

560 :132人目の素数さん:2018/05/19(土) 01:20:44.53 ID:xGV7D22I.net
「素朴集合論」ってのは慣用語の類なので、その中身は人に依りけり。

561 :132人目の素数さん:2018/05/19(土) 01:21:25.54 ID:7y7NJIN+.net
対応(多価写像)の特別な場合として写像を定義するならf({a})={b}をf(a)=bと書く
「対応」を定義せずに単に集合の元を対応させる規則ならそのままf(a)=b

これでいいだろ
前者も後者も同値だろうが

562 :132人目の素数さん:2018/05/19(土) 01:24:11.97 ID:+EZsmfjZ.net
それにケチをつけた人がいたんですよねー

563 :132人目の素数さん:2018/05/19(土) 01:25:00.31 ID:IVV+JcaX.net
あのwikipediaの記事はやっぱりアウトやろ。
あんな一般的でない “abuse of notation” なんのことわりもなく使ってんだから。
しかもあの使い方はあかんやろ。
f(a)と書いた場合それが一般的な意味におけるf(a)なのかf({a})なのか前後の文脈をみないといけなくなる。
自分の論文とかに書く分にはまぁすきにすればいいけどネットで人に公開するとこにかくのはあかんやろ。最低でもその旨但し書きがないと。
wikiは玉石混交やなぁ。

564 :132人目の素数さん:2018/05/19(土) 01:27:41.45 ID:BMhUCtvT.net
>>548
1.点ABCDのいずれも通らない場合、
2.点Aのみを通る場合
3.点Bのみを通る場合
4.点Aと点Cを通る場合
5.点Bと点Dを通る場合
の五通りかと思ったけど、Cに行くには、B経由の方が短いみたいなので4.は
4'.点Bと点Cを通る場合
に置き換えた五通りでいいんじゃない?

あと問題の意図は、図示した領域毎に、aとbの関数で表せっていう意味だよね。

565 :132人目の素数さん:2018/05/19(土) 01:29:06.63 ID:+EZsmfjZ.net
>>563
対応がわかりません、ってはっきり言ったらどうなんですか?
見苦しいですよ

566 :132人目の素数さん:2018/05/19(土) 01:29:32.28 ID:xGV7D22I.net
さて、記号の混乱も収まったようですから、あらためて
>>492 にお答えください。

567 :132人目の素数さん:2018/05/19(土) 01:30:44.14 ID:nPyNoqtI.net
Wikipediaの記事が信用に値しないのは数学に限らないがな
善意で直してやっても差し戻しされるし

568 :132人目の素数さん:2018/05/19(土) 01:31:43.92 ID:IVV+JcaX.net
すごい意味やなwww
図を切り分けてそこに式書き込んでいくのかwww

569 :132人目の素数さん:2018/05/19(土) 01:33:57.93 ID:IVV+JcaX.net
でもwikipediaまじで勉強になるときあるのよ。
へぇぇ、こんな公式あるんやぁぁ!ってびっくらこくときあるもんね〜。
その一方でもうアホかというのもまじってるのがなんとも残念。

570 :132人目の素数さん:2018/05/19(土) 01:35:45.92 ID:xhdfIuy0.net
>>538

k = Floor((√(8n-7) -1)/2) とおく。

a[n] = a[n-1] + 1/(k+1)
 = …
 = a[k(k+1)/2] + (n - k(k+1)/2)/(k+1)
 = k + (n - k(k+1)/2)/(k+1)
 = ……   >>539

571 :132人目の素数さん:2018/05/19(土) 01:36:39.91 ID:+EZsmfjZ.net
>>492
おかしくないです
以上

572 :132人目の素数さん:2018/05/19(土) 02:12:22.82 ID:xhdfIuy0.net
>>523

(1) 2p+q = a_3 > a_2 = 2 より

∴ 2p+q>2,p>0,q>0.

573 :132人目の素数さん:2018/05/19(土) 02:13:28.59 ID:D3NocoQb.net
>>572
それって必要十分なんですか?

574 :132人目の素数さん:2018/05/19(土) 02:17:12.10 ID:xhdfIuy0.net
>>456
 仁義の無さは広島の***をはるかに凌ぐ。とくに池の付く先生は…

575 :132人目の素数さん:2018/05/19(土) 02:40:47.86 ID:p1D17qL6.net
>>560
> 「素朴集合論」ってのは慣用語の類なので、その中身は人に依りけり。

まあそれはその通りだが、基本的にはラッセルの逆理の原因になる集合と真のクラスの区別をしないで
集合に関する記法を「細けぇこたぁいいんだよ」とばかりに気楽に使うのが素朴集合論だと個人的には思ってる
(これを「論」と呼ぶのは変なんだけどね、むしろ集合記法を使った計算を素朴にやってるんだから
「素朴集合算(naive set calculus)」とでも呼ぶべき代物だ)
君が賛同してくれるかどうかは知らんが

だから正にcomprehension schemeで集合を指定せずに特定の性質を満たす「もの」を平気で集めたり
基底の公理や選択公理の存在を知らない・気にしないということを素朴集合算を使う時は平気でやってるわけでね

576 :132人目の素数さん:2018/05/19(土) 03:53:28.43 ID:xhdfIuy0.net
>>572-573

q > 2(1-p)  (0≦p≦1)
q > 1-p   (1≦p≦2)
q > -(1/4)pp (2≦p≦4)
q > 2(2-p)  (4≦p)

577 :132人目の素数さん:2018/05/19(土) 07:29:36.86 ID:D3NocoQb.net
>>576
これは(1)ですか
(2)はどうですか

578 :イナ :2018/05/19(土) 11:07:37.51 ID:G63RZAIS.net
>>548a>5,0<b<5のとき、

PB+BD+DQ=2√5+10+√{(a-5)^2+(5-b)^2}

P→B→D→QとP→B→C→QでPB+BD+DQとPB+BC+CQが等しくなる点Q(a,b)(5<a<b)の式は、
10+√{(a-5)^2+(5-b)^2}=10√2+√{(a-5)^2+(b+5)^2}

579 :132人目の素数さん:2018/05/19(土) 15:37:22.68 ID:zVeK+Elc.net
0.999.... = 1 が数学的に証明できるとか言っちゃうアホ
http://itest.5ch.net/test/read.cgi/news4vip/1526706381/l50

580 :132人目の素数さん:2018/05/19(土) 15:39:39.43 ID:zVeK+Elc.net
めんどくさくなってるから上のスレお前ら来てくれ

581 :132人目の素数さん:2018/05/19(土) 15:51:56.86 ID:NIi6yyUc.net
>0.999.... = 1
定義だろ。

582 :132人目の素数さん:2018/05/19(土) 16:46:26.95 ID:KbYEcUxv.net
>>581
どういう定義がなされているんですか?

583 :132人目の素数さん:2018/05/19(土) 16:52:26.90 ID:B2etM5eT.net
お願いします。

一辺がaの正方形(a≧2)の各頂点を中心とした半径1の円Ciがある。(i=1〜4)
C1,C2,C3,C4の周上にそれぞれ点P,Q,R,Sを取る。
四角形PQRSの面積が最小となる時、PQRSは正方形か長方形となることを示し、
その最小値を求めよ。

584 :132人目の素数さん:2018/05/19(土) 17:11:38.34 ID:KgS6VdgG.net
>>582
無限級数の和

585 :132人目の素数さん:2018/05/19(土) 17:13:29.59 ID:KbYEcUxv.net
>>584
なぜですか?

586 :132人目の素数さん:2018/05/19(土) 17:29:07.97 ID:8zByjxnC.net
wとzは複素数でw+1/w-i=z+1/z-1のとき、w=ziである。
このとき、zの虚部が0のときwの実部が0であることを証明せよ
お願いします。

587 :132人目の素数さん:2018/05/19(土) 17:46:53.13 ID:IHFhkrLD.net
>>586
パッと見だけど

w=a+bi
z=c+di

で計算してみ

588 :132人目の素数さん:2018/05/19(土) 18:00:31.96 ID:GTRz6jNj.net
>>492みたいな証明してる本を見たことない
大抵はこんな感じ

f^-1:B→Aが写像であることは、定義より、Bの任意の元の像f^-1(b)がただ1つから成る、すなわちBの任意の元に対してf(a)=bとなるようなAの元aがただ1つだけあることを意味する。これは明らかにf:A→Bが全単射であることを示す性質である。
よってf^-1が写像であるための必要十分条件はfが全単射であることである。

大体の本だとこういう説明的で素朴な証明で済ませてる

589 :132人目の素数さん:2018/05/19(土) 18:04:51.85 ID:KgS6VdgG.net
>>566
これ何証明しようとしてるの?

590 :132人目の素数さん:2018/05/19(土) 19:08:30.15 ID:imsqXkpp.net
>>589
おそらくは全単射は逆写像を持つ事の証明だと思われる

591 :132人目の素数さん:2018/05/19(土) 20:02:34.92 ID:M4pEwFRY.net
計算用に↓のような電子ペーパーを使っている人っていますか?

https://av.watch.impress.co.jp/docs/news/1117150.html

592 :132人目の素数さん:2018/05/19(土) 20:08:01.86 ID:evXfHz0m.net
>>591
無地のノートでよくね?

593 :132人目の素数さん:2018/05/19(土) 20:26:55.03 ID:D3NocoQb.net
>>583
2点P,Rを固定する。
QをC2上で、SをC4上で動かす。
C2の接線でPQRSの対角線PRに平行なものをl、lとC2の接点をWとする。
同様に、C4の接線でPRに平行なものをm、mとC4の接点をXとする。
△PRQの面積はQ=Wのときに最大になる。
△PRSの面積はS=Xのときに最大になる。

594 :132人目の素数さん:2018/05/19(土) 20:29:46.67 ID:M4pEwFRY.net
>>592

今は、コピー用紙を使っています。

>>591

便利そうですが、コストが高いですね。

そのうち、誰もが持つようになるでしょうね。

595 :132人目の素数さん:2018/05/19(土) 21:18:42.41 ID:aFnYyJyL.net
自殺をしたら地獄に落ちるというのは本当ですか?
無にはなれないのでしょうか?
自分としては無になってもう二度と有になりたくないのですが、無理ですか?

596 :132人目の素数さん:2018/05/19(土) 21:19:10.49 ID:+EZsmfjZ.net
>>595
本当です

無理ですね

597 :132人目の素数さん:2018/05/19(土) 21:25:12.81 ID:xhdfIuy0.net
>>523 (1)

>>576 の補足

b_n = a_{n+1} / a_n とおくと題意より
 b_{n+1} = p + q / b_n,
を満たす。
b_n が収束する ⇔
 y = p + q/x,y = x
が交点α,βをもつ ⇔
(判別式) = pp + 4q ≧ 0  …… (イ)

α = {p - √(pp+4q)}/2,  は反発解
β = {p + √(pp+4q)}/2,  は吸引解

b_n → β (n→∞)となる ⇔
 α < b_1 = 2,
 p < 4 または q > 2(2-p)  …… (ロ)

題意から b_n > 1,
 β ≧ 1,
 p≧2 または q ≧ 1-p  …… (ハ)

q>0 の場合は b_n が振動するから、
 b_2 = a_3/a_2 > 1
 q > 2(1-p)  …… (ニ)
を追加する。

求める (p,q) は、(イ)(ロ)(ハ)(ニ) の共通部分。

598 :132人目の素数さん:2018/05/19(土) 21:28:50.37 ID:aFnYyJyL.net
>>596
根拠を教えてください。

599 :132人目の素数さん:2018/05/19(土) 21:50:47.39 ID:xhdfIuy0.net
>>591-592
 この内容なら、チラシの裏でじゅうぶんぢゃ…

600 :132人目の素数さん:2018/05/19(土) 22:45:38.12 ID:yG/94NGh.net
>>593
それって証明になってますっけ。

601 :132人目の素数さん:2018/05/19(土) 23:17:21.50 ID:IHFhkrLD.net
>>599
チラ裏だと水性インク弾くからなぁ…

602 :132人目の素数さん:2018/05/20(日) 17:30:33.18 ID:1IiDnvUy.net
>>583

・2 < a ≦ √8 の場合
 辺長 {a±√(8-aa)}/2 の長方形のとき最小
 S = (aa-4)/2,

・a > √8 の場合
 一辺が a-√2 の正方形のとき最小
 S = (a-√2)^2,

603 :132人目の素数さん:2018/05/20(日) 17:44:56.48 ID:6S24yfpt.net
>>602
それはすぐにわかるんですが、示す部分お願いします

604 :132人目の素数さん:2018/05/20(日) 22:41:10.42 ID:/rtrEB4Z.net
3次元で直交する2つのベクトルA(ax,ay,az)とB(bx,by,bz)を、Aをz軸の負の方向(0,0,-1)に、
Bをy軸の負の方向(0,-1,0)になるように回転させたいのですが、
そのときの回転軸と回転角を計算で求める方法があれば教えてください。
ベクトルの大きさはとくに問いません。方向だけ合えばよいです。
よく起こる例としてはA(0,0,1)とB(1,0,0)、A(0,0,1)とB(1/√2,1/√2,0)です。
よろしくお願いします。

605 :132人目の素数さん:2018/05/21(月) 01:45:11.71 ID:9YF4F+CN.net
>>604

↑(-Z)A = (ax,ay,az+1)
 ↑(-Y)B = (bx,by+1,bz)
はいずれも回転軸と直交する。
∴回転軸ωはこれらの外積。
 ω = (ay・bz-(az+1)(by+1),(az+1)bx-ax・bz,ax(by+1)-ay・bx),


A(0,0,1) B(sinβ,cosβ,0) のとき
 回転軸ω = (-cos(β/2),sin(β/2),0)
 回転角θ= 180°

606 :132人目の素数さん:2018/05/21(月) 03:14:15.73 ID:Jy2U15S+.net
a,b,cは実数で、a≠0かつc≠0とする。
実数xについての関数f(x)=ax^2+bx+cで、-1≦x≦1において-1≦|f(x)|≦1を満たすものを考える。
以下の問いに答えよ。

(1)f(α)=1かつf(β)=-1となる実数α,βが存在するために、a,b,cが満たすべき必要十分条件を求めよ。
(2)(1)の条件をみたすどのようなa,b,cに対しても、g(x)=|cx^2+bx+a|が-1≦g(x)≦1を満たすことを示せ。

607 :132人目の素数さん:2018/05/21(月) 12:00:06.95 ID:qj3N/8DG.net
>>606
必要十分条件好きやなぁ〜www

608 :132人目の素数さん:2018/05/21(月) 12:02:30.62 ID:qj3N/8DG.net
>>606
(1)a=-c,b=0

(2)a=-cなんだから自明じゃね?

609 :132人目の素数さん:2018/05/21(月) 12:05:19.35 ID:qj3N/8DG.net
>>608
これ凄い間違ってる
無視して

610 :132人目の素数さん:2018/05/21(月) 13:10:04.24 ID:UOXu1HFJ.net
>>606
cx^2+bx+a=x^2f(1/x)を使う

611 :132人目の素数さん:2018/05/21(月) 13:50:38.65 ID:fk94eBOD.net
毎日機械的に問題はっつけてる人、コテ付けてくれないかなあ

612 :132人目の素数さん:2018/05/21(月) 13:51:14.72 ID:ONy1xo51.net
カブリ数物連携宇宙研究機構と東京大学大学院理学系研究科附属ビッグバン宇宙国際研究センターはどっちの方がレベルが高いですか?

613 :132人目の素数さん:2018/05/21(月) 14:47:01.12 ID:5fIdOvA3.net
>>612
ネーミングセンスはカブリのが上

614 :132人目の素数さん:2018/05/21(月) 15:02:50.05 ID:nW2j6zgb.net
カブキにすれば、世界的になれたのに。

615 :132人目の素数さん:2018/05/21(月) 15:11:00.01 ID:4By2DDva.net
>>613
多元数理の方が良い

616 :132人目の素数さん:2018/05/21(月) 15:37:04.92 ID:nW2j6zgb.net
多元数となんも関係ないし・・

617 :132人目の素数さん:2018/05/21(月) 16:31:50.57 ID:ONy1xo51.net
>>613
そういうことじゃなくて、宇宙の研究に関してどっちの方がレベルが高いですか?

618 :132人目の素数さん:2018/05/21(月) 16:37:40.93 ID:q/in2eRY.net
ヒマラヤに釣られる馬鹿共

619 :132人目の素数さん:2018/05/21(月) 17:41:28.69 ID:Jy2U15S+.net
>>611
機械的ではありません、ちゃんと考えてます

620 :132人目の素数さん:2018/05/21(月) 17:59:30.86 ID:fCe1ooKs.net
会話できたんだ
よかった

621 :132人目の素数さん:2018/05/21(月) 20:03:38.92 ID:hotZ9RFh.net
誰か>>583
証明の部分よろしく

622 :132人目の素数さん:2018/05/21(月) 21:24:24.67 ID:bKyOh+u+.net
問題ではないのだけど、

無限大をあらわす∞この記号
infinityの記号は、

sin xの波をループさせたものですよね?

sin x と infinityには
どんな関連性がありますか?

623 :132人目の素数さん:2018/05/21(月) 21:42:13.41 ID:nW2j6zgb.net
あら、ひねくれ天使の輪っかなのね。
https://en.wikipedia.org/wiki/Rider-Waite_tarot_deck

624 :132人目の素数さん:2018/05/21(月) 21:53:08.15 ID:SUavv2Mw.net
>>622
タロットに決まってんじゃん

625 :132人目の素数さん:2018/05/21(月) 22:53:17.70 ID:5rl5YZ/c.net
>>604ですが、>>605の回答がよくわかっておりません。
回転角はどのように出せばよいのでしょうか。
また回転軸と回転角は最低2ついるのではと思っています。

626 :132人目の素数さん:2018/05/21(月) 23:15:56.55 ID:YccZYzuR.net
>>625
^は転置とする。
(100)^をa、(010)^をbにうつす回転行列はc=a×bとして
A=(abc)で与えられる。
Aの固有多項式の2次の係数=-2cosθ、0≦θ≦πとなるをcosθから選ぶ。
このときAの固有値λは1,cosθ+i sinθ,cosθ-i sinθでそれぞれの固有ベクトルu,v,wを計算する。具体的にはA-λIの余因子行列の列ベクトルをとれば良い。
このときAはuの方向を右ねじの方向としてθ回転の行列になる。
…たぶん。

627 :132人目の素数さん:2018/05/21(月) 23:33:41.60 ID:YccZYzuR.net
>>625 >>626
あかん、間違ってる。回転軸は簡単だけど右ねじの方向定めるのムズいorz。

628 :132人目の素数さん:2018/05/21(月) 23:37:22.17 ID:YccZYzuR.net
>>625 >>626
右ねじの方向 = vの実部×vの虚部かな?

629 :132人目の素数さん:2018/05/21(月) 23:49:36.91 ID:5rl5YZ/c.net
>>625です。
>>626-628さんありがとうございます。
詳しく調べようと思うのですが、調べる際のキーワードはありますでしょうか。本やホームページを教えていただけると助かります。

630 :132人目の素数さん:2018/05/22(火) 00:01:35.66 ID:/dd11ki1.net
うーん?あるかなぁ?とりあえず>>626->>628に書いた余因子行列とか固有多項式、固有値、固有ベクトルとかは線形代数の基本なので山程ヒットするとは思う。
ネットで調べてあるかどうか知らないのは
――
任意の直交行列(回転変換を表す行列)Aは別の直交行列Pを用いて
A=PR(θ)P^
と表される。ただし
R(θ)は(100)^,(010)^を(cosθ,sinθ,0)^, (-sinθ,cosθ,0)^に移す回転行列である。
――
あるかなぁ?そんなに証明難しくないのでやってみて下さい。
これと回転行列が外積を保存することを使えば>>626->>628は証明できると思う。

631 :132人目の素数さん:2018/05/22(火) 00:14:10.96 ID:/dd11ki1.net
>>625
ついでなので書いとくと回転を表す流儀は
・回転軸の右ねじの方向と回転量
・オイラー角
・四元数(quotanion)
がメジャーだと思う。余力があれば調べてみて下さい。

632 :132人目の素数さん:2018/05/22(火) 01:39:06.48 ID:RuE2vaj6.net
>>625 >>629

 A' = A - (A・ω)ω,
 (-Z)' = (-Z) + (Z・ω)ω,
はωに直交します。
 θ = arccos{(A'・(-Z)')/|A'||Z'|}
とします。・は内積です。

"剛体回転におけるオイラーの定理" によれば、1度の回転で可能です。
回転軸ωの方向が2、回転角θが1で、自由度3です。

3次元ユークリッド空間の回転は、行列式が1の実直交行列で表わされます。( SO(3) という。)
行ベクトル、列ベクトルは正規直交性をもつため、自由度3です。

オイラー角の場合は、回転軸は z-y-z と決まっており、回転角(α,β,γ)のみを指定します。
自由度3です。

633 :132人目の素数さん:2018/05/22(火) 01:54:12.61 ID:RuE2vaj6.net
ついでなので書いておくと、quaternion ?

quater : 1/4、 4半期

634 :132人目の素数さん:2018/05/22(火) 02:05:39.50 ID:RuE2vaj6.net
>>622

ひねくれてない天使のはランダウの記号 0 ですか。。。

635 :132人目の素数さん:2018/05/22(火) 04:03:52.82 ID:9eWKZkbD.net
NASAのゴダード宇宙飛行センターの中で圧倒的最高の頭脳を誇る理論物理学者と、
オックスフォード大学の中で圧倒的最高の頭脳を誇る数学者が、
理系学問のみに関する学力バトルで勝負をしたらどっちが勝ちますか?

636 :132人目の素数さん:2018/05/22(火) 07:47:15.68 ID:MCp9I6dg.net
>>630-633
>>629です。ありがとうございます。勉強します。

637 :132人目の素数さん:2018/05/22(火) 07:52:57.88 ID:irhSfvXR.net
がんばったのに報われない理由

638 :132人目の素数さん:2018/05/22(火) 07:57:36.57 ID:h7q96Yc3.net
神道とイギリス王室はどっちの方が崇高ですか?

639 :132人目の素数さん:2018/05/22(火) 07:59:03.33 ID:MvGiLf+3.net
幸福の科学を勉強しろ!

640 :132人目の素数さん:2018/05/22(火) 12:11:12.21 ID:yXdy01CV.net
問題:
x の2次方程式 25*x^2 - 35*x + 4*k = 0 が異なる2つの解をもち、
それぞれ sin(θ), cos(θ) で表わされるとき、定数 k の値を求めよ。

↑これについてですが、他スレで、


347 返信:132人目の素数さん[] 投稿日:2018/05/21(月) 15:40:20.96 ID:bPLA4deP [1/2]
>>339
x の2次方程式 25*x^2 - 35*x + 4*k = 0 が異なる2つの解をもち、
それぞれ sin(θ), cos(θ) で表わされる

k=3

という解答を述べているまでだから、問題文で与えられている前提の真偽は関係ない


と言われたのですが、


この問題は、↓の意味ですよね。明らかに。

問題:
x の2次方程式 25*x^2 - 35*x + 4*k = 0 が異なる2つの解をもち、
それぞれ sin(θ), cos(θ) で表わされる。そのとき、定数 k の値を求めよ。  👀
Rock54: Caution(BBR-MD5:1341adc37120578f18dba9451e6c8c3b)


641 :132人目の素数さん:2018/05/22(火) 12:22:24.03 ID:yXdy01CV.net
>>604
>>625
>>636

A → -E3 = (0, 0, -1) の回転軸は、 法線が (A + E3) / 2 で 点 (A - E3) / 2 を通るような平面 P1 に含まれる。
B → -E2 = (0, -1, 0) の回転軸は、 法線が (B + E2) / 2 で 点 (B - E2) / 2 を通るような平面 P2 に含まれる。

明らかに、

L1 := P1 ∩ P2 を軸としてある角度だけ回転すれば、

A → -E3 = (0, 0, -1)
B → -E2 = (0, -1, 0)

とできる。

642 :132人目の素数さん:2018/05/22(火) 12:26:52.38 ID:yXdy01CV.net
A の L1 への射影を pr(A) とする。

明らかに、

arccos([(A - pr(A)) ・ (pr(A) + E3)] / [|A - pr(A)| * {pr(A) + E3}])

が求める回転角である。

向きも容易に求められる。

643 :132人目の素数さん:2018/05/22(火) 12:29:12.40 ID:yXdy01CV.net
訂正します:

A の L1 への射影を pr(A) とする。

明らかに、

arccos( [(A - pr(A)) ・ (-E3 - pr(A))] / [|A - pr(A)| * |-E3 - pr(A)|] )

が求める回転角である。

向きも容易に求められる。

644 :132人目の素数さん:2018/05/22(火) 12:31:11.39 ID:yXdy01CV.net
訂正します:

>>604
>>625
>>636

A → -E3 = (0, 0, -1) の回転軸は、 法線が A + E3 で 点 (A - E3) / 2 を通るような平面 P1 に含まれる。
B → -E2 = (0, -1, 0) の回転軸は、 法線が B + E2 で 点 (B - E2) / 2 を通るような平面 P2 に含まれる。

明らかに、

L1 := P1 ∩ P2 を軸としてある角度だけ回転すれば、

A → -E3 = (0, 0, -1)
B → -E2 = (0, -1, 0)

とできる。

A の L1 への射影を pr(A) とする。

明らかに、

arccos( [(A - pr(A)) ・ (-E3 - pr(A))] / [|A - pr(A)| * |-E3 - pr(A)|] )

が求める回転角である。

向きも容易に求められる。

645 :132人目の素数さん:2018/05/22(火) 12:34:47.08 ID:yXdy01CV.net
訂正します:

>>604
>>625
>>636

A → -E3 = (0, 0, -1) の回転軸は、 法線ベクトルが A + E3 で 点 (A - E3) / 2 を通るような平面 P1 に含まれる。
B → -E2 = (0, -1, 0) の回転軸は、 法線ベクトルが B + E2 で 点 (B - E2) / 2 を通るような平面 P2 に含まれる。

明らかに、

L1 := P1 ∩ P2 を軸としてある角度だけ回転すれば、

A → -E3 = (0, 0, -1)
B → -E2 = (0, -1, 0)

とできる。

A の L1 への射影を pr(A) とする。

明らかに、

arccos( [(A - pr(A)) ・ (-E3 - pr(A))] / [|A - pr(A)| * |-E3 - pr(A)|] )

が求める回転角である。

向きも容易に求められる。

646 :132人目の素数さん:2018/05/22(火) 12:37:47.76 ID:yXdy01CV.net
>>629

平面の方程式
内積
法線ベクトル

などを調べればいいのではないでしょうか?

CGの本を見れば、外積や四元数など色々載っているのではないかと推測します。
日本語の本でまともな本があるかどうかは知りませんが。

647 :132人目の素数さん:2018/05/22(火) 12:41:19.78 ID:yXdy01CV.net
>>645

|A| = |B| = 1 も仮定しています。

648 :132人目の素数さん:2018/05/22(火) 14:46:03.62 ID:Vg28Op4O.net
xy平面の単位円の周および内部を動く点(x,y)に対して
s=ax+by
t=cxy
を考える。
実数a,bが|a|≦1かつ|b|≦1かつ|c|≦1の範囲を変わるとき、(s,t)が動きうる領域がどのように変化するかを述べよ。

649 :132人目の素数さん:2018/05/22(火) 18:53:56.41 ID:Vg28Op4O.net
pを素数、kをp-1以下の正整数とする。
k個の二項係数
pC1,pC2,...,pCk-1,pCk
をすべて割り切る整数のうち、最大のものを求めよ。

650 :132人目の素数さん:2018/05/22(火) 18:55:20.37 ID:Vg28Op4O.net
xを正の実数とする。
1/xの小数部分がx/2に等しくなるようなxを求めよ。

651 :132人目の素数さん:2018/05/22(火) 19:55:44.39 ID:RuE2vaj6.net
>>650

[1/x] = m (整数) とおくと
1/x - m = x/2,

x = √(mm+2) - m のとき
 x > 0,
 m+1 > 1/x > 0,
 ∴ m ≧ 0

x = -√(mm+2) -m のときは
 x < 0,
 1/x = m + x/2 < m,
 ∴ 不適。

652 :132人目の素数さん:2018/05/22(火) 20:01:10.41 ID:i6KIVmKy.net
ふふふ

653 :132人目の素数さん:2018/05/22(火) 22:14:57.70 ID:jnbOMwuE.net
f(x,y)=0のとき(dy/dx)(dx/dy)=1
はよく知られていますが、
熱力学ではよく
f(x,y,z)=0のとき(∂y/∂x)(∂z/∂y)(∂x/∂z)=-1
という「オイラーの連鎖律」を使います。
例えばf(P,V,T)=(PV)/(nRT)-1=0で試してみると、確かに成り立っています。
この連鎖律が一般になりたつことは数学で証明出来るのでしょうか?
ネット上には非厳密な証明しかありません。

654 :132人目の素数さん:2018/05/22(火) 22:26:24.93 ID:50gg+9Qr.net
陰関数定理から
∂y/∂x=-fx/fy
∂z/∂y=-fy/fz
∂x/∂z=-fz/fx
掛け合わせればそうなりますね

655 :132人目の素数さん:2018/05/22(火) 22:26:29.58 ID:bBTBMYkh.net
>>645-647
>>629です。
ありがとうございます。自分でも計算出来そうです。
追加で伺って恐縮なのですが、
L1 := P1 ∩ P2 は常に原点を通る直線になりますでしょうか?原点は移動させずに回転させたいです。
また向きはどのように判断するのでしょうか?
無知で申し訳ありませんがご教示をお願いします。

656 :132人目の素数さん:2018/05/22(火) 23:33:03.91 ID:Vg28Op4O.net
二項係数についての式
{(n,i)・(n,j)}/(n,k)
が整数となるとき、i,j,kが満たす関係式を述べよ。

(注)(a,b)はaCbとも書く。

657 :132人目の素数さん:2018/05/23(水) 09:28:06.70 ID:5Vv/9fhG.net
>>604

>>645

をコード化しました。

コーナーケースは一切考えていません。

https://github.com/for-2ch/for-2ch/blob/master/rotation.ipynb

658 :132人目の素数さん:2018/05/23(水) 09:29:50.69 ID:5Vv/9fhG.net
>>655

P1 および P2 が原点を含むのでその共通部分である L1 も原点を含みます。
すなわち、 L1 は原点を通ります。

向きについては、

外積
3次の行列式
右ねじ

などをキーワードにして調べてください。

659 :132人目の素数さん:2018/05/23(水) 09:44:22.76 ID:5Vv/9fhG.net
>>645
なんか変なところがあるので訂正します:

>>604
>>625
>>636

A → -E3 = (0, 0, -1) の回転軸は、 法線ベクトルが A + E3 で 原点を通るような平面 P1 に含まれる。
B → -E2 = (0, -1, 0) の回転軸は、 法線ベクトルが B + E2 で 原点を通るような平面 P2 に含まれる。

明らかに、

L1 := P1 ∩ P2 を軸としてある角度だけ回転すれば、

A → -E3 = (0, 0, -1)
B → -E2 = (0, -1, 0)

とできる。

A の L1 への射影を pr(A) とする。

明らかに、

arccos( [(A - pr(A)) ・ (-E3 - pr(A))] / [|A - pr(A)| * |-E3 - pr(A)|] )

が求める回転角である。

向きも容易に求められる。

660 :132人目の素数さん:2018/05/23(水) 09:47:41.89 ID:5Vv/9fhG.net
訂正します:

>>655

P1 および P2 が原点を含むのでその共通部分である L1 も原点を含みます。
すなわち、 L1 は原点を通ります。

向きについては、

外積
平衡六面体の体積と3次の行列式
右ねじの進む向き
右手系、左手系

などをキーワードにして調べてください。

661 :132人目の素数さん:2018/05/23(水) 09:48:13.64 ID:5Vv/9fhG.net
訂正します:

>>655

P1 および P2 が原点を含むのでその共通部分である L1 も原点を含みます。
すなわち、 L1 は原点を通ります。

向きについては、

外積
平行六面体の体積と3次の行列式
右ねじの進む向き
右手系、左手系

などをキーワードにして調べてください。

662 :132人目の素数さん:2018/05/23(水) 09:50:45.06 ID:KyFiVz12.net
問A,B,Cの3問からなるテストがあり、配点は問Aが2点、問Bが3点、問Cが5点で10点満点である。
30人の生徒がこのテストを受けたところ、
問A,B,Cの正解者数は順に22人、18人、14人であった。
このとき、得点が5点であった者(AB2問のみの正解者またはC1問のみの正解者)の人数の最大値は
いくらか。

いろいろ当てはめながら調べると、例えば
 「AB2問のみ正解・・・16人、Cのみ正解・・・8人、AC2問のみ正解・・・4人、全問正解・・・2人」の場合
がその最大値を与える場合(つまり24人が答え)になりそうかな、と思ったのですが
ちゃんと解くにはどのように考えればよいでしょうか。
たぶん不等式に持ち込むのではないかと思うのですが難しいです。

よろしきお願いします。

663 :132人目の素数さん:2018/05/23(水) 10:55:02.27 ID:ekI9gSWq.net
http://meijo.info/jump/?http://up.gc-img.net/post_img_web/2018/05/JUZdChqEiWZ2I4Z.jpeg

664 :132人目の素数さん:2018/05/23(水) 10:56:52.11 ID:ekI9gSWq.net
>>663
http://up.gc-img.net/post_img_web/2018/05/JUZdChqEiWZ2I4Z.jpeg

間違えました。こっちのリンクが正解です。
分かる人は、教えて下さい。

665 :132人目の素数さん:2018/05/23(水) 11:10:42.44 ID:SaS67Pru.net
AD=BCよりBF=EA
AD//BCより∠BFG=∠EAH
定義より∠FBG=∠AEH
2角夾辺相等

666 :132人目の素数さん:2018/05/23(水) 11:21:42.72 ID:ekI9gSWq.net
>>665
ありがとうございました。

667 :132人目の素数さん:2018/05/23(水) 12:21:35.63 ID:Waw1BERb.net
整数x,yが互いに素なときに整数a,bがあって
ax+by=1となるようにとれるというのがありますけど
1変数多項式f(x),g(x)がお互いを割り切れないときに
ある多項式a(x),b(x)があって
f(x)*a(x)+g(x)*b(x)=1となるようにとれるっていう命題は真ですか?

真ならどうやって証明できるかおしえていただけませんでしょうか

668 :132人目の素数さん:2018/05/23(水) 12:26:16.26 ID:Waw1BERb.net
>1変数多項式f(x),g(x)がお互いを割り切れないときに

ここ違いました。定数でない共通の多項式を約数に持たないとき、に変更してください

669 :132人目の素数さん:2018/05/23(水) 12:35:31.02 ID:pD12Z7zk.net
単項イデアル整域上で、f(x)とg(x)の最大公約元が1であれば成立する

670 :132人目の素数さん:2018/05/23(水) 12:38:38.91 ID:pD12Z7zk.net
R[x]が単項イデアル整域になることと、Rが体になることは同値だから体上か

671 :132人目の素数さん:2018/05/23(水) 12:53:14.71 ID:Ae/BZbF9.net
Rが整数環だとどうなるんだ?

672 :132人目の素数さん:2018/05/23(水) 12:58:41.40 ID:pD12Z7zk.net
整数環だとf(x)=x, g(x)=x+2とかが反例

673 :132人目の素数さん:2018/05/23(水) 13:27:13.53 ID:7JXq7gOx.net
すまん非常に簡単なのだろうが教えてもらえないだろうか
くだらない質問DAT落ちてたしここしかない

打率4割のバッターが5打席で2安打以上になる確率を求めよ
的な話を振られたんだが仕事に関係ないんだよこれ、4割あったら3本くらいうつやろ!

あとその確率分布に80%以上で収まるには5打席を1セットとして平均何回の試行が必要か?的な話だった

674 :132人目の素数さん:2018/05/23(水) 13:45:32.78 ID:sC+sxD3L.net
我輩は、炭鉱医である、真っ黒け。

675 :132人目の素数さん:2018/05/23(水) 14:16:23.34 ID:I5HdfiGT.net
>>673
> 打率4割のバッターが5打席で2安打以上になる確率を求めよ
> 的な話を振られたんだが仕事に関係ないんだよこれ、4割あったら3本くらいうつやろ!

確率を求めよなのに3本くらい打つやろ!っておかしくね?w

676 :132人目の素数さん:2018/05/23(水) 14:17:22.07 ID:I5HdfiGT.net
まぁ約66%かなぁ

677 :132人目の素数さん:2018/05/23(水) 14:25:16.46 ID:SaS67Pru.net
1-(5C1)((2/5)^1)((3/5)^4)-(5C0)((2/5)^0)((3/5)^5)
=1-162/625-243/3125
=2072/3125
=0.66304

678 :132人目の素数さん:2018/05/23(水) 15:28:14.98 ID:7JXq7gOx.net
>>676 >>677
ありがとう!助かりました!

679 :132人目の素数さん:2018/05/23(水) 17:56:49.12 ID:bfNiVB5g.net
初歩的でもうしわけないが303の1を教えていただけませんか?

680 :132人目の素数さん:2018/05/23(水) 17:57:10.78 ID:bfNiVB5g.net
>>679

https://i.imgur.com/E1afaEq.jpg

681 :132人目の素数さん:2018/05/23(水) 18:21:51.86 ID:RzPeFpNN.net
>>680
両辺に1/2掛けたら左辺の定数係数をcosとsinの形に直す
これで加法定理が使える

682 :132人目の素数さん:2018/05/23(水) 18:38:47.19 ID:I5HdfiGT.net
>>680
(√3)/2sin(x)+1/2cos(x)=1/2
cos(π/6)sin(x)+ sin(π/6)cos(x)=1/2
sin(x+π/6)=1/2
x=0、2π/3

683 :132人目の素数さん:2018/05/23(水) 19:47:07.81 ID:TKWX+2XB.net
>>661
>655です。
大変助かりました。
後は自分で知識を補おうと思います。
ありがとうございましたm(_ _)m

684 :132人目の素数さん:2018/05/23(水) 22:00:04.09 ID:KJNnMDAS.net
>>662
その問題についてだけならCが不正解が16人でAが不正解が8人であることからわかる。


[ABC]+[AB]+[AC]+[A]+[BC]+[B]+[C]+[]=30。
[ABC]+[AB]+[AC]+[A]=22。
[ABC]+[AB]+[BC]+[B]=18。
[ABC]+[AC]+[BC]+[C]=14。
0≦[ABC],0≦[AB],0≦[AC],0≦[A],0≦[BC],0≦[B],0≦[C],0≦[]。
から一つずつ消去していくと
0≦[C]≦8。
0≦[AB]≦16。
2[C]≦[AB]+4。
2[AB]≦[C]+26。

685 :132人目の素数さん:2018/05/23(水) 22:57:33.73 ID:9h9uzlps.net
一辺の長さが1の立方体ABCD-PQRSにおいて、ABの中点をMとする。

(1)この立方体をRMの周りに一回転させてできる立体K1の体積を求めよ。

(2)この立方体をSMの周りに一回転させてできる立体をK2とする。K1とK2の共通部分の体積を求めよ。

686 :132人目の素数さん:2018/05/24(木) 00:12:04.70 ID:XIv78poq.net
>>681-682
ありがとうございます!

687 :132人目の素数さん:2018/05/24(木) 01:05:44.90 ID:zoTaEZJy.net
誰か>>583の面積が最小になる時は長方形になることの証明の部分教えてくれよ

688 :132人目の素数さん:2018/05/24(木) 04:39:07.53 ID:tZqrH1MY.net
大英博物館とNASAはどっちの方が価値がありますか?

689 :132人目の素数さん:2018/05/24(木) 05:48:43.38 ID:y9HXJqcS.net
無限群で任意の元の位数が有限となるものはありますか?

690 :132人目の素数さん:2018/05/24(木) 06:17:29.81 ID:13AhEc3K.net
有限群の無限直積とか

691 :132人目の素数さん:2018/05/24(木) 06:52:59.54 ID:iTcg9zRk.net
>>690
直積じゃ一般にはダメじゃん直和

692 :132人目の素数さん:2018/05/24(木) 06:56:33.33 ID:iTcg9zRk.net
F2[x]とかね
位数同じだから
F2[[x]]でもいいが
直積がダメで直和なら良い例は
ΠZn⊃⊕Zn

693 :132人目の素数さん:2018/05/24(木) 07:03:50.93 ID:AtVlEevv.net
>>687
それが教えてもらう態度か?

694 :132人目の素数さん:2018/05/24(木) 07:10:14.98 ID:T145GsN7.net
わからないんですね

695 :132人目の素数さん:2018/05/24(木) 09:20:39.18 ID:ycuQ9tnZ.net
無限巡回群とヤハウェはどっちの方が偉大ですか?

696 :132人目の素数さん:2018/05/24(木) 13:06:38.76 ID:RERLVteh.net
そんなことして楽しい?

697 :132人目の素数さん:2018/05/24(木) 13:16:31.84 ID:o3kLnmYW.net
自分は尋常じゃないくらい頭が悪いのですが、東京大学理学部数学科に入るという夢があります。
猛烈に勉強をすれば可能性はありますか?

698 :132人目の素数さん:2018/05/24(木) 13:40:27.29 ID:f1FJL5tW.net
>>697
そんなことここで聞いてる暇があったら勉強しろ
お前の可能性はお前しか知らないんだから

699 :132人目の素数さん:2018/05/24(木) 13:58:22.52 ID:LfUXe4pW.net
>>697
3ヶ月前から何か進展はありましたか?

700 :132人目の素数さん:2018/05/24(木) 14:30:01.77 ID:81MNfsiO.net
>>697
6年前から何か進展はありましたか?
きゃはははとか書き込んでこのスレを流した頃ですよ。

701 :132人目の素数さん:2018/05/24(木) 14:35:42.96 ID:c5tI+u1e.net
東大入れそうなガキとか実際芽が出て理学系に進学した連中に突っかかって時間を浪費させたいのが本音だろ
この人類の足手まとい嫉妬婆は。

702 :132人目の素数さん:2018/05/24(木) 14:37:55.43 ID:81MNfsiO.net
ヒマラヤは40代の長野在住のニートおっさん、劣等感婆とは区別しろよ

703 :132人目の素数さん:2018/05/24(木) 14:39:37.90 ID:EQ5K0CF7.net
なんと6年前から荒しを続けているとは!

704 :132人目の素数さん:2018/05/24(木) 14:40:44.80 ID:Is4CTbmF.net
言っとくけど俺40代でも長野在住でもねーよ。

705 :132人目の素数さん:2018/05/24(木) 14:42:58.52 ID:RQu+sJeU.net
じゃ50代ですか?

706 :132人目の素数さん:2018/05/24(木) 14:44:33.11 ID:Is4CTbmF.net
50代でもねーよ。

707 :132人目の素数さん:2018/05/24(木) 14:45:33.59 ID:jsSCcTfs.net
「ニートおっさん」の部分は否定しないのか…

708 :132人目の素数さん:2018/05/24(木) 14:48:14.93 ID:Is4CTbmF.net
まぁ、でも俺の最大の夢は、東大理学部数学科に入ることではなくて、
無になってもう二度と有にならないことなんだ。
自殺をしても無にはなれないのかな?
それどころか、地獄に落ちるのかな?

709 :132人目の素数さん:2018/05/24(木) 14:48:32.29 ID:EQ5K0CF7.net
>>697 ぐぐったら2年前のが出てきた

https://tamae.5ch.net/test/read.cgi/shihou/1455701014/65
65 :氏名黙秘:2016/02/24(水) 14:31:46.85 ID:R4SfjPiz
東京大学理学部数学科に入りたいのですが、東大の理学部数学科は天才以外はやっていけないところなのでしょうか?
ちなみに自分は、尋常じゃないくらい頭が悪いです。
しかし、数学や物理学などに興味があります。
だから、東大の理学部数学科に入りたいのですが、やはり無理なんでしょうか?

710 :132人目の素数さん:2018/05/24(木) 14:51:47.33 ID:Is4CTbmF.net
どうすれば無になってもう二度と有にならなくて済むのか?
有は嫌だ。

711 :132人目の素数さん:2018/05/24(木) 14:53:41.22 ID:RQu+sJeU.net
今日はいつもと口数多くて口調も違いますね
どうしたんですか?

712 :132人目の素数さん:2018/05/24(木) 14:59:24.54 ID:Is4CTbmF.net
もっともっと思索を続けて究極を見つけたいという気持ちも少しはあるが、もう難しい。
究極を見つけられなくても良いからとにかく無になってもう二度と有になりたくないという気持ちの方が強くなりつつある。

713 :132人目の素数さん:2018/05/24(木) 15:01:17.97 ID:81MNfsiO.net
7年前の物理板の書き込み

335 名前:ご冗談でしょう?名無しさん [sage]: 2012/01/12(木) 12:04:04.36 ID:???
日本の山でお願いします。
もの凄く雪深い山でお願いします。

714 :132人目の素数さん:2018/05/24(木) 15:02:50.80 ID:fh7Wwi2G.net
書き込み続ける事を止めたときに無になる

715 :132人目の素数さん:2018/05/24(木) 15:04:34.65 ID:jsSCcTfs.net
5億年ボタンってあるじゃん

716 :132人目の素数さん:2018/05/24(木) 15:05:02.54 ID:Is4CTbmF.net
死後の世界とか生まれ変わりとかって本当にあるんでしょうか・・・?

717 :132人目の素数さん:2018/05/24(木) 15:13:52.35 ID:81MNfsiO.net
7年前にヒマラヤのスレが男女板に残っている。ヒマラヤの考えは興味深い(笑)

1 名前:名無しさん 〜君の性差〜[] 投稿日:2012/02/27(月) 18:51:07.40 ID:YQFwRny2 [1/2]

1.脳の左右子脳分業化の遅れによる論理的思考力、イメージ力の未発達。

2.化粧・香水・ハイヒール等外見をごまかす習慣によって作られたごまかす脳回路
  による根本的問題解決能力の欠如。
  
3.自分で自分に嘘をつく思考回路による真実を追究する脳回路の発達障害。

4.1〜3の総合的効果による客観的思考力の未発達。
  その結果、自分の願望と現実の区別が曖昧になる。

その他の多くの意見をどうぞ。

718 :132人目の素数さん:2018/05/24(木) 15:15:57.14 ID:Is4CTbmF.net
>>717
それ俺の書き込みじゃねーぞ。

719 :132人目の素数さん:2018/05/24(木) 16:15:47.75 ID:ToF5KKWY.net
正三角形△ABCの辺AC上に、AD:DC=1:3となる点Dをとります。
またBDをDの方向に延長し、BE=BAとなる点Eをとります。
△ABCと合同な△EFGを、FG⊥BE、BF<BEとなるようにつくるとき、△ABCの内部で△EFGの内部でもある部分の面積を求めなさい。

720 :132人目の素数さん:2018/05/24(木) 16:29:51.17 ID:cXIQpRm3.net
>>719
>>611

721 :132人目の素数さん:2018/05/24(木) 17:13:52.03 ID:c5tI+u1e.net
即身成仏させられてる最中の徳の高いお坊さんがお経を唱えてる限りはなんかまだ息がある証拠だと思って皆スルー推奨ってことだな。
まあこの荒らしは何の徳も感じさせないけど

722 :132人目の素数さん:2018/05/24(木) 22:28:48.48 ID:iTcg9zRk.net
>>702
何でそういう設定なん?

723 :132人目の素数さん:2018/05/24(木) 22:40:08.77 ID:XPKkhwEN.net
ヒットマークを探してるんでしょ
実際ニートでおっさんなのは確定みたいだし

724 :132人目の素数さん:2018/05/24(木) 23:11:36.41 ID:wK4NR8t+.net
120人を40人ずつ3学級に振り分けた中学校があるとします。
一年に一回クラス替えをするとして、以下の確率を教えてください。
@ある人Aが1〜3年の間に一回も同じ人と同じクラスにならない確率
Aある人Aと3年間同じクラスの人がいない確率

725 :132人目の素数さん:2018/05/24(木) 23:54:54.74 ID:XPKkhwEN.net
もっとはっきり書け

i

ある人Aが、1〜3年の間に一回もある人Bと同じクラスにならない確率?
(そこそこ高い)

ある人Aが、1〜3年の間に二回以上同じクラスになった人がいない確率?
(めっちゃ低い)

ii

ある人Aと3年間を通して一回以上同じクラスになった人がいない確率?
(0)

ある人Aと3年間三回同じクラスになった人がいない確率?
(かなり高い)

726 :132人目の素数さん:2018/05/25(金) 01:03:57.09 ID:b68wSvtw.net
俺から見て問題の趣旨は明らかだがな
はっきりも何も、問われてることは明記されてる

727 :132人目の素数さん:2018/05/25(金) 03:31:04.32 ID:FzI0O2aB.net
次の積分を求めよ
∫∫e^(x^3)dxdy
D={(x,y) : 0≦y≦1,√y≦x≦1}
お願いします

728 :132人目の素数さん:2018/05/25(金) 03:38:51.14 ID:/yyhiNqF.net
1辺の長さが1の立方体ABCD-EFGHがある。
線分GCの中点をI、線分BIを3:1に内分する点をJとする。線分AJと三角形BDEの交点をPとするとき、APベクトルをABベクトル、ADベクトル、AEベクトルの1次結合で表せ
https://i.imgur.com/duJBhkr.jpg

729 :132人目の素数さん:2018/05/25(金) 06:51:18.84 ID:ohjGIEVt.net
>>727
先にxで積分するのは大変なので、まずyで積分しよう。
 D = { (x,y):0≦x≦1,0≦y≦x^2 }
と表わして、
 ∫[0,x^2] e^(x^3) dy = e^(x^3)・x^2,
 (与式) = ∫[0,1] e^(x^3)・x^2 dx = [ (1/3)e^(x^3) ](x=0,1) = (e-1)/3,

730 :132人目の素数さん:2018/05/25(金) 09:32:14.69 ID:ohjGIEVt.net
>>719

直線BDE をx軸とする。

A (14L,(2√3)L)
B (0,0)
C (10L,-(6√3)L)
D (13L,0)
E (1,0)
F (1-(√3)/2,-1/2)
G (1-(√3)/2, 1/2)
L = 1/(4√13).

731 :132人目の素数さん:2018/05/25(金) 09:51:56.08 ID:/EbXlBQE.net
>>722
ソースは空手板、以下略

732 :132人目の素数さん:2018/05/25(金) 09:58:35.44 ID:ohjGIEVt.net
>>728

ABをx軸,ADをy軸,AEをz軸にとる。(デカルト座標)
A (0,0,0)
B (1,0,0)
C (1,1,0)
D (0,1,0)
E (0,0,1)
F (1,0,1)
G (1,1,1)
H (0,1,1)
I (1,1,1/2)
J (1,3/4,3/8)
P (x,y,z)
とする。
線分AJ  x:y:z = 8:6:3
△BDE  x+y+z = 1
より
P (8/17,6/17,3/17)

733 :132人目の素数さん:2018/05/25(金) 12:30:31.14 ID:Apkabz57.net
>>728
Vectors AB↑, AD↑, and AE↑ are linearly independent.

AJ↑=(3/4)AI↑+(1/4)AB↑=(3/4)((1/2)AC↑+(1/2)AG↑)+(1/4)AB↑=(3/4)((1/2)(AB↑+AD↑)+(1/2)(AB↑+AD↑+AE↑))+(1/4)AB↑=AB↑+(3/4)AD↑+(3/8)AE↑.

sAB↑+tAD↑+(1-s-t)AE↑=uAB↑+(3/4)uAD↑+(3/8)uAE↑
⇔s=u, t=(3/4)u, (1-s-t)=(3/8)u
⇔s=u=8/17, t=6/17.

AP↑=(8/17)AB↑+(6/17)AD↑+(3/17)AE↑.

734 :132人目の素数さん:2018/05/25(金) 12:52:11.00 ID:eFov7qL5.net
n個の物を一列に並べるパターンはn!通りというのは直感的には明らか(n個のものから1つ選んで、その後n-1個のものから1つ選んで.....を繰り返す)ですが、これはどのように数学的に正当化されているのですか?
そもそもn元集合からn元集合への全単射の個数をn!と定義しているのか、有限回の操作というのは何か公理的に特徴づけられているのか...
数学を真面目に取り組んだことが無いので変なことを言っているとは思いますが、回答よろしくおねがいします

735 :132人目の素数さん:2018/05/25(金) 13:27:01.52 ID:o41IAd7Y.net
>>734
https://ja.wikipedia.org/wiki/%E7%BD%AE%E6%8F%9B_(%E6%95%B0%E5%AD%A6)

736 :132人目の素数さん:2018/05/25(金) 16:00:45.19 ID:20K3TOAP.net
>>729
なぜxとyの範囲をそういう風に変えれるんですか?

737 :132人目の素数さん:2018/05/25(金) 16:14:13.69 ID:r6E4Z3zJ.net
>>736
マルチに答える義理はない
教科書嫁

738 :132人目の素数さん:2018/05/25(金) 19:10:07.69 ID:VrECD5RP.net
>>725
ありがとうございます。二つとも後者です。

739 :132人目の素数さん:2018/05/25(金) 21:31:20.31 ID:bFr/Rjwl.net
4つのドアがあります

それぞれのドアを開けると1または9の表記のあるプレートが1枚置かれているものとします
1のプレートは1枚
9のプレートは3枚 
あなたは4つのドアから一つを選択します
さてあなたがドアを選択した後に選択外のドアを開いたところ9のプレートがありました
あなたの選択したドアの向こうに1のプレートがある確率は変動していますか?

この問題なんですが、
4分の1のままですよね?

740 :132人目の素数さん:2018/05/25(金) 21:34:14.77 ID:ljSfkNMq.net
>>739
1/3です

1と9がそれぞれ1枚ずつの場合を考えてみましょう
もう一方が9だとわかった時点で自分のが1だということが確定しますね

741 :132人目の素数さん:2018/05/25(金) 21:46:52.82 ID:bFr/Rjwl.net
>>740
そうなりますか
ありがとうございます

742 :132人目の素数さん:2018/05/25(金) 21:55:56.04 ID:lc5apeh1.net
>>739
1/4のまま変わらない
仮に自分の選んだドアをAとし、それ以外のドアをBCDとする

選択外のDのドアを開けるという行為は

1)実際にDに1がある
2)実際にはDには1はない

この2つの分岐の判明過程にしかすぎんからな
確率は1/4

もしAのドア開けたあとBCDのドアをシャッフルするなら1/3

743 :132人目の素数さん:2018/05/25(金) 22:04:50.58 ID:fpgsNXPt.net
↑これが数学板の実力です
専門板なのに異常にレベルが低い
せいぜい数学の少しできる高校生レベル


ここの回答者って、レベル低いんですね

744 :132人目の素数さん:2018/05/25(金) 22:05:00.39 ID:lc5apeh1.net
ほんまかどうかはしらんのやが
うわさでは、数学板では、早稲田の問題
間違えてる答えのほうが「正しい」とする意見が主流になったらしい

さすが5ちゃん、アホばっかりwwwwwwwwwwwwwwwwwwwwww

ID:ljSfkNMqも自分が再抽選してることに気づいてないアホ

745 :132人目の素数さん:2018/05/25(金) 22:07:32.53 ID:fpgsNXPt.net
>>744
4つのドアがあります

それぞれのドアを開けると1または9の表記のあるプレートが1枚置かれているものとします
1のプレートは1枚
9のプレートは1枚 
あなたは2つのドアから一つを選択します
さてあなたがドアを選択した後に選択外のドアを開いたところ9のプレートがありました
あなたの選択したドアの向こうに1のプレートがある確率は変動していますか?

746 :132人目の素数さん:2018/05/25(金) 22:07:53.92 ID:fpgsNXPt.net
>>744
2つのドアがあります

それぞれのドアを開けると1または9の表記のあるプレートが1枚置かれているものとします
1のプレートは1枚
9のプレートは1枚 
あなたは2つのドアから一つを選択します
さてあなたがドアを選択した後に選択外のドアを開いたところ9のプレートがありました
あなたの選択したドアの向こうに1のプレートがある確率は変動していますか?

747 :132人目の素数さん:2018/05/25(金) 22:09:05.68 ID:lc5apeh1.net
52枚の正しいトランプを俺が一枚引く時
「俺が赤を引ける確率」は頻度主義によって26/52という数式によって1/2なんやが

俺が一枚引いたあと
「俺が赤を引けた確率」は1/2なんかに絶対ならんwwwwwwwwwwwwwww
100%か0%かやwwwwwwwwwwwwww

無限に繰り返す前提あるなら、両者はかぎりなく近づきはするんやが、そもそも
「俺が赤を引ける確率」と「俺が赤を引けた確率」は別種のもんやねん

多分数学板にそれを理解できてる奴はほとんどおらんと思うwwwwww
ほとんどが理学部数学科未満の
数1数2レベルの理解で確率を語ってる

748 :132人目の素数さん:2018/05/25(金) 22:10:25.50 ID:fpgsNXPt.net
>>747
2つのドアがあります

それぞれのドアを開けると1または9の表記のあるプレートが1枚置かれているものとします
1のプレートは1枚
9のプレートは1枚 
あなたは2つのドアから一つを選択します
さてあなたがドアを選択した後に選択外のドアを開いたところ9のプレートがありました
あなたの選択したドアの向こうに1のプレートがある確率は変動していますか?

749 :132人目の素数さん:2018/05/25(金) 22:11:56.42 ID:lc5apeh1.net
早稲田問題の解が10/49だと信じている知的障害猿が観察できるスレ

馴れ合い [無断転載禁止]c2ch.net
ttp://potato.2ch.net/test/read.cgi/mj/1483111206/
馴れ合い2 [無断転載禁止]c2ch.net
ttp://egg.2ch.net/test/read.cgi/mj/1485659365/
【麻雀に】何切る?【正解はない】 [転載禁止]c2ch.net
ttp://potato.2ch.net/test/read.cgi/mj/1438922977/
中森明菜11・30ロックアルバムDSも追加 [無断転載禁止]c2ch.net
ttps://potato.5ch.net/test/read.cgi/mj/1476413552/

750 :132人目の素数さん:2018/05/25(金) 22:13:02.19 ID:fpgsNXPt.net
>>749
一方が1で他方が9

一方が9だとわかったとき他はどっちになっている?

これをわからない、と答えるということはどういうことですか?

あなたがバカだということですよね

751 :132人目の素数さん:2018/05/25(金) 22:13:47.68 ID:lc5apeh1.net
>>751
続きは?

752 :132人目の素数さん:2018/05/25(金) 22:14:15.26 ID:fpgsNXPt.net
>>751
続きはあなたの答えを聞いてからにしましょうか

753 :132人目の素数さん:2018/05/25(金) 22:18:22.10 ID:6I7obK3m.net
常に9のプレートがあるドアを開けるので変わらない。

754 :132人目の素数さん:2018/05/25(金) 22:18:48.32 ID:fpgsNXPt.net
>>753
一方が1で他方が9

一方が9だとわかったとき他はどっちになっている?

これをわからない、と答えるということはどういうことですか?

あなたがバカだということですよね

755 :132人目の素数さん:2018/05/25(金) 22:21:01.63 ID:6I7obK3m.net
9のプレートは3枚

756 :132人目の素数さん:2018/05/25(金) 22:22:15.45 ID:fpgsNXPt.net
具体例を無視するんですね

9のドアを必ず開ける
しかし、その場合でも、その開けたドアに1がある場合が全体から除かれてるんですよ
だから母数が減るから確率が増えるんです

757 :132人目の素数さん:2018/05/25(金) 22:27:56.44 ID:6I7obK3m.net
常に1を開けずに9を開ければいい。

758 :132人目の素数さん:2018/05/25(金) 22:31:07.42 ID:lc5apeh1.net
1/4で当たるカードが外れた時
「目の前に一枚ある、ハズレと大きく書かれたカードは
ハズレなのかアタリなのか最抽選してみよう!!!」

ここに答えあるが
これだけコピペしても
猿には理解できんやろ???

俺は猿までいちいち相手する気ないで

759 :132人目の素数さん:2018/05/25(金) 22:31:20.57 ID:fpgsNXPt.net
開ける前
1999
9199
9919
9991

開けたあと
199(9)
919(9)
991(9)
999(1)

19(9)9
91(9)9
99(1)9
99(9)1

1(9)99
9(1)99
9(9)19
9(9)91

(1)999
(9)199
(9)919
(9)991

()のところのドアを開けるとしましょう

頭が悪いなら、具体的に書き出せばいいんです

760 :132人目の素数さん:2018/05/25(金) 22:31:38.43 ID:lc5apeh1.net
アホ猿は自分から「母数が減るから」とか言うてるなw
とど松再抽選

761 :132人目の素数さん:2018/05/25(金) 22:32:16.78 ID:fpgsNXPt.net
>>758
できませんね
ハズレはハズレでしょうからね
トンデモ猿の考えは全くわかりませんね

762 :132人目の素数さん:2018/05/25(金) 22:32:45.05 ID:fpgsNXPt.net
>>760
>>759

(1)はダメなんですよ
ドアを開けたら1があったんです

763 :132人目の素数さん:2018/05/25(金) 22:34:40.64 ID:lc5apeh1.net
>>759
それ懐かしいなwふいた
次は漸化式も出してくるやろうな

764 :132人目の素数さん:2018/05/25(金) 22:35:44.35 ID:fpgsNXPt.net
>>763
どこがおかしいのかいってみてください?

あと

一方が1で他方が9

一方が9だとわかったとき他はどっちになっている?

これをわからない、と答えるということはどういうことですか?

あなたがバカだということですよね

↑これにも一言お願いします
逃げないでくださいね

765 :132人目の素数さん:2018/05/25(金) 22:36:16.69 ID:6I7obK3m.net
1があるドアを開けないようにすればいい。

766 :132人目の素数さん:2018/05/25(金) 22:38:03.88 ID:lc5apeh1.net
>>764
逃げとんのどっちやねんw
これ以上ここでやると迷惑やろしこっち来いや

***何切る?統一スレッド 6***
http://egg.5ch.net/test/read.cgi/mj/1527211154/

俺は逃げも隠れもせんがお前が逃げるのは自由やw

767 :132人目の素数さん:2018/05/25(金) 22:38:40.11 ID:fpgsNXPt.net
>>765
開ける前
1999
9199
9919
9991

開けたあと
199(9)
919(9)
991(9)

19(9)9
91(9)9
99(9)1

1(9)99
9(9)19
9(9)91

(9)199
(9)919
(9)991

()のところのドアを開けるとしましょう
(1)は1があったので開けませんでしたから、今回の状況と合致しませんから、この表から覗きました

はい、めでたく確率が1/3になりましたね

768 :132人目の素数さん:2018/05/25(金) 22:38:44.53 ID:lc5apeh1.net
ノナメってコテ付けとるからすぐわかるやろ
まぁ逃げるんやろけどw

769 :132人目の素数さん:2018/05/25(金) 22:40:13.06 ID:fpgsNXPt.net
私はしつこいですよ?
いいですよ行きましょう

770 :132人目の素数さん:2018/05/25(金) 22:40:28.89 ID:R0Bevpsx.net
両方NGにしました

771 :132人目の素数さん:2018/05/25(金) 22:41:18.19 ID:R0Bevpsx.net
両方出てってくれ

772 :132人目の素数さん:2018/05/25(金) 22:41:43.44 ID:6I7obK3m.net
>>767
それ等確率じゃない。

773 :132人目の素数さん:2018/05/25(金) 22:42:18.56 ID:fpgsNXPt.net
>>772
どこがどう等確率でないんですか?
具体的な数字を出して説明してください

774 :132人目の素数さん:2018/05/25(金) 22:47:03.97 ID:fpgsNXPt.net
まーじゃんの話しかしなくなったので帰ってきました
つまらないですね

775 :132人目の素数さん:2018/05/25(金) 22:48:26.97 ID:CY9jQQ5L.net
やっぱり逃げたなw
もし気がむいたら、時間かかると思うけど
上にリンク張ってる4スレ全部読んでみて
自分がいかに痴呆かわかると思うから

776 :132人目の素数さん:2018/05/25(金) 22:49:18.10 ID:fpgsNXPt.net
>>775
どうしてID変えたんですか?

あなたが逃げたんですよね?
私はあなたの問いに答えましたよ?
あなたの俺が赤を引けた確率云々の話は正しいです

なにが言いたいのかはっきりしてくださいね

777 :132人目の素数さん:2018/05/25(金) 22:52:06.26 ID:fpgsNXPt.net
おや?レスが途絶えましたね

778 :132人目の素数さん:2018/05/25(金) 22:53:10.72 ID:fpgsNXPt.net
>>772
さーん、まだですかー?

779 :132人目の素数さん:2018/05/25(金) 22:53:39.82 ID:CY9jQQ5L.net
>>776
上の4スレ読んだ?

780 :132人目の素数さん:2018/05/25(金) 22:54:13.47 ID:fpgsNXPt.net
>>779
読む気がないので要点だけ言ってください

781 :132人目の素数さん:2018/05/25(金) 22:56:23.49 ID:b68wSvtw.net
数学で議論するだけなら勝手だが、罵り合うんだったら目障りだから消えろ

782 :132人目の素数さん:2018/05/25(金) 23:18:27.40 ID:6I7obK3m.net
最初に選んだドア,選んでないドア,選んでないドア,選んでないドア
1999 a
9199 b
9919 c
9991 d

1(9)99 e
19(9)9 f
199(9) g
91(9)9 h
919(9) i
9(9)19 j
991(9) k
9(9)91 l
99(9)1 m

e+f+g=a
h+i+j+k+l+m=b+c+d
だから変わらない。

783 :132人目の素数さん:2018/05/25(金) 23:22:36.82 ID:eiF8uoF2.net
1/4が3/9になっとるやんけ

784 :132人目の素数さん:2018/05/25(金) 23:29:35.51 ID:fpgsNXPt.net
一方が1で他方が9

一方が9だとわかったとき他はどっちになっている?

やっぱ、これでわからなきゃ、もうなのやったってダメですね

785 :132人目の素数さん:2018/05/25(金) 23:43:09.29 ID:YH4wabrP.net
>>735
読みましたが疑問は晴れませんでした

786 :132人目の素数さん:2018/05/25(金) 23:48:15.72 ID:6I7obK3m.net
a=e+f+g
b=h+i
だから
a=b>0
e=f=g=h=i
にはならない。

787 :132人目の素数さん:2018/05/25(金) 23:53:19.76 ID:fpgsNXPt.net
>>786
では、具体的にはいくらになるんですか?

788 :132人目の素数さん:2018/05/25(金) 23:55:47.67 ID:ow7D2QsB.net
>>785
は?2個の置換はいくつ?3個は?

789 :132人目の素数さん:2018/05/25(金) 23:57:48.94 ID:YH4wabrP.net
>>788
それは分かりますがと書こうと思ったら数学的帰納法でn!になることが示せますね
ありがとうございます

790 :132人目の素数さん:2018/05/26(土) 00:09:06.43 ID:f9RTKmih.net
「群Gのある部分群Hが全てのGの部分群を含むとき、Gは巡回群で位数は素数のべき乗になることを示せ」
という問題で、Hに含まれないaでGは生成されるので巡回群になることは分かったのですがそこから進めません
1.aの位数が無限だと矛盾
2.aの位数をnとしたときnを割る素数が2つ存在したら矛盾
の2つを示せれば良いとは思うんですがヒントを下さい

791 :132人目の素数さん:2018/05/26(土) 00:10:11.27 ID:f9RTKmih.net
>>790
Hは真の部分群でGの全ての真の部分群を含む
です

792 :132人目の素数さん:2018/05/26(土) 00:14:13.81 ID:efhvyUI2.net
>>782
今思いましたけどこれ問題すり替わってますよね

モンティホールじゃないですよ
たまたま1がなかったんです

793 :132人目の素数さん:2018/05/26(土) 00:14:31.40 ID:efhvyUI2.net
でも選択的に選んだら変わらないんですね

794 :132人目の素数さん:2018/05/26(土) 00:14:46.94 ID:efhvyUI2.net
私もよくわかってなかったですね

あぁ、頭よくなりたい

795 :132人目の素数さん:2018/05/26(土) 00:35:28.15 ID:d0tKLffX.net
>>790
>1.aの位数が無限だと矛盾
無限だとZと同型だから極大部分群がいくつもある

796 :132人目の素数さん:2018/05/26(土) 00:39:20.19 ID:d0tKLffX.net
>>790
>2.aの位数をnとしたときnを割る素数が2つ存在したら矛盾
Znm
(n,m)=1
とするとZnとZmと同型な部分群がありそれらを両方含む部分群の位数はnmの倍数だからZnm全体つまり真部分群ではない

797 :132人目の素数さん:2018/05/26(土) 01:33:49.94 ID:vnDRDQ0M.net
>>790
Hを極大部分群としてx∈G\HをとればG=<x>になる。
こいつの位数が有限で素数べきを言えば良い。
結局位数nが∞でもnを割り切る素数が2つある場合でも極大部分群が2つあることを示せば良い。

798 :132人目の素数さん:2018/05/26(土) 01:49:06.66 ID:Tm+bfCXy.net
>>790

2.
#G は有限として、素因数に分解する。
 #G = Σ[i=1,k] (p_i)^(e_i),  e_i≧1
Sylowの定理より、位数 (p^i)^(e^i) の部分群 h_i が存在。
k≧2 と仮定すると h_i ⊂ G    (真部分群)
>>791 により h_i ⊆ H
Lagrangeの定理より、 (p_i)^(e_i) = #(h_i)| #H
これがすべての i について成り立つから
 #G = Σ[i=1,k] (p_i)^(e_i) | #H,
一方、 >>791 より
 G ⊃ H    (真部分群)
 #G > #H
(矛盾)

799 :132人目の素数さん:2018/05/26(土) 03:00:37.14 ID:kUd1LxPB.net
一辺の長さが1の立方体ABCD-EFGHの面CDHGの内接円をKとする。
またC上に点Pをとり、直線APを軸にこの立方体を一回転させてできる立体をVpとする。
PがC上を一周するとき、空間内でVpに含まれうるを領域Vpcとする。点AとVpc上の点Qの距離をLaqとするとき、Laqの最大値を求めよ。

800 :132人目の素数さん:2018/05/26(土) 03:34:11.62 ID:rN3zALnO.net
問題よんで損した。

801 :132人目の素数さん:2018/05/26(土) 04:12:30.94 ID:P1CW6u7K.net
>>790
H は G の真の部分群だから、a∈G−H が取れる。<a> は G の部分群だから、
もしこれが真の部分群なら、<a>⊂H となって a∈G−H に矛盾する。
よって、G=<a> である。次に、異なる素数 p,q を任意に取る。<a^p>,<a^q> は
ともに G の部分群である。もし両方とも真の部分群なら、<a^p>,<a^q>⊂H となるので、
特に a^p,a^q∈H である。Hは群であるから、k,l∈Z に対して a^{pk+ql}∈H である。
gcd(p,q)=1 に注意して、ある k.l に対して pk+ql=1 なので、a^1∈H となり、
<a>⊂H となり、よって G⊂H となって矛盾する。よって、<a^p>=G または <a^q>=G が成り立つ … (1)
もし a^n=e なる n≧1 が存在しないなら、<a^p>≠G, <a^q>≠G となることが言えるので(1)に矛盾する。
よって、a^n=e なる n≧1 が存在する。そのような n のうち最小のものを再び n と置く。
このとき、G=<a> は位数nの巡回群である。n=p_1^{e_1}…p_m^{e_m} と素因数分解する。
もし m≧2 ならば、<a^{p_1}>≠G, <a^{p_2}>≠G となることが言えるので(1)に矛盾する。
よって m=1 であり、n=p_1^{e_1} となる。よって、G は位数が素数ベキの巡回群である。

802 :132人目の素数さん:2018/05/26(土) 18:44:24.54 ID:DXXWtdJl.net
260 ノナメ ◆fR1KiTvorM [] 2018/05/26(土) 18:37:08.45 ID:e5pwOtO8

野球板もプロ選手が書いてるんちゃうからな
バット20年以上握ったこともないトラキチとかカープファンや
数学板もそんなもんやでアホばっかり

803 :132人目の素数さん:2018/05/26(土) 19:06:07.80 ID:HUORxEdm.net
X={a,b,c}の時にXを覆う集合族はいくつあるか?
さらに、その中で分割はいくつあるか?

考え方がわかりません。
覆っているというものはつまり
2^Xからいくつかの要素を取り出した集合族Uがあり、
要素をu_iとしたとき、u_iの和集合がXになるもの全てということでしょうか?

また、分割というものは
{{a},{b},{c}}、{{a,b},{c}}(似たようなもの他2つ)
{{a,b,c}}の5つということになりますか?
回答お願いします

804 :132人目の素数さん:2018/05/26(土) 19:23:22.54 ID:efhvyUI2.net
>>802
しばいてきますね

805 :132人目の素数さん:2018/05/26(土) 19:30:17.72 ID:f9RTKmih.net
790です
皆さんのおかげで解けました
ありがとうございました

もう一つ、aの位数が無限のとき、整数全体のなす加法群Zとの同型を考えればa^2が生成元にならないことは分かるのですが、同型を取らずに直接示すにはどのようにすればいいでしょうか

806 :132人目の素数さん:2018/05/26(土) 22:10:52.49 ID:p7ZlenKz.net
>>805
a^2が生成源ならa=(a^2)^nを満たす整数が取れるけど、それは位数有限に矛盾してるでヨサゲ

807 :132人目の素数さん:2018/05/26(土) 22:15:02.65 ID:p7ZlenKz.net
>>803
集合族全体は256個。
被覆がaを含まないのは16個
被覆がa,bを含まないのが4個
被覆が何にも含まないのが2個

808 :132人目の素数さん:2018/05/26(土) 22:29:17.22 ID:FBhGdsdh.net
>>806
位数無限に矛盾しているですね
ありがとうございます

809 :132人目の素数さん:2018/05/27(日) 13:10:32.22 ID:poggp8He.net
199 焼き鳥名無しさん sage 2018/05/14(月) 21:11:39.39 ID:x3xQuMQL
えっ、要するにこういうことか?

1p,9p,9p,9pの4枚をよく混ぜ伏せて並べる

A B C D

この初期状態の時、左端、Aの牌が1pである確率は1/4。
ここで、Aの牌をめくったら9pでした。

ノナメ理論だと、この時『Aが1pである確率』って1/4のままなの?


これも、これだけは、yesかnoかだけたのむわ


200 ノナメ ◆fR1KiTvorM 2018/05/14(月) 21:12:36.66 ID:Z8v6AYYw
いえす

810 :132人目の素数さん:2018/05/27(日) 15:26:31.37 ID:WlCVafzc.net
一辺の長さが2である立方体ABCD-EFGHのABの中点をMとする。
対角線BHを軸とする半径1の円柱をC_1、直線MGを軸とする半径1の円柱をC_2とするとき、以下の問いに答えよ。

(1)xyz空間の円柱x^2+y^2=1を、x軸を含みx軸と角θで交わる平面αθで切る。その切り口の面積をθで表せ。
ただしθはxy平面からz軸の正の方向に回転した角度とし、0≦θ<π/2とする。

(2)C_1とC_2の共通部分の体積を求めよ。

811 :132人目の素数さん:2018/05/27(日) 16:06:06.42 ID:s9yJF/4c.net
(1)は(2)のヒントになってるかなぁ?2軸を含む平面で切った菱形の面積積分する方が楽な希ガス。やる気ないのでどうでもいいけど。

812 :132人目の素数さん:2018/05/27(日) 16:23:25.96 ID:CGYiTgTM.net
>>807
>集合族全体は256個。
2^2^3
>被覆がaを含まないのは16個
2^2^2
>被覆がa,bを含まないのが4個
2^2^1
>被覆が何にも含まないのが2個
2^2^0
空集合と空集合を含む集合族は除かないの?

813 :132人目の素数さん:2018/05/27(日) 16:24:59.33 ID:CGYiTgTM.net
>>803
それでいいよ
ただ
>>807
のように空集合を含む集合族を許すのなら倍

814 :132人目の素数さん:2018/05/27(日) 16:56:02.76 ID:MiYdiExp.net
追加ですいません、
(P→Q)→(R→notS)
を連言標準形にせよ。という問題ですが、
これ、Fになるパターンが1つしかないので選言標準形の方が1項のみで、逆に連言標準形が15項の論理積になるかと思ったのですがあってますか?

815 :132人目の素数さん:2018/05/27(日) 19:12:43.34 ID:yiDHP8Qn.net
>>814
門外漢なのでよくわかんないけどwikiに書いてある事を信じると

not (P→Q)→(R→notS)
= ((not p) and r and s) or (q and r and s)

の否定だから2項の積になるのでわ?

816 :132人目の素数さん:2018/05/27(日) 19:28:14.78 ID:DWIbh3ID.net
>>814
一般に4変数の標準形で15個も節は要らない
(P→Q)→(R→¬S)
=(P∧¬Q)∨(¬R)∨(¬S)
=(P∨¬R∨¬S)∧(¬Q∨¬R∨¬S)

817 :132人目の素数さん:2018/05/27(日) 19:30:31.02 ID:MiYdiExp.net
>>815
連言なので∧を使って繋げるんですよね
いろいろ考えたんですけどやっぱり選言標準形だとすっきり表せて、連言なら15通り出ると思いました…どうなんでしょう

818 :132人目の素数さん:2018/05/27(日) 19:32:14.36 ID:MiYdiExp.net
>>816
あ、そうかそういうことですか…
ちょっと勘違いしてたみたいですすいません
どうもありがとうございます

819 :132人目の素数さん:2018/05/27(日) 19:35:02.48 ID:WlCVafzc.net
(1)自然数nに対してn^2+1が10の倍数になるとき、nはどのような数かを述べよ。

(2)kを2でない自然数とする。n^2+1とn^k+1をともに10の倍数とするようなnが存在するとき、kはどのような数か。

820 :132人目の素数さん:2018/05/27(日) 19:39:12.47 ID:DWIbh3ID.net
>>817
Fが1通りの場合はむしろ簡単で、
例えばP∨Q∨R∨Sは選言標準形であり連言標準形でもある

821 :132人目の素数さん:2018/05/27(日) 22:07:49.48 ID:EdUtsj53.net
(1) mod 10でn^2≡9⇔n≡3,7(⇔n≡±3)
(2) kは非負整数
mod 10で
n≡3,7のときn^4≡1より
n^(4k+0)=((n^4)^k)*(n^0)≡1≡1
n^(4k+1)=((n^4)^k)*(n^1)≡n≡3,7
n^(4k+2)=((n^4)^k)*(n^2)≡n^2≡9
n^(4k+3)=((n^4)^k)*(n^3)≡n^3≡7,3

1の位に相当

822 :132人目の素数さん:2018/05/27(日) 22:09:57.61 ID:EdUtsj53.net
同じkを使ってしまったが察して

823 :132人目の素数さん:2018/05/28(月) 00:30:02.72 ID:35mGdcfM.net
ひてい

824 :132人目の素数さん:2018/05/28(月) 00:46:55.38 ID:35mGdcfM.net
ミスしました

述語論理についてなんですけど、
∀x Ey P(x,y)
とすると、すべてのxについてyが存在するかどうかについて考えるのが良いのですか?

例えば、P(x,y,z)がx+y=zだとして

∀x∈N , Ey∈N P(x,y,0)
の場合は、すべての自然数xに自然数yを足して0になるyが存在するかどうかを考えれば良いのでしょうか?
この場合だと、不可能?偽?どのように答えるのが良いのでしょうか?

825 :132人目の素数さん:2018/05/28(月) 00:48:57.32 ID:W2N/CyZK.net
どんなxを選んでも、y=-xと選べばx+y=0となるので、その命題は正しい命題ですね

826 :132人目の素数さん:2018/05/28(月) 01:01:37.62 ID:AujL21eY.net
一般に-xは自然数ではないので偽

827 :132人目の素数さん:2018/05/28(月) 01:04:10.94 ID:W2N/CyZK.net
自然数なんですね
ならそうですね

828 :132人目の素数さん:2018/05/28(月) 01:34:23.45 ID:BgTET7DV.net
a,b,cを正の実数とするとき、以下のA,Bの最小値を求めよ。
A={a/(b+c)}+{b/(c+a)}+{c/(a+b)}
A=ln{a/(b+c)}+ln{b/(c+a)}+ln{c/(a+b)}

829 :132人目の素数さん:2018/05/28(月) 01:54:37.16 ID:1VyhgqB1.net
Bはないな。あってもないけどww

830 :132人目の素数さん:2018/05/28(月) 06:32:59.95 ID:TTo2rnUU.net
>>728

A = (1/2){(2a+b+c)/(b+c) + (a+2b+c)/(c+a) + (a+b+2c)/(a+b) -3}
 = {(a+b)/(b+c)+(b+c)/(a+b)}/2 + {(b+c)/(c+a)+(c+a)/(b+c)}/2 + {(c+a)/(a+b)+(a+b)/(c+a)}/2 - 3/2
 ≧ 1 + 1 + 1 - 3/2
 = 3/2.
(*) x>0 ⇒ x + 1/x ≧ 2 を使った。

A = (a+b+c){1/(b+c) + 1/(c+a) + 1/(a+b)} - 3
 = {(b+c) + (c+a) + (a+b)}{1/(b+c) + 1/(c+a) + 1/(a+b)}/2 -3
 ≧ (3^2)/2 - 3   (←チェビシェフ or コーシー)
 = 3/2.

e^B = abc/{(b+c)(c+a)(a+b)}
 ≦ abc/{(2√bc)(2√ca)(2√ab)}
 = abc/(8abc)
 = 1/8,

∵ (b+c)(c+a)(a+b) - 8abc = a(b-c)^2 + b(c-a)^2 + c(a-b)^2 ≧ 0.

831 :132人目の素数さん:2018/05/28(月) 15:58:14.44 ID:7L/Au6Pq.net
たとえばフェルマー予想とかの
具体的な数の問題が数学を抽象化することで解けるようになるのってなんでなんですか?
具体的な方法でも解けるけど記述が長くなりすぎるから抽象的な記述をしているだけなのか
それとも抽象化以外の解法がないのか、どっちなんでしょうか?

832 :132人目の素数さん:2018/05/28(月) 18:53:36.94 ID:1GO2+eBu.net
>>831
>具体的な数の問題が数学を抽象化することで解けるようになるのってなんでなんですか?


833 :132人目の素数さん:2018/05/28(月) 19:12:36.08 ID:35mGdcfM.net
有向グラフで二項関係Rを考え、反射性、対称性、反対称性、推移性を答える問題

反射性とは、いわば自分への辺なのはわかります。
推移性に関しては、1→2, 2→3 があったら1→3もある、というようになっているかどうかもわかります。
対称性、および反対称性についてが不明です。

対称性とは、有向グラフでいうとどの部分を表しているのでしょうか?
1→2,2→1のようになっていることでしょうか?
そもそも、対称性と反対称性の有向グラフでの違いはどう考えるのでしょうか

834 :132人目の素数さん:2018/05/28(月) 19:15:23.87 ID:GtBRvjFW.net
問題による

835 :132人目の素数さん:2018/05/28(月) 21:42:31.68 ID:1GO2+eBu.net
>>833
反対称性って何だっけ?
a≦b∧a≧b→a=b
のこと?

836 :132人目の素数さん:2018/05/28(月) 23:04:25.69 ID:3gPI6er1.net
鎌倉の大仏の知能は、圧倒的世界一の超絶天才数学者をも凌駕しているのでしょうか?

837 :132人目の素数さん:2018/05/28(月) 23:29:01.34 ID:3gPI6er1.net
秘密曼陀羅十住心論を書いた空海は、東大理V首席よりも賢いですか?

838 :132人目の素数さん:2018/05/29(火) 01:29:02.66 ID:vRlAKW/L.net
>>835
それです、
対称性については
例えばノードaからbへの辺があるならb→aの辺があること。
つまるところ、関係行列としては対角成分を軸に対称的な位置の要素が1になってることなのはわかったんですけど、
反対称性についてが未だに理解出来てません…

839 :132人目の素数さん:2018/05/29(火) 01:30:12.90 ID:ZAB9lIJR.net
>>831
抽象化することによって見えていなかった性質が見えてきたり、既に研究が進んでいる他の分野を応用できたりするため
抽象化というのはある意味で本質や実体を捉えるためのステップ

840 :132人目の素数さん:2018/05/29(火) 01:52:54.81 ID:qEPn7ntH.net
>>838
ノードaからbへ、行って来いができたと思ったが、そんなことはなかったぜ
な、何のことを言っているのかわからないと思うが、俺も最初分からなかった
実際には俺は一歩も動いていなかったんだ

ってことだろ?

841 :132人目の素数さん:2018/05/29(火) 02:32:19.23 ID:cuIjcuNH.net
【ホリエモン】なんでみんな就職するの?やる気がない人ほど起業して利益率の高い仕事を選択し、有望な者に投資しろ
https://www.youtube.com/watch?v=y3WFObrOIoQ
ホリエモンのQ&A vol.155起業のすすめ
https://www.youtube.com/watch?v=2n1O4oUeIXg
堀江貴文「大企業に就職なんて、とっくにオワコン」「今の時代、金ですらオワコン」
https://www.youtube.com/watch?v=gSvIk_Bnwlo
堀江貴文の名言がすごい!「つまらない仕事なんか今すぐ辞めろ!楽しいことだけやれ!」
https://www.youtube.com/watch?v=4w3XOl5CoU8
堀江貴文 決められたレールの上を歩く⇒人生終了で、自殺者増える
https://www.youtube.com/watch?v=CYRo8o2Y_D8
【堀江貴文】※サラリーマン必見!君らいい加減仕事辞めたら?wはっきり言って全部無駄だ!!
https://www.youtube.com/watch?v=IgyRIVdvxhk
これからは個人の時代!ヒカルは話が上手いしヒカキンは編集が上手い。
これからの通貨の未来はどうなるのかも話そう
https://www.youtube.com/watch?v=4hQngvBCugA
個人が大金を稼ぐ!ライブ配信時代が本格的にやって来てその領域は
さらに拡大していき無名から著名になる人も増加する
https://www.youtube.com/watch?v=1H0R-kBtUOo

842 :132人目の素数さん:2018/05/29(火) 06:18:06.18 ID:U0bqQoKK.net
>>838
じゃ
異なるabの間にa→bがあるならb→aはないってことだよ

843 :132人目の素数さん:2018/05/29(火) 07:20:01.38 ID:vRlAKW/L.net
対称性の反対というか否定を考えるのか

844 :132人目の素数さん:2018/05/29(火) 13:54:48.07 ID:tflU+QS4.net
fをXからYの写像、ψをXの冪集合からYの冪集合への写像としたとき、
fが単射であることと、ψが単射であることが同値であることを示せ
という問題です。

fが単射→ψが単射
とその逆のψが単射→fが単射
の両方が成り立つことを示すのがわかりますが、
仮にfが単射→ψが単射とはどうやって示すことが出来るのでしょうか?

そもそも、単射を示すということがわかっていませんが、ここではfが単射であることは前提として、そこからψが単射であることを導くのでしょうか?
(もっと言うと、P→Qを示すと言うのは、
P⊆Q とその逆を示すことなので、4パターン示さなければならないという考えであっていますか?)

何が分からないのか分かってないのでどうか解説か、回答だけでもいいのでよろしくお願い致します。

845 :132人目の素数さん:2018/05/29(火) 14:02:09.98 ID:J8fJm/T7.net
示せるわけない。

846 :132人目の素数さん:2018/05/29(火) 14:08:59.90 ID:ag2nHzc1.net
有限集合の全単射の存在くらいだな

847 :132人目の素数さん:2018/05/29(火) 14:10:55.40 ID:o91+BNqn.net
>>844
元の問題のコピーをアップしてみな

848 :132人目の素数さん:2018/05/29(火) 14:51:59.78 ID:+BWlXE/G.net
fが単射なら f^(-1)(f(A)) = A

849 :132人目の素数さん:2018/05/29(火) 14:53:15.38 ID:TSoO8D7M.net
sin(n)を十進法表示したときの、小数点以下第一位の数を求めよ。

850 :132人目の素数さん:2018/05/29(火) 14:55:29.74 ID:ZeYVVUmu.net
>>844
fとψの関係は?

851 :132人目の素数さん:2018/05/29(火) 15:03:47.62 ID:LrJ8VHO5.net
これです、

https://i.imgur.com/SiPc8G3.jpg

852 :132人目の素数さん:2018/05/29(火) 16:10:40.64 ID:qEPn7ntH.net
844と851じゃ全然別の問題じゃねーかwww

853 :132人目の素数さん:2018/05/29(火) 16:40:19.13 ID:LrJ8VHO5.net
すいませんそれすら分かってないです。
ちょっと写像の根本から勉強しないとですね。。。

写像に触れないまま授業進んでしまってるので(泣)

854 :132人目の素数さん:2018/05/29(火) 16:49:08.48 ID:h5GyrdT7.net
写像なんて、関数じゃん。対象が数以外のものでも構わないだけ

855 :132人目の素数さん:2018/05/29(火) 17:27:20.01 ID:eh5lc/QA.net
答 解けません

856 :132人目の素数さん:2018/05/29(火) 17:32:16.81 ID:6ltIIHde.net
>>844
写像を学習したら、
"ψ" をどのように定義すれば >>851 記述の問題の趣旨に合致するかを考えてみよう。

857 :132人目の素数さん:2018/05/29(火) 17:58:05.46 ID:TSoO8D7M.net
以下の条件(1)(2)を満たすxの関数f(x)の例を1つ挙げ、それが条件を満たしていることを説明せよ。
(1)-∞<x<∞で何回でも微分可能である
(2)xy平面の曲線y=f(x)はちょうど3つの異なる変曲点を持ち、それらは同一直線上にある

858 :132人目の素数さん:2018/05/29(火) 18:31:40.94 ID:yW/K5Bs7.net
まず三次関数を用意します。以下容易なので読者の演習とする。

859 :132人目の素数さん:2018/05/29(火) 20:55:51.74 ID:8U3hRUxB.net
線形の本質ってなんですか?
グラフが直線になることですか?
交換法則や結合法則が成り立つことですか?

860 :132人目の素数さん:2018/05/29(火) 21:01:42.50 ID:1Wf45i++.net
線形になること

861 :132人目の素数さん:2018/05/30(水) 00:12:38.30 ID:nZstFZFd.net
直線代数

862 :132人目の素数さん:2018/05/30(水) 02:41:43.36 ID:LUI2iFYL.net
なぜか観てしまう!!サバイバル系youtuberまとめ
http://tokyohitori.hatenablog.com/entry/2016/10/01/102830
あのPewDiePieがついに、初心YouTuber向けに「視聴回数」「チャンネル登録者数」を増やすコツを公開!
http://naototube.com/2017/08/14/for-new-youtubers/
27歳で年収8億円 女性ユーチューバー「リリー・シン」の生き方
https://headlines.yahoo.co.jp/article?a=20170802-00017174-forbes-bus_all
1年で何十億円も稼ぐ高収入ユーチューバー世界ランキングトップ10
https://gigazine.net/news/20151016-highest-paid-youtuber-2015/
おもちゃのレビューで年間12億円! 今、話題のYouTuberは6歳の男の子
https://www.businessinsider.jp/post-108355
彼女はいかにして750万人のファンがいるYouTubeスターとなったのか?
https://www.businessinsider.jp/post-242
1億円稼ぐ9歳のYouTuberがすごすぎる……アメリカで話題のEvanTubeHD
https://weekly.ascii.jp/elem/000/000/305/305548/
専業YouTuberがYouTubeでの稼ぎ方具体的に教えます。ネタ切れしない方法は〇〇するだけ。
https://www.youtube.com/watch?v=Co9a9fHfReo
YouTubeで稼げるジャンルは〇〇動画です。YouTube講座
https://www.youtube.com/watch?v=_Nps8xb5czQ
富を築くYouTuber年収7億円〜15億円稼ぐツワモノも!
https://www.kaigainet.com/youtube10.html
月に推定26億円稼ぐ人も 低予算の動画で驚くほどの金額を手にする「ユーチューバー」トップ10
https://www.huffingtonpost.jp/2015/02/09/youtube-stars-huge-earnings_n_6642684.html

863 :132人目の素数さん:2018/05/30(水) 13:15:58.94 ID:My3C3KVt.net
If a baseball and a bat cost $1.10 together,
and the bat costs $1.00 more than the ball,
how much does the ball cost?

WRONG ANSWXER = 10&cent;
CORRECT ANSWER = 5&cent;

英語力の無さもあってどうして5セントになるか理由がわかりません。
よろしくお願いします。

864 :132人目の素数さん:2018/05/30(水) 13:18:20.51 ID:BuOX6i6+.net
野球ボールとバット合わせて1.10ドル、バットはボールより1ドル高い
ボールはいくら?

間違い 1.10-1=0.10
正しい0.05+1.05=1.1

865 :132人目の素数さん:2018/05/30(水) 15:36:48.22 ID:/iCOYSYN.net
バットにはイチローのサイン、ボールにはダルのサインがありました

866 :132人目の素数さん:2018/05/30(水) 16:24:40.16 ID:Zov7LODs.net
殺す殺す殺す殺す殺す殺す殺す殺す殺す殺す殺す殺す殺す殺す
殺す殺す殺す殺す殺す殺す殺す殺す殺す殺す殺す殺す殺す殺す
殺す殺す殺す殺す殺す殺す殺す殺す殺す殺す殺す殺す殺す殺す
殺す殺す殺す殺す殺す殺す殺す殺す殺す殺す殺す殺す殺す殺す 👀
Rock54: Caution(BBR-MD5:1341adc37120578f18dba9451e6c8c3b)


867 :132人目の素数さん:2018/05/30(水) 23:46:20.49 ID:tB7oIF94.net
∫∫∫ (x^2) / (x^2+y^2+z^2+1)^3
という積分が分かりません。教えてください。

868 :132人目の素数さん:2018/05/30(水) 23:53:34.89 ID:DRfOW4UB.net
>>867
>∫∫∫ (x^2) / (x^2+y^2+z^2+1)^3
∫∫∫ (x^2+y^2+z^2) / (x^2+y^2+z^2+1)^3 /3

869 :132人目の素数さん:2018/05/31(木) 05:07:35.91 ID:WOMjP57J.net
y=e^xの0≦x≦log2の部分の長さを求めよという問題が分かりません。
∫√(1+e^2x)dxなんて計算できるんですか?

870 :132人目の素数さん:2018/05/31(木) 06:04:27.65 ID:r643jL3Z.net
逆関数

871 :132人目の素数さん:2018/05/31(木) 06:56:43.58 ID:YExPTj9n.net
>>869
t=√(1+e^(2x))と置換
x=(1/2)log(t^2-1)
dx=t/(t^2-1)dt

872 :132人目の素数さん:2018/05/31(木) 10:25:15.29 ID:1i3xzGBS.net
>>867

D(R) = { (x,y,z) | xx+yy+zz ≦ RR } とする。
>>868 に従い
∫∫∫_D xx/(xx+yy+zz+1)^3 dxdydz
= (1/3)∫∫∫_D (xx+yy+zz)/(xx+yy+zz+1)^3 dxdydz
= (1/3) ∫[0,R] rr /(rr+1)^3 (4πrr)dr   (← 極座標)
= (π/6) ∫[0,R] 8(r^4)/(rr+1)^3 dr
= (π/6) [ 3arctan(r) - r(5rr+3)/(rr+1)^2 ](r=0,R)
= (π/6) { 3arctan(R) - R(5RR+3)/(RR+1)^2 }
→ (π/2)^2   (R→∞)

873 :132人目の素数さん:2018/05/31(木) 11:41:02.86 ID:1i3xzGBS.net
>>869

√{1+e^(2x)} = e^(2x)/√{1+e^(2x)} + 1/√{1+e^(2x)},

∴∫√{1+e^(2x)} dx = √{1+e^(2x)} + ∫1/√{1+e^(2x)} dx

>>871 を使って

(右辺第2項) = ∫1/√{1+e^(2x)} dx
 = ∫ 1/(tt-1)dt
 = (1/2)∫{1/(t-1) - 1/(t+1)} dt
 = (1/2)log{(t-1)/(t+1)}
 = …

874 :132人目の素数さん:2018/05/31(木) 12:40:06.52 ID:1i3xzGBS.net
>>869

>>871 を使って
= ∫[√2,√5] tt/(tt-1) dt
= ∫[√2,√5] { 1 + (1/2)[1/(1-t) - 1/(1+t)] } dt
= [ t + (1/2)log((t-1)/(t+1)) ](t=√2,√5)
= {√5 - log((√5 +1)/2)} - {√2 - log(√2 +1)}
= 1.222016177

875 :132人目の素数さん:2018/05/31(木) 13:15:23.81 ID:1i3xzGBS.net
>>857
 5次関数で可能…
 f(x) = x^5 -(10/3)aax^3 +bx +c,
 f "(x) = 20x(x+a)(x-a)
 変曲点
 (-a,(7/3)a^5 -ab+c)
 (0,c)
 (a,-(7/3)a^5 +ab+c)

876 :132人目の素数さん:2018/05/31(木) 14:31:51.48 ID:WOMjP57J.net
エスイーエックス

877 :132人目の素数さん:2018/05/31(木) 14:32:57.06 ID:WOMjP57J.net
y=se^xの0≦x≦log2の長さは?

878 :132人目の素数さん:2018/05/31(木) 21:58:32.01 ID:lSPNDXw4.net
無限級数Σ(((n!)^2)a^n)/(2n)!) (0<a)についての質問です
収束判定法でa<4と4<aの時で収束発散が変わることは分かったのですが
a=4の時はどう判定すればいいのでしょうか?

879 :132人目の素数さん:2018/05/31(木) 22:08:06.84 ID:r643jL3Z.net
収束半径ゼロだろ

880 :132人目の素数さん:2018/05/31(木) 22:11:52.49 ID:Mw5UFFh6.net
>>878
スターリングの公式を使うと、0<a<4のとき収束し、a≧4 のとき発散することが分かる。

881 :132人目の素数さん:2018/05/31(木) 22:36:42.69 ID:aZkDH4FO.net
まあa=4のときは
第n+1項=第n項×(2n+2)/(2n+1)
だから明らかに発散するけどね。
でもスターリングの公式使うのが本格的。

882 :132人目の素数さん:2018/05/31(木) 22:44:35.41 ID:ycBSSWv7.net
>>868,872
ありがとうございます。

883 :132人目の素数さん:2018/05/31(木) 23:12:44.22 ID:lSPNDXw4.net
>>880
>>881
ありがとうございます
スターリングの公式使って考えてみます

884 :132人目の素数さん:2018/06/01(金) 16:31:49.69 ID:4Skgtpo9.net
使う必要のないものにわざわざ使うのもどうかと
遊びならいいが

885 :132人目の素数さん:2018/06/01(金) 16:56:00.64 ID:Msi5d0uy.net
中心極限定理では、元の分布によって収束する速さが異なると思いますが、キュミュラントの視点から何が言及できますか?

886 :132人目の素数さん:2018/06/01(金) 20:17:42.63 ID:gsMzIl4U.net
1と書かれたカードがn枚、2と書かれたカードが2n枚、3と書かれたカードが3n枚、4と書かれたカードが4n枚、合計10n枚のカードがある。
この中から無作為に3n枚のカードを選び、それらを並べて3n桁の整数Nを作る。
Nが3の倍数となる確率と1/3の大小を比較せよ。

887 :132人目の素数さん:2018/06/02(土) 06:55:32.44 ID:qc99k5Fr.net
>>886

N ≡ (各桁の数字の和) (mod 9)

1または4のカード j枚
2 のカード k枚
3 のカード (3n-j-k)枚
のとき
 N ≡ j + 2k (mod 3)

888 :132人目の素数さん:2018/06/02(土) 10:58:20.86 ID:A868xrYW.net
で?

889 :132人目の素数さん:2018/06/02(土) 11:26:48.91 ID:SKZf7qZs.net
朗報、誤答爺さんが2チャンやめるって

75 名前:132人目の素数さん[sage] 投稿日:2018/06/01(金) 10:46:59.65 ID:wHdQHx/t [3/3]
(>>74の続き)
以後、私は2チャンには書かないことにする。今回はしっかりと明記する。
2チャンでゴタゴタさせられ濡れ衣を着せられたりして、巻き込まれるのが嫌になった。
2チャンでは、背理法の原理を教わったことが唯一の救いだ。

890 :132人目の素数さん:2018/06/02(土) 12:45:27.34 ID:/KhMK+Le.net
どこか記述に不備がありますか?

https://i.imgur.com/N1Ng72Y.jpg
https://i.imgur.com/BGlOrlr.jpg

891 :132人目の素数さん:2018/06/02(土) 14:18:00.23 ID:fg2B06o8.net
これこういうことですか?
https://imgur.com/a/QPqpfS7
θが一致するからこんな式になるだけで
点oに関係ないθってありえないのかな
左の三角形のcos、y/rかと思った

892 :132人目の素数さん:2018/06/02(土) 14:46:17.05 ID:Sh5silUc.net
なんかこれエロくない?
https://cdn.dribbble.com/users/81337/screenshots/3226084/rainbow_torus_spiral-01-png_rgba-800x600-imagemagick-gifsicle.gif

893 :132人目の素数さん:2018/06/02(土) 14:52:52.79 ID:fg2B06o8.net
>>892
えっち

894 :DJgensei artchive gemmar:2018/06/02(土) 15:10:14.50 ID:gd1gvViy.net
https://www.youtube.com/watch?v=fXJc2NYwHjw

895 :DJgensei artchive gemmar:2018/06/02(土) 15:18:34.33 ID:gd1gvViy.net
https://www.youtube.com/watch?v=uLlZ7mjHPRs
https://www.youtube.com/watch?v=Y5uYzVG0quU

896 :132人目の素数さん:2018/06/02(土) 17:03:04.97 ID:fg2B06o8.net
回答しねえなら意味不明な動画載せんなよ!!!!!

897 :132人目の素数さん:2018/06/02(土) 17:46:46.86 ID:pvQNq803.net
>>886
傑作なので誰か解いてください

898 :132人目の素数さん:2018/06/02(土) 21:07:36.50 ID:mWY0yoZc.net
9で割り切れる数は各桁の和が9の倍数
3で割り切れる数は各桁の和が3の倍数
とわかりました。

証明は出来ないのですが、11で割り切れるかどうかの判定は、
例えば、整数nがK桁だとして、順に桁を並べ2桁ずつで11の倍数を引いていき、最終的に0になるってことで合ってますか?

つまり、
121なら12-11=1.11-11=0よって割り切れる

123456784なら12-11=1、13-11=2、24-22=2、25-22=3、36-33=3、37-33=4、48-44=4、44-44=0
よって、割り切れる。
(しかも、11×整数を並べると1122,33,44で商が作れると思います)
これを一般化したものをご存知でしたら名前を教えていただけたら助かります。

899 :132人目の素数さん:2018/06/02(土) 21:13:56.60 ID:pvQNq803.net
>>886
これまじ傑作なので解いてください
感動すると思います
あまりの緻密さに

900 :132人目の素数さん:2018/06/02(土) 21:16:43.66 ID:iQRqDI5D.net
>>898
あってるよ
けどそれ割り算の筆算じゃ?

901 :132人目の素数さん:2018/06/02(土) 21:18:05.70 ID:iQRqDI5D.net
書きこんでから思ったけど、釣られてる?

902 :132人目の素数さん:2018/06/02(土) 21:48:18.52 ID:F1Wu7UKt.net
Kontsevichという馬鹿コテが物理板を荒らしてる、みんな来て

「物理数学の直感的方法」とかいう本
https://rio2016.5ch.net/test/read.cgi/sci/1523151554/

903 :132人目の素数さん:2018/06/02(土) 22:18:05.59 ID:iniu0Mjz.net
k上のベクトル空間Vでn次元のものというのは、ただk上のベクトル空間k^nの標準基底でない基底を採用したものという認識でオッケーですか?

904 :132人目の素数さん:2018/06/02(土) 22:19:52.79 ID:Iwqbrmxy.net
k^nでもいいしそうでなくてもいい
標準基底でもいいし別の基底でもいい

何故変な仮定をつけようとするのか

905 :132人目の素数さん:2018/06/02(土) 22:28:24.23 ID:hqzOfJsV.net
k^nには標準基底を入れて考えていると思っていたのですがそうではないんですか?
だとしたらわざわざVをk^nと書いているときはなぜ...(もちろん集合としての違うはあるのは知ってますが)

906 :132人目の素数さん:2018/06/02(土) 22:33:16.22 ID:Qe1TBQQS.net
体k上のベクトル空間Vには底が存在し、その底が有限個のとき、
どの底も同じ個数からなり、一つ底をさだめるごとに、その底を用いて
Vとk^nの同型な線形写像を構成することができる。

907 :132人目の素数さん:2018/06/02(土) 22:51:00.62 ID:hqzOfJsV.net
有限次元k-ベクトル空間の同型類は次元によって完全に決定される
しかしその同型の決め方はcanonicalじゃなく基底を定めないといけないからそれぞれk^nとかVとか(基底を込めて)違う記号で書いてると思ったのですが違う??

908 :132人目の素数さん:2018/06/03(日) 00:49:28.79 ID:LciQfjpc.net
>>899
計算機で計算したら1/3よりでかいときと小さい時が4周期前後で交代にでてくるけど、ぴったり4周期でもなく結構入り乱れてる。持ってる答えほんとに合ってる?

909 :132人目の素数さん:2018/06/03(日) 20:27:45.38 ID:SbqQM1O0.net
真理値表は書いたのですがこれの論理式ってわかる方いますか?

https://i.imgur.com/6ERI1T6.jpg
https://i.imgur.com/iJyB8sY.jpg

910 :132人目の素数さん:2018/06/03(日) 20:37:41.75 ID:FMlwT4S5.net
>>907
「Vは(有限次元)ベクトル空間」と言えばそれ以上の意味はない
Vが集合として何であるかは決めてないし、もちろん基底も固定していない

もう一度言うが、何で変な仮定をつけようとするの?
V≠(k^n,標準基底)とするなら具体例としてV=(k^n,標準基底)を考えることができなくなるけど、それでもいいの?

911 :132人目の素数さん:2018/06/03(日) 21:06:12.81 ID:GUNpMLhn.net
>>659の人かわかる人いますかー?
この方法でなんで求まるかが自分の中で理解出来ていません。
教えるの上手い人教えてください

912 :132人目の素数さん:2018/06/03(日) 21:55:01.60 ID:SbqQM1O0.net
3枚目の4行目なんですけど、なぜx+y「または」x-yは2pの倍数なのでしょうか?
「少なくとも」x+y、x-yのどちらかは2pの倍数
ではないのですか?

https://i.imgur.com/qzAiUXy.jpg
https://i.imgur.com/wqzRcFp.jpg
https://i.imgur.com/Wuqi5jE.jpg

913 :132人目の素数さん:2018/06/03(日) 22:42:51.58 ID:uThlUoHH.net
>>910
ここでのVは一般のVというわけではなく、自分が書いた意味での(k^n,標準基底)とは別の任意のn次元ベクトル空間(たとえば(k^n,標準基底じゃないやつ)とか)というつもりで書いてました

ええと、では係数体が固定されていて次元も確定(有限次元)している場合、そのベクトル空間をわざわざVと書く理由がわかりません(k^nしかないのに)

914 :132人目の素数さん:2018/06/03(日) 22:46:08.33 ID:8QbmVSQ9.net
>>912
数学用語での「または」には排他的な意味はありません。
A「または」B は  「A「かつ」B」も含みます。
つまり、A「または」B は AとBの少なくとも一方は、と同じ意味です。

915 :132人目の素数さん:2018/06/03(日) 22:59:12.52 ID:8QbmVSQ9.net
>>913
k^n は特に数ベクトル空間などとよばれる体k上のn次元ベクトル空間の具体例の一つです。
数ベクトル空間ではないn次元ベクトル空間としてよく現れるものに、
不定元xに関する体k上のn-1次以下の多項式全体のなす集合に通常の多項式の和の構造をい入れたものがあります。
当然ながらこれはk^nに線形同型であり、記号的には k[x](<n)などと書かれることもありますが、k^nではありません。

916 :132人目の素数さん:2018/06/03(日) 22:59:34.70 ID:SbqQM1O0.net
>>914
ああ、両方なんですか

できれば>>909もお願いします

917 :132人目の素数さん:2018/06/04(月) 01:22:27.31 ID:eLfureAF.net
>>866 >>897
j:1〜100での答え。出題者の持ってる答えと合ってる?なんか規則ありそうでなさそうで。
ホントに出題ミスじゃないの?そうじゃないなら考えてみるけど。

(1,LT),(2,LT),(3,GT),(4,GT),(5,GT),(6,GT),(7,LT),(8,LT),(9,LT),(10,LT),
(11,GT),(12,GT),(13,GT),(14,GT),(15,LT),(16,LT),(17,LT),(18,LT),(19,GT),(20,GT),
(21,GT),(22,GT),(23,LT),(24,LT),(25,LT),(26,LT),(27,GT),(28,GT),(29,GT),(30,GT),
(31,GT),(32,LT),(33,LT),(34,LT),(35,LT),(36,GT),(37,GT),(38,GT),(39,GT),(40,LT),
(41,LT),(42,LT),(43,LT),(44,GT),(45,GT),(46,GT),(47,GT),(48,LT),(49,LT),(50,LT),
(51,LT),(52,GT),(53,GT),(54,GT),(55,GT),(56,LT),(57,LT),(58,LT),(59,LT),(60,LT),
(61,GT),(62,GT),(63,GT),(64,GT),(65,LT),(66,LT),(67,LT),(68,LT),(69,GT),(70,GT),
(71,GT),(72,GT),(73,LT),(74,LT),(75,LT),(76,LT),(77,GT),(78,GT),(79,GT),(80,GT),
(81,LT),(82,LT),(83,LT),(84,LT),(85,GT),(86,GT),(87,GT),(88,GT),(89,GT),(90,LT),
(91,LT),(92,LT),(93,LT),(94,GT),(95,GT),(96,GT),(97,GT),(98,LT),(99,LT),(100,LT)

918 :132人目の素数さん:2018/06/04(月) 02:04:24.66 ID:t2ICP58s.net
正規分布の平均値周りの偶数次モーメントを求めてくださいませんか。
途中式を書いてくださると助かります。

σ^2nが出ることはわかるのですが、積分ができません。

919 :132人目の素数さん:2018/06/04(月) 02:24:08.29 ID:eLfureAF.net
>>918
σ=1のときは
E((x-0)^(2n))
=(1/√(2π))∫[-∞, ∞]x^(2n)exp(-x^2/2)dx
=(2/√(2π))∫[0, ∞]x^(2n)exp(-x^2/2)dx
=(2/√(2π))∫[0, ∞](2t)^(n)exp(-t)dt/(√(2t))
=(2^n/√(π))∫[0, ∞]t^(n-1/2)exp(-t)dt
=(2^n/√(π))Γ(n+1/2)
=(n-1)!!
一般はこれのσ^n倍。

920 :132人目の素数さん:2018/06/04(月) 03:49:09.19 ID:Ew3FIvyX.net
>>917
適当に作った問題だと思うよ
別にそれが悪いことじゃないけど
解いてもあまり面白くもないよね

921 :132人目の素数さん:2018/06/04(月) 04:10:29.17 ID:Utc1nkXv.net
>>920

でも本人の口ぶりだとキチンと解けて感動するとかいってたけどねぇ?
検算くらいしてから出せば余計な恥ずかしい思いしなくて済むのに。

922 :132人目の素数さん:2018/06/04(月) 06:19:15.13 ID:26vcFj3P.net
「いま〜と置く」の「いま」はなぜつけるんですか?
付けないと駄目なんですか?

923 :132人目の素数さん:2018/06/04(月) 09:02:58.52 ID:SYEVbRdt.net
A を平面の点の空でない集合とし、 f(x, y) を A で定義された関数とする。
平面の点の集合 S に対し、最大値の定理の証明の中だけで使う記号
A ≦ S と A > S を定義する。 A の任意の点 (x, y) に対し、 S に含まれる
A の点 (s, t) で f(x, y) ≦ f(s, t) をみたすものが存在するとき、 A ≦ S と
書く。 A の点 (x, y) で、 S に含まれる A の任意の点 (s, t) に対し
f(x, y) > f(s, t) となるものが存在するとき、 A > S と書く。記号 A ≦ S と
A > S の意味は関数 f(x, y) によって決まるものだが、記号からは省略した。

A が S の部分集合ならば A ≦ S である。 A は空集合ではないから、
A と S が交わらないならば A > S である。 A ≦ S ならば f(x, y) の最大値を
とる A の点で S に含まれるものがあるはずであり、 A > S ならば f(x, y) の
A での最大値をとる点は S には含まれない。

(1)と(2)のどちらが A ≦ S の定義でしょうか?

(1)
A ≦ S



∀(x, y) ∈ A, ∃(s, t) ∈ A ∩ S such that f(x, y) ≦ f(s, t)

(2)
A ≦ S



∃(s, t) ∈ A ∩ S, ∀(x, y) ∈ A such that f(x, y) ≦ f(s, t)

924 :132人目の素数さん:2018/06/04(月) 09:03:21.86 ID:SYEVbRdt.net
>>923

(1)だと解釈すると、

「A が S の部分集合ならば A ≦ S である。」

は成り立ちますが、

「A ≦ S ならば f(x, y) の最大値をとる A の点で S に含まれるものがある。」

は成り立ちません。

(2)だと解釈すると

「A が S の部分集合ならば A ≦ S である。」

は成り立ちませんが、

「A ≦ S ならば f(x, y) の最大値をとる A の点で S に含まれるものがある。」

は成り立ちます。

925 :132人目の素数さん:2018/06/04(月) 09:39:15.00 ID:DrdlgPon.net
本ぐらい自分で訂正して読め。

926 :132人目の素数さん:2018/06/04(月) 09:48:05.29 ID:NoT+jWYT.net
>>909
Y2=G1・X2+G2・X3+G3・X4
Y1=G1・X1+G2・X2+G3・X3
Y0=G1・X0+G2・X1+G3・X2
・はAND、+はORまたはXOR

927 :132人目の素数さん:2018/06/04(月) 15:54:42.50 ID:CW5QHAZO.net
sを与えられた無理数とする。
(1)任意の正の数εに対して、不等式0<|s-p|≦ε…(A)を満たす有理数pが存在することを示せ
(2)(1)においてεをある無理数に固定する。ただしεは有理数qを用いてε=qsとは表されないものとする。このとき、(A)を満たすpの最大値および最小値が存在するかを述べよ。

928 :132人目の素数さん:2018/06/04(月) 16:08:15.24 ID:CW5QHAZO.net
BC=1,AB=ACの二等辺三角形△ABCがある。
kを正の実数とし、ABをk:1に内分する点をK、内角∠BKCの2等分線をl、lと直線BCとの交点をLとする。

(1)Lは線分BC上にあることを示せ。

(2)l//ACのとき、KLの長さをkで表せ。

929 :132人目の素数さん:2018/06/04(月) 16:08:31.27 ID:KN0vzly3.net
何これ?最大値存在するか否かなんてs+εが有理数かって聞いてるだけちゃうの?

930 :132人目の素数さん:2018/06/04(月) 16:29:54.66 ID:SYEVbRdt.net
四角形 ABCD が、半径 65/8 の円に内接している。この四角形の周の長さが 44
で、辺 BC と辺 CD の長さがいずれも 13 であるとき、残りの2辺 AB と DA の長さ
を求めよ。

このような問題を解くとき、解があるとすれば、こうなるはずだというところまでで
解答としてはパーフェクトでしょうか?

931 :132人目の素数さん:2018/06/04(月) 17:29:06.71 ID:+L5w0iOl.net
アルティン環Aから環Bへの全射準同型があればBはアルティン環になります

証明はBの任意のイデアルIに対しf^-1(I)がAのイデアルになることから分かりますが、なぜ全射性が必要なのでしょうか

932 :132人目の素数さん:2018/06/04(月) 18:04:04.38 ID:CW5QHAZO.net
空間のx軸を中心軸とする半径1の円柱を、z軸の周りにθ回転させた円柱をC(θ)とする。
どのC(θ)にも含まれるような空間の点全体からなる領域をDとするとき、Dは球であるか。

933 :132人目の素数さん:2018/06/04(月) 18:13:39.06 ID:vjcUHvHA.net
宇宙飛行士とリーマン予想を証明した人はどっちの方が頭が良いですか?

934 :132人目の素数さん:2018/06/04(月) 19:08:08.45 ID:KN0vzly3.net
>>931

AからBへ準同型があるだけではだめに決まってるやん。
kが体、A=k、B=k[x]でfを自然なk射にするときBのどの真のイデアルIをとってもfによる引き戻しは0イデアルになる。

935 :132人目の素数さん:2018/06/04(月) 19:09:17.64 ID:KN0vzly3.net
>>932
はい

936 :132人目の素数さん:2018/06/04(月) 19:12:11.41 ID:nC9cJMea.net
ロピタルの定理って高校範囲で示せるのでは?

ロルの定理よりg'(x)≠0からg(b)≠g(a)である
そこで(f(b)-f(a))/(g(b)-g(a))=C(定数)と置く

関数F(x)=f(x)-f(a)-c{g(x)-g(a)}を考えてF(a)=F(b)である

ロルの定理からF'(c)=f'(c)-Cg'(c)=0かつa<c<bを満たすcの存在が認められる

又f(x)とg(x)の定義よりf(a)=g(a)=0である
∴f(b)/g(b)=f'(c)/g'(c)かつa<c<bを満たすcが存在する

はさみうちの原理からb→a+0のときc→a+0である
∴lim[b→a+0]f(b)/g(b)=lim[c→a+0]f'(c)/g'(c)

x→a-0は同様に, 両者を使えばx→aも示せる

∴主張(ロピタルの定理)が示された

937 :132人目の素数さん:2018/06/04(月) 19:53:24.25 ID:cy8DIAit.net
どうみても場違いなわからない問題があるのですが(中学レベル)、適応するスレをどなたか教えていただけませんか

938 :132人目の素数さん:2018/06/04(月) 20:16:35.13 ID:tzM+Pvvj.net
>>930
いいえ。

外接円の半径を R とすると
 AB = 2R sin(∠AOB/2),
 BC = 2R sin(∠BOC/2),
 CD = 2R sin(∠COD/2),
 DA = 2R sin(∠DOA/2),
また題意より
 R = 65/8,
 BC = CD = 13,
 AB+BC+CD+DA = 44,
したがって
 AB + DA = 44 -13 -13 = 18,
 ∠AOB/2 = 2arctan(4/7) = arcsin(56/65) = 59.4897626゚
 ∠BOC/2 = 2arctan(1/2) = arcsin(4/5) = 53.130102゚
 ∠COD/2 = 2arctan(1/2) = arcsin(4/5) = 53.130102゚
 ∠DOA/2 = 2arctan(1/8) = arcsin(16/65) = 14.25003゚
よって
 AB = 14
 DA = 4

939 :132人目の素数さん:2018/06/04(月) 20:30:38.81 ID:SYEVbRdt.net
>>938

でも問題集の解答には、解があるとすれば、こうなるはずだというところまでしか
書いてありません。

940 :132人目の素数さん:2018/06/04(月) 20:32:21.86 ID:SYEVbRdt.net
四角形 ABCD が、半径 65/8 の円に内接している。この四角形の周の長さが 44
で、辺 BC と辺 CD の長さがいずれも 13 である

と書いてあるので、存在のほうは出題者が保証している形の問題ではないでしょうか?

941 :132人目の素数さん:2018/06/04(月) 21:00:27.26 ID:1w5RTCiM.net
>>940
受験数学ならそうだな。その手のやつは存在証明しなくても点はもらえる。
しかし満点もらえるんだからこれでOKとか思ってる奴はソコソコ止まり。

942 :132人目の素数さん:2018/06/04(月) 21:34:33.17 ID:N4QA1GQW.net
>>939
その問題集の解答のスクショを見せろ
ちなみに東大の問題なので探せば幾らでも載ってる本が見つかる
疑問があるならそれらも参照すればいいんじゃね

943 :132人目の素数さん:2018/06/04(月) 22:09:21.37 ID:SYEVbRdt.net
>>923-924

「A の任意の点 (x, y) に対し、 S に含まれる A の点 (s, t) で
f(x, y) ≦ f(s, t) をみたすものが存在する」の意味ですが、これ
を素直に解釈した(1)の意味らしいです。

「A ≦ S ならば f(x, y) の最大値をとる A の点で S に含まれるものがある。」

は本当に成り立ちますか?

(1)
A ≦ S



∀(x, y) ∈ A, ∃(s, t) ∈ A ∩ S such that f(x, y) ≦ f(s, t)

(2)
A ≦ S



∃(s, t) ∈ A ∩ S such that ∀(x, y) ∈ A, f(x, y) ≦ f(s, t)

944 :132人目の素数さん:2018/06/04(月) 22:12:15.13 ID:nCZ3kvV0.net
(スクショという単語の意味)

945 :132人目の素数さん:2018/06/04(月) 22:12:35.63 ID:SYEVbRdt.net
>>942

どんな問題集でも、

>>930

の問題に限らず、他の問題でも存在のほうは出題者が
保証しているから示さないという立場の解答ばかりである
といってもいいような状況ではないでしょうか?

ですので、わざわざ画像をアップロードするまでもないように
思いますが、そうではない問題集も存在するのでしょうか?

946 :132人目の素数さん:2018/06/04(月) 22:14:25.88 ID:SYEVbRdt.net
>>923-934
>>943

正誤表を見てみてもこの件については書いてありませんでした。

947 :132人目の素数さん:2018/06/05(火) 00:41:33.11 ID:+e1CS0n2.net
まぁ残念ながら>>945の言ってることは正しいな。
存在性の確認なんか全く無意味のような解答のほうが正しいような風潮が受験数学の世界にははびこってる。
しかし、だから存在証明の抜けてる解答が数学的に正しいわけではないし、その解答で納得してるような生徒は結局大学では行き詰まる。
ホントに理系で一歩抜け出ることができるのはそんな解答よんで少なくとも ”なんかおかしい” と思える人間に限られる。

948 :132人目の素数さん:2018/06/05(火) 00:44:07.19 ID:SY5SVVbZ.net
これこれこういうときどうなりますか?って聞かれてるのに、本当にそういう状況が成しえるかどうかを気にする人は、バカな人です
少なくとも国語はできないんでしょうね

949 :132人目の素数さん:2018/06/05(火) 01:01:08.77 ID:SY5SVVbZ.net
Q:ドラえもんとコロ助が戦ったらどっちが勝つんですか?

A:どっちも架空の存在なので、両者が戦うという事象は起こるはずがないため質問自体が無意味

はぁ?て感じになりますよね
>>930のような問題で存在性にぐちぐち言うということは、このようなことなわけですね

950 :132人目の素数さん:2018/06/05(火) 01:01:23.61 ID:+e1CS0n2.net
行き詰まったのがなんか言ってるなwww

951 :132人目の素数さん:2018/06/05(火) 01:03:32.24 ID:k73mhkxG.net
「 P という前提条件のもとで、Q という結論が成り立つことを示せ 」

という問題 ―― すなわち、「 P⇒Q 」が真であることを示せという問題に対して、

「 P を用いて Q を導いただけではダメ。P が真であることまでチェックしなければ不完全。大学では行き詰まる」

などと考えるのはただのバカ。なんで「 P⇒Q 」を示すときに「 P の 」真偽を気にするんだよw

・ P が偽のときは「 P⇒Q 」は真。
・ P が真のときは、Q が真であるときに限り「 P⇒Q 」は真。

従って、我々が気にすべきは「 Q の 」真偽だけであり、
Qを導いだたけで完全な解答になるのであり、

「 P が真であることまでチェックしなければ不完全。大学では行き詰まる 」

なんてことにはならないんだよ。むしろ P の真偽を気にしてるバカの方が行き詰まるわ。
「 P⇒Q 」が何を表しているのか理解してない証拠だからな。

952 :132人目の素数さん:2018/06/05(火) 01:08:20.35 ID:+e1CS0n2.net
>>951に書いてありような反論は聞き飽きてるんだよ。それ自分が発見した新解釈だとでも思ってる時点でおめでたいんだよ。
まぁ、数学っぽいことやって自己満足しとけ。

953 :132人目の素数さん:2018/06/05(火) 01:13:39.35 ID:k73mhkxG.net
>>952
意味が分からない。自分が発見した新解釈ってどういうこと?
>>951なんて常識だろ?「 P⇒Q 」について書いてるだけだよ?

そんな常識が「聞き飽きてる」ってのも意味不明。
聞き飽きてるくらいに「 P⇒Q 」のことを把握してるなら、
そもそも>>947みたいなバカな間違いはしないよね?

954 :132人目の素数さん:2018/06/05(火) 01:14:04.09 ID:SY5SVVbZ.net
素直にならばがわかりませんでした、って認めたらどうですか?

955 :132人目の素数さん:2018/06/05(火) 01:15:27.89 ID:+e1CS0n2.net
>>954
はいはい。わかりませんでした。君の勝ち〜。おめでと〜〜

956 :132人目の素数さん:2018/06/05(火) 01:20:46.27 ID:SY5SVVbZ.net
今日も「解いた側」の圧勝かぁ・・・。
毎日毎日、ラクラク解ける問題ばかりだから常勝なんだよね・・・。
たまには、解けない解けないっと悩んで負けてみたい、それが今の切実な悩み。

957 :132人目の素数さん:2018/06/05(火) 01:36:31.02 ID:HmwKrHOU.net
本を読んで少しは謙虚になれ

958 :132人目の素数さん:2018/06/05(火) 01:52:03.86 ID:K/6mvYAb.net
y=(exp(-a*x))*(b+(c+a*b)*x)

いきないりですけど、これの逆関数が計算できません。
つまりxに付いて解くにはどうやったらいいでしょうか。

959 :132人目の素数さん:2018/06/05(火) 01:58:03.26 ID:+e1CS0n2.net
解ける問題ばっかりに囲まれて羨ましいかぎりだなぁ

960 :132人目の素数さん:2018/06/05(火) 02:07:58.92 ID:HmwKrHOU.net
>>958
計算してないからわかんないけど、expならとりあえず両辺に対数とってみる

961 :132人目の素数さん:2018/06/05(火) 07:02:08.26 ID:RI7aB28L.net
>>886 >>897 >>899

>>887 より
1または4のカード j枚
2 のカード k枚
3 のカード (3n-j-k)枚
のとき
 N ≡ j+2k ≡ j-k (mod 3)
∴ j-k ≡ 0 (mod 3) となる確率を考える。

P(n) = {1/C[10n,3n]} Σ[0≦k≦2n,0≦j+k≦3n] C[5n,j] C[2n,k] C[3n,3n-j-k] {ω^(j-k) + ω^(k-j) + 1}/3,
ω = (-1+i√3)/2  … 1の3乗根。

P(1) = 1/3 + 1/C[10,3]
P(2) = 1/3 + 1/C[20,6]

962 :132人目の素数さん:2018/06/05(火) 07:10:52.13 ID:1s9rbgtS.net
四角形 ABCD が、半径 65/8 の円に内接している。この四角形の周の長さが 44
で、辺 BC と辺 CD の長さがいずれも 13 であるとき、残りの2辺 AB と DA の長さ
を求めよ。

「四角形 ABCD が、半径 65/8 の円に内接している。この四角形の周の長さが 44
で、辺 BC と辺 CD の長さがいずれも 13 である」

と言っているので、やはりそのような四角形が存在するが、その2辺 AB と DA の
長さを求めよ、と言っていますよね。

もし、この問題でそのような四角形がそもそも存在しないとすると、出題ミスという
ことになるかと思います。 👀
Rock54: Caution(BBR-MD5:1341adc37120578f18dba9451e6c8c3b)


963 :132人目の素数さん:2018/06/05(火) 07:16:15.14 ID:lLmH4xhF.net
>>958
>これの逆関数が計算できません。
W関数

964 :132人目の素数さん:2018/06/05(火) 07:55:14.05 ID:RI7aB28L.net
>>958

いきなりですけど
X = -a {b/(c+ab) + x},
とおくと
 exp(-ax) = exp(X) exp{ab/(c+ab)},
 b + (c+ab)x = -{(c+ab)/a}X,
辺々かけて
 y = -{(c+ab)/a} exp{ab/(c+ab)} X・exp(X),
 X・exp(X) = -{a/(c+ab)} exp{-ab/(c+ab)} y,
∴ X = W(-{a/(c+ab)} exp{-ab/(c+ab)} y) … Lambert のW函数
 x = -X/a - b/(c+ab) に入れる。

a=0 のときは y = b+cx
c+ab=0 のときは y = b・exp(-ax)

965 :132人目の素数さん:2018/06/05(火) 08:04:00.48 ID:SY5SVVbZ.net
>>962
Q:ドラえもんとコロ助が戦ったらどっちが勝つんですか?

A:どっちも架空の存在なので、両者が戦うという事象は起こるはずがないため質問自体が無意味

はぁ?て感じになりますよね
>>930のような問題で存在性にぐちぐち言うということは、このようなことなわけですね

966 :132人目の素数さん:2018/06/05(火) 08:30:31.10 ID:1s9rbgtS.net
実数 x は2次方程式 x^2 = -1 を満たしている。

x^4 の値を求めよ。

967 :132人目の素数さん:2018/06/05(火) 08:35:55.84 ID:SY5SVVbZ.net
1ですね

968 :132人目の素数さん:2018/06/05(火) 08:43:21.78 ID:SY5SVVbZ.net
1+1の答えを求めるのに、自然数の和の定義の無矛盾性から考えますか、って話ですね

969 :132人目の素数さん:2018/06/05(火) 09:04:30.97 ID:RI7aB28L.net
>>967

http://www.discogs.com/ja/release/6994946-それはないよ子猫ちゃん-しあわせ色の猫/images
http://page.auctions.yahoo.co.jp/jp/auction/f235003951
http://blogs.yahoo.co.jp/hm6666hero/GALLERY/show_image.html?id=12527955&no=6

970 :132人目の素数さん:2018/06/05(火) 10:37:37.51 ID:7Wnt1KgV.net
添付の画像の問題を教えていただきたいです
(書き込みが多くあり見辛い画像となってしまい申し訳ありません)
2枚目はzに対する微分方程式を立ててみたのですが、自信がありません…
何卒宜しくお願い致しますm(_ _)m

https://i.imgur.com/Eex87JJ.jpg
https://i.imgur.com/Eex87JJ.jpg

971 :132人目の素数さん:2018/06/05(火) 10:39:11.84 ID:7Wnt1KgV.net
画像間違えました
2枚目はこちらです
https://i.imgur.com/gpyPSD9.jpg

972 :スイカ割り:2018/06/05(火) 11:05:10.39 ID:y5eMxvts.net
2÷3は割りきれないけど、実際に長さ2mの(1次元的な)棒を3等分しようとしようとすれば出来るの?
それから3等分してできた棒の1つの長さを計ると何mになるの?
教えて!goo

973 :132人目の素数さん:2018/06/05(火) 11:07:44.66 ID:CPUOXfyJ.net
不確定性原理により正しい真の値というのは測定不可能です
残念でしたね

974 :132人目の素数さん:2018/06/05(火) 15:01:04.11 ID:/OSKgl8g.net
>>972
https://oshiete.goo.ne.jp/

975 :132人目の素数さん:2018/06/05(火) 18:44:00.76 ID:r5HYSakC.net
「0でない2つの関数f(x)とg(x)を用いてf(x)g(x)=0となるようなものを一組挙げよ」

お願いします。

976 :132人目の素数さん:2018/06/05(火) 18:49:30.61 ID:y93ap0Jy.net
異なる2点を取ってそれぞれ一点を除いてf=0,g=0

977 :132人目の素数さん:2018/06/05(火) 19:05:09.90 ID:vwYy1rXt.net
わら

978 :132人目の素数さん:2018/06/05(火) 19:45:43.93 ID:JrF6lphB.net
リーマン予想とP≠NP予想を証明したいのですが、とりあえず数学と物理学と計算機科学の全分野を網羅した方が良いのでしょうか?

979 :132人目の素数さん:2018/06/05(火) 21:22:09.01 ID:MR5aOihI.net
AをR^nの凸集合、BをAの閉包の内点とするとき、B⊂Aを示せ
どのようにやればいいのでしょうか

980 :132人目の素数さん:2018/06/05(火) 21:46:47.26 ID:SY5SVVbZ.net
閉包と内点の定義を述べてください

981 :132人目の素数さん:2018/06/05(火) 22:22:19.86 ID:y93ap0Jy.net
位相知らないなら黙ってればいいのに、と一瞬思ってしまった

>>979
問題文は正確に書いてね

982 :132人目の素数さん:2018/06/05(火) 22:27:27.97 ID:ekSWY8PA.net
「次の命題が成り立つことを対偶を用いて証明せよ
x,yがともに正の数のとき、x^2+y^2≧6 ならば x≧√3 または y≧√3である」
これって元の命題成り立ちますか?

983 :132人目の素数さん:2018/06/05(火) 22:27:52.13 ID:SY5SVVbZ.net
問題文は正確でしょう

984 :132人目の素数さん:2018/06/05(火) 22:28:29.86 ID:SY5SVVbZ.net
>>982
はい

985 :132人目の素数さん:2018/06/05(火) 22:32:40.58 ID:ekSWY8PA.net
ありがとうございます

986 :132人目の素数さん:2018/06/05(火) 22:35:56.98 ID:+ryOilXm.net
>>982
xy平面に図示すれば一発

987 :132人目の素数さん:2018/06/05(火) 22:58:43.52 ID:lrQo+Jb0.net
分からない問題はここに書いてね444
https://rio2016.5ch.net/test/read.cgi/math/1528207105/

988 :132人目の素数さん:2018/06/05(火) 23:34:16.84 ID:PbqFpKWz.net
内点は点

989 :132人目の素数さん:2018/06/06(水) 00:27:36.34 ID:n39uR33J.net
>>979
http://web.econ.keio.ac.jp/staff/ito/pdf03/me03sepa.pdf

990 :132人目の素数さん:2018/06/06(水) 00:29:01.43 ID:/3pfmLjy.net
>>980
>>981
ああ、すいません
内点全体の集合(内部)です

正しい問題文は以下のとおりです

AをR^nの凸集合
Bを、Aの閉包の内点全体の集合とするとき、B⊂Aを示せ

991 :132人目の素数さん:2018/06/06(水) 01:44:15.29 ID:I5g3t//e.net
>>990
B-A∋xとするとxの近傍で全部Aの触点なのがある

992 :132人目の素数さん:2018/06/06(水) 06:23:47.51 ID:xxwxn7ab.net
>>975

xが自然数または0のとき  f(x) = 1,g(x) = 0,
xが負の整数のとき     f(x) = 0,g(x) = 3,
xが有理数(≠整数)のとき f(x) = 5,g(x) = 0,
xが代数的無理数のとき   f(x) = 0,g(x) = 6,
xが超越的実数のとき    f(x) = 7,g(x) = 0,
xが複素数(≠実数)のとき f(x) = 0,g(x) = 2,
xが4元数のとき      f(x) = 0,g(x) = 4,
xが8元数のとき      f(x) = 0,g(x) = 8,

>>982

2・Max{xx-3,yy-3} ≧ (xx-3) + (yy-3) = xx+yy - 6,

993 :132人目の素数さん:2018/06/06(水) 17:02:28.41 ID:xxwxn7ab.net
>>992

xが16元数のとき  f(x) = a,g(x) = b, (←a,b は零因子)

994 :132人目の素数さん:2018/06/06(水) 18:34:09.94 ID:Q1+o1co8.net
1個のさいころを3n回続けて振り,
出た目の数の和のS, 二乗の総和をTとする.
条件3|Tのもとで, 3|Sである条件付き確率を求めよ.

この問題を教えてください

995 :132人目の素数さん:2018/06/06(水) 19:19:20.34 ID:R4P7zQR3.net
高校数学やん

996 :132人目の素数さん:2018/06/06(水) 19:38:50.86 ID:i4OBaZ0i.net
>>994
何これ解けない

997 :132人目の素数さん:2018/06/06(水) 22:57:18.67 ID:eT3pen36.net
>>994
P(3|T)
=(1/3+2/3)^3n + (1/3 + 2ω/3)^3n + (1/3 + 2ω^2/3)^3n
=1 + (√3/9i)^n + (-√3/9i)^n

P(3|T & 3|S)
=(1/3+1/3+1/3)^3n + (1/3+ω/3+1/3)^3n + (1/3+ω^2/3+1/3)^3n
+(1/3+1/3+ω/3)^3n + (1/3+ω/3+ω/3)^3n + (1/3+ω^2/3+ω/3)^3n
+(1/3+1/3+ω^2/3)^3n + (1/3+ω/3+ω^2/3)^3n + (1/3+ω^2/3+ω^2/3)^3n
=1+3(√3/9i)^n + 3(-√3/9i)^n

998 :132人目の素数さん:2018/06/07(木) 02:05:34.17 ID:4qUpYzhu.net
うんこぶりぶり。

999 :132人目の素数さん:2018/06/07(木) 06:43:56.49 ID:LV2DjXT6.net
>>994

>>961 より
1または4の目 j回
2または5の目 k回
3または6の目 (3n-j-k)回
出たとき
 S ≡ j-k (mod 3)
 T ≡ j+k (mod 3)

P(3|T) = P(j+k≡0)
 = Σ[L=0,3n] C[3n,L] (2/3)^L (1/3)^(3n-L) {1 +ω^L +ω^(-L)}/3
 = {(1/3 + 2/3)^(3n) + (1/3 + 2ω/3)^(3n) + (1/3 + 2/(3ω))^(3n)}/3
 = {1 + (i/√3)^(3n) + (-i/√3)^(3n)}/3
 = {1 + 2(1/3)^(3n/2)cos(nπ/2)}/3,

P(3|T ∧ 3|S) = P(j≡0 ∧ k≡0)
 = Σ[0≦j+k≦3n] (3n)!/{j! k! (3n-j-k)!} (1/3)^(3n) {1+ω^j +ω^(-j)}/3・{1+ω^k +ω^(-k)}/3
 = {(1/3+1/3+1/3)^(3n) + 2(1/3+1/3+ω/3)^(3n) + 2(1/3+1/3+1/3ω)^(3n) + (1/3+ω/3+ω/3)^(3n) + (1/3+1/3ω+1/3ω)^(3n) + 2(1/3+ω/3+1/3ω)^(3n)}/9
 = {1 + 2((2+ω)/3)^(3n) + 2((2+1/ω)/3)^(3n) + ((1+2ω)/3)^(3n) + ((1+2/ω)/3)^(3n) + 0^(3n)}/9
 = …

1000 :132人目の素数さん:2018/06/07(木) 13:54:52.19 ID:DXwRCU0Z.net
q

1001 :2ch.net投稿限界:Over 1000 Thread
2ch.netからのレス数が1000に到達しました。

総レス数 1001
286 KB
掲示板に戻る 全部 前100 次100 最新50
read.cgi ver.24052200